Download as pdf or txt
Download as pdf or txt
You are on page 1of 99

Parabola

5. PARABOLA 363-459
LEARNING PART — Intersection of a Line and a Parabola
Session 1 — Equation of Tangent in Different Forms
— Introduction — Point of Intersection of Tangents at any Two
— Conic Section Points on the Parabola
— Section of a Right Circular Cone by Different — Equation of Normals in Different Forms
Planes — Point of Intersection of Normals at any Two
— Conic Section : Definition Points on the Parabola
— Equation of Conic Section — Relation Between ‘t1’ and ‘t2’ if Normal at ‘t1’
— Recognisation of Conics meets the Parabola Again at ‘t2’
— How to Find the Centre of Conics — Co-normal Points
— Parabola : Definition — Circle Through Co-normal Points
— Standard Equation of Parabola Session 3
— Some Terms Related to Parabola — Pair of Tangents SS1 = T 2
— Other forms of Parabola with — Chord of Contact
Latusrectum 4a — Equation of the Chord Bisected at a
— Smart Table Given Point
— General Equation of a Parabola — Diameter
— Equation of Parabola if Equation of — Lengths of Tangent, Subtangent, Normal and
axis, Tangent at Vertex and Latusrectum Subnormal
are given — Some Standard Properties of the Parabola
2
— The Generalised form (y-k) = 4a (x-h) — Reflection Property of a Parabola
— Parabolic Curve — Study of Parabola of the Form
2
Session 2 (ax + by) + 2gx + 2fy + c = 0
— Position of a Point (x1 , y1 ) with respect to a PRACTICE PART
Parabola y2 = 4ax — JEE Type Examples
— Parametric Relation between the Coordinates — Chapter Exercises
of the Ends of a Focal Chord of a Parabola
CHAPTER

05
Parabola
Learning Part
Session 1
● Introduction ● Conic Section
● Section of a Right Circular Cone by Different Planes ● Conic Section : Definition
● Equation of Conic Section ● Recognisation of Conics
● How to Find the Centre of Conics ● Parabola : Definition
● Standard Equation of Parabola ● Some Terms Related to Parabola
● Other forms of Parabola with Latusrectum 4a ● Smart Table
● General Equation of a Parabola ● Equation of Parabola if Equation of axis, Tangent
at Vertex and Latusrectum are given
● The Generalised form (y - k ) 2 = 4 a ( x - h) ● Parabolic Curve

Session 2
● Position of a Point ( x , y ) with respect to a Parabola ● Parametric Relation between the Coordinates of
1 1
y 2 = 4 ax the Ends of a Focal Chord of a Parabola
● Intersection of a Line and a Parabola ● Equation of Tangent in Different Forms
● Point of Intersection of Tangents at any Two Points ● Equation of Normals in Different Forms
on the Parabola
● Point of Intersection of Normals at any Two Points ● Relation Between ’ t1 ’ and ‘ t 2 ’ if Normal at ‘ t1 ’
on the Parabola meets the Parabola Again at ’t 2 ’
● Co-normal Points ● Circle Through Co-normal Points
Session 3
● Pair of Tangents SS
2
1 =T
● Chord of Contact
● Equation of the Chord Bisected at a Given Point ● Diameter
● Lengths of Tangent, Subtangent, Normal and Subnormal ● Some Standard Properties of the Parabola
● Reflection Property of a Parabola ● Study of Parabola of the Form
(ax + by ) 2 + 2 gx + 2 fy + c = 0

Practice Part
● JEE Type Examples ● Chapter Exercises

Arihant on Your Mobile !


Exercises with the #L
symbol can be practised on your mobile. See inside cover page to activate for free.
Session 1
Introduction, Conic Section, Section of a Right Circular
Cone by Different Planes, Conic Section : Definition, Equation
of Conic Section, Recognisation of Conics, How to Find the
Centre of Conics, Parabola : Definition, Other forms of Parabola
with Latusrectum 4a, General Equation of a Parabola, The
Generalised form (y-k)2 = 4a (x-h), Parabolic Curve

Introduction Section of a Right Circular


The famous Greek mathematician Euclid, the father of
creative Geometry, near about 300 BC considering various
Cone by Different Planes
plane sections of a right circular cone found many curves, 1. Section of a right circular cone by a plane which is
which are called conics or conic sections. passing through its vertex is a pair of straight lines,
lines always passes through the vertex of the cone.

Conic Section V Plane

Let l 1 be a fixed vertical line and l 2 be another line


intersecting it at a fixed point V and inclined to it at an
angle a.
P
Axis
Q
O′ 2. Section of a right circular cone by a plane which
l1
l2 parallel to its base is a circle.
Upper Generator
α V
nappe
V Plane

Lower
nappe
O Circle
Circular base
O

Suppose we rotate the line l 2 around the line l 1 in such a 3. Section of a right circular cone by a plane which is
way the angle remains constant then, the surface parallel to a generator of the cone is a parabola.
generated is a double-napped right circular hollow cone. Plane
V
The point V is called the vertex, the line l 1 is the axis of
the cone. The rotating line l 2 is called a generator of the
cone. The vertex separates the cone into two parts called
nappes. The constant angle a is called the semi-vertical
angle of the cone. Parabola

O
Chap 05 Parabola 365

4. Section of a right circular cone by a plane which is The fixed point is called the focus of the conic and this
not parallel to any generator and not parallel or fixed line is called the directrix of the conic. Also, this
perpendicular to the axis of the cone is an ellipse. constant ratio is called the eccentricity of the conic and is
V
denoted by e.
SP
Plane In the figure, = constant = e
PM
Þ SP = e PM
Ellipse

O
Equation of Conic Section
5. Section of a right circular cone by a plane which is If the focus is (a,b) and the directrix is ax + by + c = 0, then
parallel to the axis of the cone is a hyperbola. the equation of the conic section whose
eccentricity = e is SP = e PM
O′

M P(x, y)

ax + by + c = 0
Hyperbola V

S (α, β)
Axis

|ax + by + c |
3D View Þ ( x - a ) 2 + (y - b) 2 = e ×
(a 2 + b 2 )

(ax + by + c ) 2
Circle Ellipse
Þ ( x - a ) 2 + (y - b) 2 = e 2 × .
(a 2 + b 2 )

Important Terms
Axis The straight line passing through the focus and
perpendicular to the directrix is called the axis of the conic
section.
Vertex The points of intersection of the conic section
Parabola Hyperbola
and the axis is (are) called vertex (vertices) of the conic
section.
Focal Chord Any chord passing through the focus is
Conic Section : Definition called focal chord of the conic section.
The locus of a point which moves in a plane such that the Double Ordinate A straight line drawn perpendicular to
ratio of its distance from a fixed point to its perpendicular the axis and terminated at both end of the curve is a
distance from a fixed straight line is always constant, is double ordinate of the conic section.
known as a conic section or a conic.
Latusrectum The double ordinate passing through the
M P focus is called the latusrectum of the conic section.
Centre The point which bisects every chord of the conic
passing through it, is called the centre of the conic section.
S (focus)
Directrix

Remark
Parabola has no centre but circle, ellipse and hyperbola have
centre.
366 Textbook of Coordinate Geometry

y Example 1 Find the locus of a point, which moves The nature of the conic section depends upon the position
such that its distance from the point (0, –1) is twice its of the focus S with respect to the directrix and also upon
distance from the line 3x + 4 y + 1 = 0. the value of the eccentricity e. Two different cases arise.
Sol. Let P ( x 1,y1 ) be the point, whose locus is required. Case I (When the focus lies on the directrix)
Its distance from (0, - 1) = 2 ´ its distance from the line In this case Eq. (i) represents the Degenerate conic
3x + 4y + 1 = 0. whose nature is given in the following table :
|3x + 4y1 + 1 |
Þ ( x 1 - 0) 2 + ( y 1 + 1) 2 = 2 ´ 1 Condition Nature of Conic
( 32 + 4 2 ) 2
e >1 ; D = 0, h > ab The lines will be real and
Þ 5 x 12 2
+ ( y 1 + 1) = 2 | 3x 1 + 4y 1 + 1 | distinct intersecting at S.

Squaring and simplifying, we have e = 1 ; D = 0, h 2 = ab The lines will coincident


2
25 ( x 12 + y12 + 2y1 + 1) e < 1 ; D = 0, h < ab The lines will be imaginary.
=4 (9 x 12 + 16y12 + 24 x 1y1 + 6x 1 + 8y1 + 1) Case II (When the focus does not lie on the
or 11x 12 + 39y12 + 96x 1y1 + 24 x 1 - 18y1 - 21 = 0 directrix)
Hence, the locus of ( x 1,y1 ) is In this case Eq. (i) represents the Non-degenerate conic
11x 2 + 39y 2 + 96xy + 24 x - 18y - 21 = 0 whose nature is given in the following table :
Condition Nature of Conic
y Example 2 What conic does the equation 2
e =1 ; D ¹ 0, h = ab a parabola
25 ( x 2 + y 2 - 2x + 1) = (4 x - 3y + 1) 2 represent ? 2
0 < e < 1; D ¹ 0, h < ab an ellipse
Sol. Given equation is 2
e > 1; D ¹ 0, h > ab a hyperbola
25 ( x 2 + y 2 - 2x + 1) = ( 4 x - 3y + 1)2 …(i)
2
e > 1; D ¹ 0, h > ab; a + b = 0 rectangular hyperbola
Write the right hand side of this equation, so that it appears
in perpendicular distance form, then
2
Remark
æ 4 x - 3y + 1 ö 1. If conic represents an empty set, then D ¹ 0, h2 < ab.
( 4 x - 3y + 1)2 = 25 ç ÷
2. If conic represents a single point, the D = 0, h2 < ab.
ç ( 4 2 + 32 ) ÷
è ø
then, Eq. (i) can be re-written as y Example 3 What conic does
é 4 x - 3y + 1 ù
2
13x 2 - 18 xy + 37 y 2 + 2x + 14 y - 2 = 0 represent?
25 [( x - 1)2 + (y - 0)2 ] = 25 ê ú
Sol. Compare the given equation with
êë ( 4 2 + 32 ) úû
ax 2 + 2hxy + by 2 + 2gx + 2 fy + c = 0
| 4 x - 3y + 1 |
or ( x - 1) 2 + ( y - 0) 2 = \ a = 13, h = - 9, b = 37, g = 1, f = 7, c = - 2 ,
( 4 2 + 32 ) then, D = abc + 2 fgh - af 2 - bg 2 - ch 2
Here, e = 1 = (13) (37) ( -2) + 2 (7) (1) ( -9)
Thus, the given equation represents a parabola. It may - 13 (7 )2 - 37(1)2 + 2 ( -9 )2
noted that (10, ) is the focus and 4 x - 3y + 1 = 0 is the
= - 962 - 126 - 637 - 37 + 162 = - 1600 ¹ 0
directrix of the parabola.
and also h 2 = ( -9 )2 = 81 and ab = 13 ´ 37 = 481
Here, h 2 < ab
Recognisation of Conics So, we have h 2 < ab and D ¹ 0.
The equation of conics represented by the general Hence, the given equation represents an ellipse.
equation of second degree
y Example 4 What conic is represented by the
ax 2 + 2hxy + by 2 + 2 gx + 2 fy + c = 0 ...(i) equation ax + by = 1 ?
can be recognised easily by the condition given in the Sol. Given conic is ax + by = 1
tabular form. For this, first we have to find discriminant of
the equation. We know that the discriminant of above On squaring both sides, we get
equation is represented by D, where ax + by + 2 abxy = 1
D = abc + 2 fgh - af 2 - bg 2 -ch 2 Þ ax + by - 1 = - 2 abxy
Chap 05 Parabola 367

Again, on squaring both sides, then 1 3 5


then, a = 2, h = , b = 3 , g = - , f = , c = l.
(ax + by - 1)2 = 4abxy 2 2 2
1
Þ a 2 x 2 + b 2y 2 + 1 + 2abxy - 2by - 2ax = 4abxy Q 2
h = , ab = 6
4
Þ a 2 x 2 + b 2y 2 - 2abxy - 2ax - 2by + 1 = 0 2
\ h < ab
Þ a 2 x 2 - 2abxy + b 2y 2 - 2ax - 2by + 1 = 0 …(i) and D = abc + 2 fgh - af 2 - bg 2 - ch 2
Comparing the Eq. (i) with the equation 5 3 1
= ( 2) ( 3) ( l ) + 2 ´
´- ´
Ax 2 + 2Hxy + By 2 + 2Gx + 2Fy + C = 0 2 2 2
25 9 1
\ A = a 2 , H = - ab, B = b 2 , G = - a , F = -b, C = 1 -2´ -3´ - l ´
4 4 4
then, D = ABC + 2FGH - AF 2 - BG 2 - CH 2 15 25 27 l
= 6l - - - -
= a 2b 2 - 2a 2b 2 - a 2b 2 - a 2b 2 - a 2b 2 4 2 4 4
= -4a 2b 2 ¹ 0 and H 2 = a 2b 2 = AB 23l
= - 23 = 0
4
So, we have D ¹ 0 and H 2 = AB.
\ l=4
Hence, the given equation represents a parabola.
y Example 8 For what value of l the equation of conic
y Example 5 If the equation x 2 - y 2 - 2x + 2y + l = 0 2xy + 4 x - 6 y + l = 0 represents two intersecting
represents a degenerate conic, find the value of l. straight lines, if l = 17, then this equation represents?
Sol. For degenerate conic D = 0 Sol. Comparing the given equation of conic with
Comparing the given equation of conic with ax 2 + 2hxy + by 2 + 2gx + 2 fy + c = 0
ax 2 + 2hxy + by 2 + 2gx + 2 f y + c = 0 \ a = 0 , b = 0, h = 1 , g = 2 , f = -3, c = l
\ a = 1, b = - 1, h = 0, g = -1, f = 1, c = l For two intersecting lines,
\ D = abc + 2 fgh - af 2 - bg 2 - ch 2 = 0 h 2 > ab, D = 0
Q ab = 0, h = 1
Þ ( 1) ( - 1) ( l ) + 0 - 1 ´ ( 1) 2 + 1 ´ ( - 1) 2 - l ( 0) 2 = 0
\ h 2 > ab
Þ - l -1+1=0 Þ l =0 2
and D = abc + 2 fgh - af - bg 2 - ch 2
y Example 6 If the equation x 2 + y 2 - 2x - 2y + c = 0 = 0 + 2 ´ - 3 ´ 2 ´ 1 - 0 - 0 - l ( 1) 2
represents an empty set, then find the value of c. = -12 - l = 0
Sol. For empty set D ¹ 0 and h 2 < ab. \ l = - 12
Now, comparing the given equation of conic with For l = 17, then the given equation of conic
2xy + 4 x - 6y + 17 = 0 according to the first system but
ax 2 + 2hxy + by 2 + 2gx + 2 f y + c ¢ = 0 here c = 17.
then a = 1, h = 0, b = 1, g = -1, f = -1, c ¢ = c \ a = 0, b = 0, h = 1, g = 2 , f = -3, c = 17,
Q h 2 < ab \ D = abc + 2 f gh - af 2 - bg 2 - ch 2
\ 0 < 1 which is true = 0 + 2 ´ - 3 ´ 2 ´ 1 - 0 - 0 - 17 ´ (1)2
and D = abc ¢ + 2 fgh - af 2 - bg 2 - c ¢ h 2 ¹ 0 = - 12 - 17 = -29 ¹ 0
Þ (1)(1)(c ) + 0 - 1 ´ ( -1)2 - 1 ´ ( -1)2 - 0 ¹ 0 \ D ¹ 0 and h 2 > ab
Þ c -2 ¹ 0 So, we have D ¹ 0 and h 2 > ab.
\ c ¹2 Hence, the given equation represents a hyperbola.
Hence, c Î R~ (2)

y Example 7 If the equation of conic How to Find the Centre of Conics


2x 2 + xy + 3y 2 - 3x + 5y + l = 0
If S º ax 2 + 2hxy + by 2 + 2 gx + 2 fy + c = 0.
represent a single point, then find the value of l.
Sol. For single point, Partially differentiating w.r.t.x and y, we get
h 2 < ab and D = 0 ¶S ¶S
= 2ax + 2hy + 2 g ; = 2hx + 2by + 2 f
Comparing the given equation with ¶x ¶y
ax 2 + 2hxy + by 2 + 2gx + 2 fy + c = 0 (Treating y as constant) (Treating x as constant)
368 Textbook of Coordinate Geometry

¶S ¶S æ hf - bg gh - af ö
For centre, = 0 and =0 \ Centre ç , ÷
¶x ¶y è ab - h 2 ab - h 2 ø
\ 2ax + 2hy + 2 g = 0 and 2hx + 2by + 2 f = 0 æ ( -2)( -29 ) - (11)( -22) ( -22) ( -2) - (14 ) ( -29 ) ö
or =ç , ÷
Þ ax + hy + g = 0 and hx + by + f = 0 è (14 )(11) - ( -2)2 (14 ) (11) - ( -2)2 ø
Solving these equations we get the centre or = (2, 3)
æ hf - bg gh - af ö
( x ,y ) = ç , ÷.
è ab - h 2 ab - h 2 ø
Parabola : Definition
Remembering Method
A parabola is the locus of a point which moves in a plane
½a h g½ such that its distance from a fixed point (i.e. focus) is
Since, D =½
½h b f½
½ always equal to its distance from a fixed straight line
½g f c½ (i.e., directrix).
Write first two rows,
i.e.
a h g a
(Repeat Ist member) Standard Equation of Parabola
h b f h
Let S be the focus and ZM be the directrix of the parabola.
\ ab - h 2 , hf - bg, gh - af Draw SZ perpendicular to ZM, let A be the mid point of
SZ, then as AS = AZ
æ hf - bg gh - af ö æ C 13 C 23 ö
or points ç , ÷ or ç , ÷. Y
è ab - h 2 ab - h 2 ø è C 33 C 33 ø M
P (x, y)

OR x+a=0

According to first two rows,


X
ax + hy + g = 0 and hx + by + f = 0. Z A S N
(a, 0)
After solving we get find the centre of conic.

y Example 9 Find the centre of the conic


14 x 2 - 4 xy + 11y 2 - 44 x - 58 y + 71 = 0 So, A lies on the parabola. Take A as the origin and a line
2 2 AY through A perpendicular to AX as Y-axis.
Sol. Let f ( x , y ) º 14 x - 4 xy + 11y - 44 x - 58y + 71 = 0
Let AS = AZ = a > 0
Differentiating partially w.r.t. x and y, then
¶f ¶f
then, coordinate of S is (a,0 ) and the equation of ZM is
= 28x - 4y - 44 and = - 4 x + 22y - 58 x = - a or x + a = 0
¶x ¶y
¶f ¶f
Now, take P ( x , y ) be any point on the parabola. Join SP
For centre, = 0 and = 0, and from P draw PM perpendicular to the directrix ZM.
¶x ¶y
Then, SP = ( x - a ) 2 + (y - 0 ) 2 = ( x - a ) 2 + y 2
\ 28x - 4y - 44 = 0
or 7 x - y - 11 = 0 ¼(i) and PM = ZN = AZ + AN = a + x
and -4 x + 22y - 58 = 0 Now, for the parabola SP = PM
or -2x + 11y = 29 ¼ (ii) Þ (SP ) 2 = ( PM ) 2 Þ ( x - a ) 2 + y 2 = (a + x ) 2
On solving Eqs. (i) and (ii) we get, Þ y 2 = (a + x ) 2 - ( x - a ) 2 = 4ax
x = 2 and y = 3 \ y 2 = 4ax ,
\ Centre is (2, 3). which is required equation of the parabola.
Aliter : Comparing the given conic with
ax 2 + 2hxy + by 2 + 2gx + 2 fy + c = 0
Remark
A parabola has two real foci situated on its axis one of which
\ a = 14, h = - 2, b = 11, g = - 22, f = - 29, c = 71
is the focus S and the other lies at infinity. the corresponding
directrix is also at infinity.
Chap 05 Parabola 369

Some Terms Related to Parabola 5. Focal chord A chord of a parabola which is passing
through the focus is called a focal chord of the
1. Axis The axis of the parabola is the straight line parabola. In the given figure, PP ¢ and LL¢ are the focal
which is passes through focus and perpendicular to chords.
the directrix of the parabola.
Y Remarks
M L P Q (h, 2Öah) 1. In objective questions use LL¢ as focal chord and in
Focal subjective questions use PP¢ as focal chord.
x+a=0

Latus chord
rectum 2. Length of smallest focal chord of the parabola 4a. Hence, the
Double
(0, 0) ordinate latusrectum of a parabola is the smallest focal chord.
S (a, 0) N X
Z A 6. Focal distance The focal distance of any point P on
Focus Axis
the parabola is its distance from the focus S i.e. SP
P¢ Also, SP = PM = Distance of P from the directrix.
L¢ Q¢ (h, – 2Öah)
Directrix If P º (x , y )
For the parabola y 2 = 4ax , X-axis is the axis. then, SP = PM = x + a
Here, all powers of y are even in y 2 = 4ax then, 7. Parametric equations From the equation of the
y 2x
parabola y 2 = 4ax is symmetrical about its axis (i.e. parabola y 2 = 4ax , we can write = =t
2a y
X-axis).
or where ‘t ’ is a parameter.
If the point ( x , y ) lie on the parabola y = 4ax , then 2 Then, y = 2at and x = at 2
the point ( x , - y ) also lies on it. Hence, the parabola is The equations x = at 2 and y = 2at are called
symmetrical about X-axis (i.e. axis of parabola).
parametric equations. The point (at 2 , 2at ) is also
2. Vertex The point of intersection of the parabola and referred to as the point ‘t ’.
its axis is called the vertex of the parabola. For the
parabola y 2 = 4ax . Remarks
1. Coordinates of any point on the parabola y 2 = 4 ax, may be
A (0, 0 ) i.e. the origin is the vertex.
taken as ( at 2, 2at ).
3. Double ordinate If Q be the point on the parabola,
2. Equation of chord joining t1 and t2 is 2x - ( t1 + t2 ) y + 2at1t2 = 0.
draw QN perpendicular to the axis of parabola and
3. If the chord joining t1, t2 and t3, t4 pass through a point ( c, 0 )
produced to meet the curve again at Q ¢, then QQ ¢ is c
on the axis, then t1t2 = t3t4 = - .
called a double ordinate. a
If abscissa of Q is h, then ordinate of Q,
y 2 = 4ah or y = 2 ah (for first quadrant)
Other forms of Parabola with
and ordinate of Q ¢ is y = -2 ah (for fourth quadrant)
Latusrectum 4a
(1) Parabola opening to left (i.e. y 2 = - 4ax ) : (a > 0 )
Hence, coordinates of Q and Q ¢ are (h, 2 ah ) and
(i) Vertex is A (0, 0 ).
(h, -2 ah ), respectively. (ii) Focus is S ( -a, 0 ).
4. Latusrectum The double ordinate LL¢ passes through (iii) Equation of the directrix MZ is x - a = 0.
the focus is called the latusrectum of the parabola. (iv) Equation of the axis is y = 0 i.e. X-axis.
(v) Equation of the tangent at the vertex is x = 0 i.e. Y-axis.
Since focus S (a, 0 ) the equation of the latusrectum of Y
the parabola is x = a, then solving
x– a=0

L
P M
x =a and y 2 = 4ax
then, we get y = ± 2a X′ X
S (–a,0) A Z
Hence, the coordinates of the extremities of the
latusrectum are L (a, 2a ) and L ¢ (a, - 2a ), respectively. L′
Since, LS = L ¢ S = 2a Y′
\ Length of latusrectum LL ¢ = 2 ( LS ) = 2 ( L ¢ S ) = 4a. (vi) Length of latusrectum = LL ¢ = 4a.
370 Textbook of Coordinate Geometry

(vii) Ends of latusrectum are L ( -a , 2a ) and L ¢ ( -a , - 2a ). (ix) Parametric coordinates is (2at , at 2 ).


(viii) Equation of latusrectum is x = - a i.e. x + a = 0.
(3) Parabola opening downwards (i.e.
(ix) Parametric coordinates is ( -at 2 , 2at ).
x 2 = - 4ay ) : (a > 0 )
(2) Parabola opening upwards (i.e. x 2 = 4ay ) : (a > 0 ) (i) Vertex is A (0, 0 ). (ii) Focus is S (0, - a ).
(i) Vertex is A (0, 0 ). (iii) Equation of the directrix MZ is y - a = 0.
(ii) Focus is S (0, a ). (iv) Equation of the axis is x = 0 i.e.Y-axis.
(iii) Equation of the directrix MZ is y + a = 0. (v) Equation of the tangent at the vertex is y = 0 i.e.
Y
X-axis.
Y
S (0, a)
L′ L Z My–a=0

P
X′ X A
A X′ X
P

y+a=0 Z M L′ L
S(0,–a)
Y′
(iv) Equations of the axis is x = 0 i.e.Y-axis. Y′
(v) Equation of the tangent at the vertex is y = 0 i.e. X-axis. (vi) Length of latusrectum = LL ¢ = 4a
(vi) Ends of latusrectum are L (2a, a ) and L ¢ ( -2a, a ). (vii) Ends of latusrectum are L (2a, -a ) and L ¢ ( -2a, - a )
(vii) Length of latusrectum = LL ¢ = 4a. (viii) Equation of latusrectum is y = - a i.e. y + a = 0.
(viii) Equation of latusrectum is y = a i.e. y - a = 0. (ix) Parametric coordinates are (2at , - at 2 ).

Smart Table : The Study of Standard Parabolas


Equation and Graph of y 2 = 4ax, a > 0 y 2 = - 4ax, a > 0 x 2 = 4ay , a > 0 x 2 = - 4ay , a > 0
the parabola
Y Y
Y Z M
Y
P L S
M L P M L′ L

P A
X′ X
X′ X X′ X
Z A S S A Z X′ X P
A

L′ L′ L′ L
S
Y′ Z M
Y′
Y′ Y′
Vertex ( 0, 0 ) ( 0, 0 ) ( 0, 0 ) ( 0, 0 )
Focus (a, 0 ) ( - a, 0 ) ( 0, a ) ( 0, - a )
Equation of the axis y=0 y =0 x =0 x =0
Equation of tangent at x =0 x =0 y =0 y =0
vertex
Equation of directrix x +a =0 x -a = 0 y +a =0 y -a = 0
Length of latusrectum 4a 4a 4a 4a
Ends points of (a, ± 2a ) ( - a, ± 2a ) ( ± 2a, a ) ( ± 2a, - a )
latusrectum
Equation of x -a = 0 x +a =0 y -a = 0 y +a =0
latusrectum
Focal distance of a x +a a -x y +a a -y
point P (x , y )
Parametric coordinates (at 2, 2at ) ( - at 2, 2at ) (2at, at 2 ) (2at, - at 2 )
Eccentricity (e) 1 1 1 1
Chap 05 Parabola 371

General Equation of a Parabola Þ 5 ( x 2 + y 2 + 2x + 4y + 5)

Let S (a,b ) be the focus, and lx + my + n = 0 is the equation = ( x 2 + 4y 2 - 4 xy + 6x - 12y + 9 )


of the directrix. Let P ( x ,y ) be any point on the parabola. \ 4 x 2 + y 2 + 4 xy + 4 x + 32y + 16 = 0
Then by definition SP = PM
Y y Example 11 Find the equation of the parabola whose
focus is (4, - 3) and vertex is (4, - 1) .
Sol. Let A ( 4, - 1) be the vertex and S ( 4, - 3) be the focus.
M
y)
-3 + 1
(x,
=0

L \ Slope of AS = =¥
P

A
+n

Z 4-4
my

S (a, b) which is parallel to Y-axis.


lx +

L′ \ Directrix parallel to X-axis.


X
O Y
M Z y –1 = 0

O
|lx +my + n | X′ X
Þ ( x - a ) 2 + (y - b ) 2 = A (4, –1)
(l 2 +m 2 )
(lx +my + n ) 2
Þ ( x - a ) 2 + (y - b ) 2 = P (x, y)
(l 2 +m 2 )
Þ m 2 x 2 + l 2 y 2 - 2lmxy + x term+ y term + constant = 0 S (4, –3)
2
This is of the form (mx - ly ) + 2 gx + 2 fy + c = 0.
Y′
This equation is the general equation of parabola. Let Z ( x 1,y1 ) be any point on the directrix, then A is the
mid-point of SZ .
Remark x +4
\ 4= 1 Þ x1 = 4
Second degree terms in the general equation of a parabola 2
forms a perfect square.
y -3
and -1 = 1 Þ y1 = 1
y Example 10 Find the equation of the parabola whose 2
focus is at ( -1,-2) and the directrix is the straight line \ Z = ( 4, 1)
x - 2y + 3 = 0. Also, directrix is parallel to X-axis and passes through
Z ( 4,1) , so equation of directrix is
Sol. Let P ( x ,y ) be any point on the parabola whose focus is
y = 1 or y - 1 = 0
S( -1,-2) and the directrix x - 2y + 3 = 0 . Draw PM
perpendicular from P ( x ,y ) on the directrix x - 2y + 3 = 0 . Now, let P ( x ,y ) be any point on the parabola. Join SP and
draw PM perpendicular to the directrix. Then, by definition
0
Y 3= SP = PM
y+
x–
2 Þ (SP )2 = ( PM )2
2
2 æ| y - 1 |ö
2
Þ ( x - 4 ) + ( y + 3) = ç ÷
è 12 ø
X′ X
M O
Þ ( x - 4 ) 2 + ( y + 3) 2 = ( y - 1) 2

P
\ x 2 - 8x + 8y + 24 = 0
(x, y)
S Aliter :
(–1,–2)
Here a = AS = 2
Y′ \ Length of latusrectum = 4a = 8
Then, by definition Equation of parabola with vertex (0, 0) and open downward
SP = PM is x 2 = - 8y .
Þ (SP ) 2 = ( PM )2 Shifting ( 4, - 1) on (0, 0), we get required parabola
2
æ | x - 2y + 3| ö ( x - 4 )2 = - 8(y + 1)
Þ ( x + 1) 2 + ( y + 2) 2 = ç ÷
ç ( 1) 2 + ( - 2) 2 ÷ \ x 2 - 8x + 8y + 24 = 0
è ø
372 Textbook of Coordinate Geometry

y Example 12 The focal distance of a point on a Since, R divide QQ ¢ in 1 : 2 (internally)


parabola y 2 = 8 x is 8. Find it. and T divide QQ ¢ in 2 : 1 (internally).
For locus, let R (h ,k ), then
Sol. Comparing y 2 = 8x with y 2 = 4ax
y1
Y x1 = h and = k or y1 = 3k
M P (x1, y1) 3
x + 2 = 0 Directrix

On substituting the values of x 1 and y1in Eq. (ii), then


Focal
distance = 8 (3k )2 = 4a (h ) or 9k 2 = 4ah
X
Z A S \ The required locus is 9y 2 = 4ax similarly, let T (h ',k ' )
y1
then, x 1 = h ' and - = k'
3
or y1 = - 3k '
\ 4a = 8 Þ a = 2
On substituting the values of x 1 and y1 in Eq. (ii), then
\ Equation of directrix is x + 2 = 0.
( - 3k ' )2 = 4a (h ' )
Let P ( x 1, y1 ) on the parabola
y 2 = 8x or 9k ¢ 2 = 4ah '
\ y12 = 8x 1 …(i) \ The required locus is 9y 2 = 4ax .
Q SP = 8 Hence, the locus of point of trisection is
Þ PM = 8 [QSP = PM ] 9y 2 = 4ax .
Þ x1 + 2 = 8 Aliter : Let R and T be the points of trisection of double
or x1 = 6 ordinates QQ'. Let (h ,k ) be the coordinates of R,
From Eq. (i), y12 = 8 ´ 6 then, AL = h and RL = k
\ y1 = ± 4 3 RT = RL + LT = k + k = 2k .
Since, RQ = TR = Q ¢ T = 2k
\ The required points are (6, 4 3 ) and (6, - 4 3 ).
\ LQ = LR + RQ = k + 2k = 3k
y Example 13 QQ ¢ is a double ordinate of a parabola Thus, the coordinates of Q are (h , 3k ).
y 2 = 4ax . Find the locus of its point of trisection . Since, (h , 3k ) lies on y 2 = 4ax
Þ 9k 2 = 4ah
Sol. Let the double ordinate QQ' meet the axis of the parabola
Hence, the locus of (h , k ) is 9y 2 = 4ax .
y 2 = 4ax …(i)
Let coordinates of Q be ( x 1, y1 ), then coordinates of Q ¢ be y Example 14 Prove that the area of the triangle
( x 1 - y1 ) since, Q and Q ¢ lies on Eq. (i), then
inscribed in the parabola y 2 = 4ax is
y12 = 4ax 1 …(ii)
1
Let R and T be the points of trisection of QQ'. Then, the ( y 1 ~ y 2 ) ( y 2 ~ y 3 ) ( y 3 ~ y 1 ), where y 1 , y 2 , y 3 are
8a
coordinates of R and T are
the ordinates of the vertices.
æ 1 × x 1 + 2 × x 1 1 × ( -y 1 ) + 2 × y 1 ö æ y1 ö
ç , ÷ or ç x 1, ÷ Sol. Let the vertices of the triangle be ( x 1, y1 ) , ( x 2 , y 2 ) and
è 1+2 1+2 ø è 3ø
( x 3 , y 3 ).
æ 2 × x 1 + 1 × x 1 2 × ( -y 1 ) + 1 × y 1 ö æ y ö Q ( x 1, y1 ) is a point on the parabola y 2 = 4ax .
and ç , ÷ or ç x 1,- 1 ÷
è 2+1 2+1 ø è 3ø
\ y12 = 4ax 1
respectively.
Y y12
\ x1 =
Q 4a
y2
Located Similarly, x2 = 2
R parabola 4a
X y2
A L and x3 = 3
4a
T Now , vertices of triangle are
Q′ æ y12 ö æy 2 ö æy 2 ö
ç ,y1 ÷ , ç 2 ,y 2 ÷ and ç 3 ,y 3 ÷.
è 4a ø è 4a ø è 4a ø
Chap 05 Parabola 373

y12 \ Equation of directrix is x - y + l = 0


y1 1 where, l is constant.
4a
1 y 2 Q A is the mid-point of SZ .
\ Required area of the triangle = | 2 y 2 1 | \ SZ = 2SA
2 4a
y 32 |0 - 0 + l | | 0 - 0 + 1|
y3 1 Þ =2´
2 2
4a ( 1 + ( - 1) ) ( 12 + ( - 1) 2 )

y12 y1 1 |l | 2
Þ =
1 2 1 2 2
= | y2 y2 1 | = (y 1 ~ y 2 ) (y 2 ~ y 3 ) (y 3 ~ y 1 )
8a 2 8a \ l = ±2
y3 y3 1
Þ l =2
[Q l is positive since directrix in this
y Example 15 Find the length of the side of an case always lies in II quadrant]
equilateral triangle inscribed in the parabola y 2 = 4ax , \ Equation of directrix is x - y + 2 = 0.
so that one angular point is at the vertex. Now, take P ( x , y ) be any point on the parabola, draw
Sol. Let ABC be the inscribed equilateral triangle, with one PM ^ ZM , then from definition,
angular point at the vertex A of the parabola SP = PM
y 2 = 4ax …(i) Þ (SP )2 = ( PM )2
2
Let the length of the side of equilateral triangle = l æ | x - y + 2| ö
Þ ( x - 0) 2 + ( y - 0) 2 = ç ÷
\ AB = BC = CA = l è 2 ø
Y
B
Þ 2 ( x 2 + y 2 ) = ( x - y + 2) 2
Þ 2x 2 + 2y 2 = x 2 + y 2 - 2xy + 4 x - 4y + 4
l
\ x 2 + y 2 + 2xy - 4 x + 4y - 4 = 0
X′ 30° X
A 30°
l

C
Equation of Parabola if
Y′
\ The coordinates of B is (l cos 30° , l sin 30° )
Equation of axis, Tangent at
æl 3 l ö
Vertex and Latusrectum are
i.e., ç , ÷.
è 2 2ø 2
æl 3 ö
given
æl ö
Since, B lies on Eq. (i), then ç ÷ = 4a ç ÷ or l = 8a 3 Let equation of axis is ax + by + c = 0 and equation of
è2ø è 2 ø tangent at vertex is bx - ay + d = 0.
y Example 16 Prove that the equation of the parabola
Equation of parabola is
whose focus is (0,0) and tangent at the vertex is
t

x - y + 1 = 0 is x 2 + y 2 + 2xy - 4 x + 4 y - 4 = 0.
en
rte ng
ve ta
x

Sol. Let focus is S (0, 0) and A is the vertex of the parabola take
at n of
tio

any point Z such that AS = AZ given tangent at vertex is


ua

N
x - y + 1 = 0, since directrix is parallel to the tangent at
Eq

the vertex.
A P(x,y)
Y 0
2=
+ 0
–y M
x 1=
Z
M y+ axis
x–
A
P (x, y)
X′ X
S ( PM ) 2 = (Latusrectum) (PN )
2
æ ax + by + c ö æ ö
Þ ç ÷ =(Latusrectum) ç bx - ay + d ÷
Y′ ç (a 2 + b 2 ) ÷ ç (b 2 + a 2 ) ÷
è ø è ø
374 Textbook of Coordinate Geometry

y Example 17 Find the equation of the parabola whose This is called generalised form of the parabola Eq. (i)
latusrectum is 4 units, axis is the line 3x + 4 y - 4 = 0 and and axis A ¢ X ¢¢|| AX with its vertex at A ¢ (h, k ). Its focus
the tangent at the vertex is the line 4 x - 3y + 7 = 0 . is at (a + h, k ) and length of latusrectum = 4a, the
Sol. Let P ( x , y ) be any point on the parabola and let PM and PN equation of the directrix is
are perpendiculars from P on the axis and tangent at the x =h -a Þ x +a -h =0
vertex respectively, then
Another form is ( x - h ) 2 = 4a (y - k ) axis parallel to
Y 0
7=

Y Y′′
y+
–3
4x

N
S
P (x, y)
A a

M A′ (h, k)
X′ X y=k–a Directrix
O 3x Z′
+
4y X′ X
–4 A
=
0
Y′
Y′
Y-axis with its vertex (h,k ) its focus is at (h,a + k ) and
( PM )2 = (latusrectum) ( PN )
2
length of latusrectum = 4a, the equation of the directrix
æ 3x + 4y - 4 ö æ ö is
Þ ç ÷ = 4 ç 4 x - 3y + 7 ÷
ç 32 + 4 2 ÷ ç 4 2 + ( - 3) 2 ÷ y = k - a Þ y + a - k = 0.
è ø è ø
\ (3x + 4y - 4 )2 = 20 ( 4 x - 3y + 7 ) Remark
which is required parabola. The parametric equation of ( y - k )2 = 4a(x - h) are x = h + at 2
and y = k + 2at.

The Generalised form


(y - k)2 = 4 a(x - h) Parabolic Curve
The parabola The equations y = Ax 2 + Bx + C and x = Ay 2 + By + C
are always represents parabolas generally called
y 2 = 4ax …(i)
parabolic curve.
2
can be written as (y - 0 ) = 4a ( x - 0 ). Now, y = Ax 2 + Bx + C
The vertex of this parabola is A(0,0 ) ì B Cü
= Aí x 2 + x + ý
Y î A Aþ
x=h–a ìï æ 2
B ö B 2 C üï
= Aí ç x + ÷ - + ý
ïî è 2A ø 4 A 2 A ïþ
a
X′′ 2
Z′ A′ (h, k) S ìï æ B ö ( B 2 - 4 AC ) üï
= Aí ç x + ÷ - ý
ïî è 2A ø 4A2 ïþ
Directrix
2
X′ X æ B ö 1æ B 2 - 4 AC ö
A or ç x + ÷ = ç y + ÷
è 2A ø Aè 4A ø
Y′
Comparing it with ( x - h ) 2 = 4a (y - k ) it represent a
Now, when origin is shifted at A' (h, k ) without changing the æ B B 2 - 4 AC ö
direction of axes, its equation becomes parabola with vertex at (h,k ) = ç - ,- ÷
è 2A 4A ø
(y - k ) 2 = 4a ( x - h ) …(ii)
Chap 05 Parabola 375

1 The optimum distance of its vertex V from OY is


and axis parallel to Y-axis and latusrectum =
| A| B 2 - 4 AC
- .
and the curve opening upwards and downwards depending 4A
upon the sign of A and B.
Y Remarks
1. The optimum distance of vertex from OX or OY can be easily
V (h, k)
obtained using calculus Method.
2. Equation of the parabola with axis parallel to the X-axis is of
the form x = Ay 2 + By + C.
k 3. Equation of the parabola with axis parallel to the Y-axis is of
the form y = Ax 2 + Bx + C.
h
O M
X Method to Make Perfect Square
If x = ay 2 ± by + g
The optimum distance of its vertex V from OX is
B 2 - 4 AC first make the coefficient of y 2 is unity
-
4A ì b gü
i.e., x = a íy 2 ± y + ý
and x = Ay 2 + By + C î a aþ
ì B Cü Now, in braces write y and put the sign after y which
= A íy 2 + y + ý between y 2 and y i.e. ± and after this sign write the half
î A Aþ
b
ìïæ 2 the coefficient of y i.e. .
B ö B2 C üï 2a
= A íç y + ÷ - + ý
ïîè 2A ø 4 A 2 A ïþ æ b ö
2
Now, write in braces ç y ± ÷
ìïæ B ö
2
B 2 - 4 AC üï è 2a ø
= A íç y + ÷ - ý 2
ïîè 2A ø 4 A 2 ïþ æ bö b2
and always subtract ç ÷ = 2
2 è 2a ø 4a
æ B ö 1 æ B 2 - 4 AC ö
Þ çy + ÷ = çx + ÷ ìï æ 2
è 2A ø Aè 4A ø bö b2 g üï
\ x = aí ç y ± ÷ - + ý
îï
è 2a ø 4a 2 a ïþ
Comparing it with (y - k ) 2 = 4a( x - h ), it represent a
ìï æ 2
parabola with vertex at b ö (b2 - 4 ga ) üï
= aí ç y ± ÷ - ý
æ B 2 - 4 AC B ö è 2a ø 4a 2 þï
(h, k ) = ç - ,- ÷ îï
è 4A 2A ø
y Example 18 Find the vertex, focus, latusrectum, axis
1
and axis parallel to X-axis and latusrectum = and the directrix of the parabola x 2 + 8 x + 12y + 4 = 0.
| A|
Sol. The equation of parabola is
and the curve opening left and right depending upon the x 2 + 8x + 12y + 4 = 0 …(i)
sign of A and B. 2
Þ ( x + 4 ) - 16 + 12y + 4 = 0
Y
Þ ( x + 4 )2 - 12 + 12y = 0
Þ ( x + 4 )2 = - 12y + 12
h Þ ( x + 4 )2 = - 12 (y - 1)
N V (h , k )
Let x + 4 = X,y -1 =Y …(ii)
k \ X 2 = -12Y …(iii)
Comparing it with X 2 = -4aY
O X \ a=3
\ Vertex of Eq. (iii) is (0, 0)
i.e. X = 0 ,Y = 0
376 Textbook of Coordinate Geometry

From Eq. (ii), Hence, axis of parabola parallel to X-axis.


x + 4 = 0,y -1 = 0 Y
\ x = - 4, y = 1
\ Vertex of Eq. (i) is ( -4, 1).
Foucs of Eq. (iii) is (0, –3)
(3, 2)
i.e. X = 0, Y = - 3 A S (5, 2)
From Eq. (ii),
x + 4 = 0, y - 1 = - 3
\ x = -4 , y = -2 O
X
\ Focus of Eq. (i) is (–4, –2).
and latusrectum = 4a = 12. The equation is of the form
Equation of axis of Eq. (iii) is X = 0 ( y - k ) 2 = 4a ( x - h )
\ Equation of axis of Eq. (i) is x + 4 = 0 or ( y - 2) 2 = 4a ( x - 3)
Equation of directrix of Eq. (iii) is as (h,k) is the vertex (3,2)
Y = 3 or y - 1 = 3 a = distance between the focus and the vertex
\ y-4 =0
= ( 5 - 3) 2 + ( 2 - 2) 2 = 2
\ Equation of directrix of Eq. (i) is
y - 4 = 0. Hence, the required equation is
( y - 2) 2 = 8( x - 3)
y Example 19 Prove that the equation or y 2 - 8x - 4y - 28 = 0.
y 2 + 2ax + 2by + c = 0 represents a parabola whose axis
is parallel to the axis of x. Find its vertex. y Example 21 Find the equation of the parabola with
Sol. The equation of parabola is latusrectum joining the points (3, 6) and (3, –2).
y 2 + 2ax + 2by + c = 0 -2-6
Sol. Slope of (3,6) and (3, –2) is = ¥, since latusrectum
2 2 3-3
(y + b ) - b + 2ax + c = 0
is perpendicular to axis. Hence, axis parallel to X-axis. The
Þ (y + b )2 = - 2ax + b 2 - c equation of the two possible parabolas will be of the form
æ b2 - c ö ( y - k ) 2 = ± 4a ( x - h ) …(i)
Þ (y + b )2 = - 2a ç x - ÷ …(i)
è 2a ø Since, latusrectum = (3 - 3)2 + (6 + 2)2 = 8
2
b -c \ 4a = 8
Let y +b =Y, x - =X
2a Þ a=2
From Eq. (i) , \ From Eq. (i),
Y 2 = -2aX …(ii) (y - k )2 = ± 8 ( x - h )
axis of its parabola is Y = 0 Since, (3,6) and (3, –2) lie on the parabola, then
or y + b = 0, (6 - k )2 = ± 8 (3 - h ) …(ii)
which is parallel to X-axis 2
and ( -2 - k ) = ± 8 ( 3 - h ) …(iii)
and vertex of Eq. (ii) is X = 0, Y = 0
b2 - c Y
L (3, 6)
Þ x- = 0, y + b = 0
2a
b2 - c
Þ x= , y = -b
2a
æb2 - c ö A
\ Vertex of given parabola is ç , - b ÷. (1, 2) S (3, 2) A′ (5,2)
è 2a ø
X
y Example 20 Find the equation of the parabola with O
its vertex at (3, 2) and its focus at (5, 2). L′ (3, –2)
Sol. Let Vertex A (32
, ) and focus is S(52
, )
2-2 On solving Eqs. (ii) and (iii), we get
Slope of AS = = 0, which is parallel to X-axis.
5-3 k =2
Chap 05 Parabola 377

From Eq. (ii), Þ 9 = A+B+4 [QC = 4 ]


16 = ± 8 (3 - h ), \ A+B=5 …(iii)
\ h = 3 ±2 and 5 = 16A + 4 B + C [QC = 4 ]
\ h = 5,1 Þ 5 = 16A + 4 B + 4
Hence, values of (h, k) are (5,2) and (1, 2). \ 16A + 4 B = 1
The required parabolas are 1
Þ 4A + B = …(iv)
( y - 2) 2 = 8( x - 5) 4
On solving Eqs. (iii) and (iv), we get
and (y - 2)2 = -8( x - 1).
19 79
A = - ,B = …(v)
y Example 22 Find the equation to the parabola whose 12 12
axis parallel to the Y-axis and which passes through On substituting the values of A , B and C from Eqs. (ii) and
Eq. (v) in Eq. (i), then equation of parabola is
the points (0, 4) (1, 9) and (4, 5) and determine its
19 79
latusrectum . y = - x2 + x + 4
12 12
Sol. The equation of parabola parallel to Y-axis is
1 12
y = Ax 2 + Bx + C …(i) Hence, length of latusrectum = = .
19 19
The points (0,4) , (1,9) and (4,5) lie on Eq. (i), then -
12
4 = 0+0+C Þ C = 4 …(ii)
Þ 9 = A + B +C

Exercise for Session 1


1. The vertex of the parabola y 2 + 6x - 2y + 13 = 0 is
(a) (-2, 1) (b) (2, - 1)
(c) (1, 1) (d) (1 , –1)

2. If the parabola y 2 = 4 ax passes through (3, 2), then the length of latusrectum is
1 2
(a) (b)
3 3
4
(c) 1 (d)
3

3. The value of p such that the vertex of y = x 2 + 2px + 13 is 4 units above the X-axis is
(a) ± 2 (b) 4
(c) ± 3 (d) 5

4. The length of the latusrectum of the parabola whose focus is (3,3) and directrix is 3 x - 4y - 2 = 0, is
(a) 1 (b) 2
(c) 4 (d) 8

5. If the vertex and focus of a parabola are (3,3) and (–3, 3) respectively , then its equation is
(a) x 2 - 6x + 24y - 63 = 0 (b) x 2 - 6x + 24y + 81 = 0
(c) y 2 - 6y + 24x - 63 = 0 (d) y 2 - 6y - 24x + 81 = 0

6. If the vertex of the parabola y = x 2 - 8x + c lies on X-axis, then the value of c is


(a) 4 (b) – 4
(c) 16 (d) –16

7. The parabola having its focus at (3,2) and directrix along the Y-axis has its vertex at
3 3
(a) æç , 1ö÷ (b) æç , 2ö÷
è2 ø è2 ø
3 1ö 3 1ö
(c) æç , ÷ (d) æç , - ÷
è2 2ø è2 2ø
378 Textbook of Coordinate Geometry

8. The directrix of the parabola x 2 - 4x - 8y + 12 = 0 is


(a) y = 0 (b) x = 1
(c) y = -1 (d) x = -1

9. The equation of the latusrectum of the parabola x 2 + 4x + 2y = 0 is


(a) 3y - 2 = 0 (b) 3y + 2 = 0
(c) 2y - 3 = 0 (d) 2y + 3 = 0

10. The focus of the parabola x 2 - 8x + 2y + 7 = 0 is


1
(a) æç 0, - ö÷ (b) (4,4)
è 2ø
9 9
(c) æç 4, ö÷ (d) æç -4, - ö÷
è 2ø è 2ø

11. The equation of the parabola with the focus (3,0) and directrix x + 3 = 0 is
(a) y 2 = 2x (b) y 2 = 3x
(c) y 2 = 6x (d) y 2 = 12x

12. Equation of the parabola whose axis is parallel to Y-axis and which passes through the points (1,0) , (0,0) and
( -2,4), is
(a) 2x 2 + 2x = 3y (b) 2x 2 - 2x = 3y
(c) 2x 2 + 2x = y (d) 2x 2 - 2x = y

13. Find the equation of the parabola whose focus is (5,3) and directrix is the line 3x - 4y + 1 = 0 .

14. Find the equation of the parabola is focus is at (–6, –6) and vertex is at (–2, 2).

15. Find the vertex, focus, axis, directrix and latusrectum of the parabola 4y 2 + 12x - 20y + 67 = 0.

æxö æyö
16. Find the name of the conic represented by ç ÷ +
èa ø
ç ÷ = 1.
èb ø

17. Determine the name of the curve described parametrically by the equations
x = t 2 + t + 1, y = t 2 - t + 1.

18. Prove that the equation of the parabola whose vertex and focus are on the X-axis at a distance a and a ¢ from
the origin respectively is y 2 = 4 (a ¢ - a ) ( x - a ).

19. Find the equation of the parabola whose axis is parallel to X-axis and which passes through the points (0, 4) ,
(1, 9) and (–2, 6). Also, find its latusrectum.
20. The equation ax 2 + 4xy + y 2 + ax + 3y + 2 = 0 represents a parabola, then find the value of a.
Session 2
Position of a Point (x1, y1) with respect to a Parabola y2 = 4ax,
Parametric Relation between the Coordinates of the Ends of a Focal
Chord of a Parabola, Intersection of a Line and a Parabola, Equation of
Tangent in Different Forms, Point of Intersection of Tangents at any
Two Points on the Parabola, Equation of Normals in Different Forms,
Point of Intersection of Normals at any Two Points on the Parabola,
Circle Through Co-normal Points

Position of a Point (x 1 , y 1 ) with Remarks


1. The point ( x1, y1 ) lies inside, on or outside y 2 = - 4 ax
Respect to a Parabola y 2 = 4 ax according as y12 + 4 ax1 <, =,or > 0

Theorem The point ( x 1 , y 1 ) lies outside, on or inside the 2. The point ( x1, y1 ) lies inside, on or outside x 2 = 4 ay according
as x12 - 4 ay1 <, = ,or > 0
parabola y 2 = 4ax according as
3. The point ( x1, y1 ) lies inside, on or outside x 2 = - 4 ay
y 12 - 4ax 1 >, =, or < 0 . according as x12 + 4 ay1 <, = ,or > 0
Proof Let P ( x 1 , y 1 ) be a point. From P draw PM ^ AX
(on the axis of parabola) meeting the parabola y 2 = 4ax at y Example 23 Show that the point (2, 3) lies outside
the parabola y 2 = 3x .
Q let the coordinate of Q be ( x 1 , y 2 ).
Sol. Let the point (h , k ) = (2, 3)
Y P (x1, y1)
We have, k 2 - 3h = 32 - 32
. = 9 -6= 3 > 0
Q (x1, y2) \ k 2 - 3h > 0
This shows that (2,3) lies outside the parabola y 2 = 3x .
X′ X
A M
y Example 24 Find the position of the point ( -2,2)
y 2 = 4ax with respect to the parabola y 2 - 4 y + 9 x + 13 = 0.
Y′ Sol. Let the point (h ,k ) = ( -2,2)
We have, k 2 - 4k + 9h + 13 = (2)2
Since, Q ( x 1 , y 2 ) lies on the parabola
- 4 (2) + 9 ( -2) + 13 = 4 - 8 - 18 + 13 = -9 < 0
y 12 = 4ax
Hence, k 2 - 4k + 9h + 13 < 0
then, y 22 = 4ax 1 K(i) Therefore, the point ( -2,2) lies inside the parabola
Now, P will be outside, on or inside the parabola y = 4ax 2 y 2 - 4y + 9 x + 13 = 0.
according as
PM >, =, or <QM Parameteric Relation between
Þ 2
( PM ) >, = ,or <(QM ) 2
the Coordinates of the Ends of
Þ y 12 > , = ,or < y 22 a Focal Chord of a Parabola
Þ y 12 >, = , or < 4ax 1 [from Eq. (i)] Let y 2 = 4ax be a parabola, if PQ be a focal chord.
Then, P º (at 12 , 2at 1 ) and Q º (at 22 , 2at 2 )
Hence, y 12 - 4ax 1 > , = , or < 0
Since, PQ passes through the focus S (a ,0 ).
380 Textbook of Coordinate Geometry

\ Q ,S , P are collinear. Remark


Y 1
2 ) Q t+ ³ 2for all t ¹ 0 [Q AM ³ GM]
P (at1 , 2at1 t
2
1
\ a æçt + ö÷ ³ 4a
è tø
X Þ Length of focal chord ³ latusrectum i.e. The length of
A S (a , 0 )
smallest focal chord of the parabola is 4a . Hence, the
Q (a latusrectum of a parabola is the smallest focal chord.
t2 2,
2at
2) y Example 26 Prove that the semi-latusrectum of the
\ Slope of PS = Slope of QS parabola y 2 = 4ax is the harmonic mean between the
2at 1 - 0 0 - 2at 2 2t 2t 2 segments of any focal chord of the parabola.
Þ = Þ 1 = Sol. Let parabola be y 2 = 4ax
at 12 - a a - at 22 t 12 - 1 t 22 - 1
If PQ be the focal chord, if
Þ t 1 (t 22 - 1) = t 2 (t 12 - 1)
æ a -2a ö
P º (at 2 , 2at ), then Q º ç 2 , ÷
Þ t 1 t 2 (t 2 - t 1 ) + (t 2 - t 1 ) = 0 èt t ø
Þ t 2 - t 1 ¹ 0 or t 1 t 2 + 1 = 0 \ Length of latusrectum LL ' = 4a.
1 1
Þ t 1 t 2 = -1 or t 2 = - , …(i) \ Semi-latusrectum = ( 4a ) = 2a.
t1 2
which is required relation. If sections of focal chord are k1 and k 2 ,
then, k1 = SP = PM = a + at 2 = a(1 + t 2 )
Remark a a(1 + t 2 )
If one extremity of a focal chord is (at 12 , 2at1 ) then the other and k 2 = SQ = QN = a + 2
=
t t2
æ a 2a ö
extremity (at22 , 2at2 ) becomes çç 2 ,- ÷÷ by virtue of relation Eq. (i). Y
è t1 t1 ø P (at2, 2at)
M
x+a=0

L K1
y Example 25 If the point (at 2 , 2at ) be the extremity of
(0,0)
a focal chord of parabola y 2 = 4ax then show that the Z A d S (a, 0)
X
2
æ 1ö
length of the focal chord is a çt + ÷ . N
Q L′
è tø a, 2a

Sol. Since, one extremity of focal chord is P (at 2 ,2at ), then the t2 t
æ a 2a ö \ Harmonic mean of k1
other extremity is Q ç 2 ,- ÷ [Replacing t by –1/t]
èt t ø 2k1k 2
Y and k2 =
2,
P (at 2
at) k1 + k 2
M 2 2
= = 2
1 1 t 1
+ +
X k 2 k 1 a (1 + t 2 ) a (1 + t 2 )
A S (a, 0)
2
x+a=0

= = 2a = Semi-latusrectum.
N Q 1
a, 2a a

t2 t

\ Length of focal chord = PQ


Remarks
1. The length of focal chord having parameters t1 and t2 for its
= SP + SQ [QSP = PM and SQ = QN ] end points is a ( t2 - t1 ) 2.
= PM + QN
a 2. If l1 and l2 are the length of segments of a focal chord of a
= at 2 + a + 2 + a 4l l
t parabola, then its latusrectum is 1 2 .
2 l1 + l2
æ 2 1 ö æ 1ö
= a çt + 2 + 2÷ = a çt + ÷
è t ø è tø
Chap 05 Parabola 381

y Example 27. Show that the focal chord of |2at | 2a


= =
parabola y 2 = 4ax makes an angle a with the ( t 2 + 1) t +
1
t
X-axis is of length 4a cosec 2a.
4a 2
Sol. Let P (at 12 , 2at 1 ) and Q (at 22 , 2at 2 ) be the end points of a Þ d2 = 2
…(i)
focal chord PQ which makes an angle a with the axis of æ 1ö
çt + ÷
the parabola. Then, è tø
PQ = a (t 2 - t 1 )2 æa 2a ö
The other end of the focal chord is Q ç 2 , - ÷
2
= a [(t 2 + t 1 ) - 4t 1t 2 ] …(i) èt t ø
2 If length of focal chord = PQ = (l say )
= a[(t 2 + t 1 ) + 4 ] [Q t 1t 2 = - 1]
\ l = PQ = PS + SQ = PM + QN
a
\ l = at 2 + a + 2 + a
\ tana = slope of PQ t
2at 2 - 2at 1 2
= æ 2 1 ö æ 1 ö
at 22 - at 12 Þ l = a çt + 2 + 2÷ Þ l = a çt + ÷
è t ø è t ø
2 2
Þ tana = l æ 1ö
t 2 + t1 Þ = çt + ÷ …(ii)
a è tø
Þ t 2 + t 1 = 2 cota …(ii)
4a 2 4a 3
On substituting the value of t 2 + t 1 from Eq. (ii) in Eq. (i), From Eqs. (i) and (ii), d 2 = =
then (l / a ) l
PQ = a( 4 cot 2 a + 4 ) 4a 3 1
\ l= 2
Þ lµ
d d2
= 4a cosec 2 a.
i.e. the length of the focal chord varies inversely as the
square of its distance from vertex.
y Example 28 Prove that the length of a focal chord
of a parabola varies inversly as the square of its
distance from the vertex.
Sol. Let P (at 2 , 2at ) be one end of a focal chord of the parabola
Intersection of a Line and a
y 2 = 4ax . The focus of its parabola is S (a ,0). Parabola
Y Let the parabola be y 2 = 4ax …(i)
2,
P (at 2at)
M and the given line be y = mx + c …(ii)
x+a=0

On eliminating x from Eqs. (i) and (ii), then


(0,0) æy -c ö
Z A d S (a, 0)
X y 2 = 4a ç ÷
è m ø
N
Q Þ my 2 - 4ay + 4ac = 0 …(iii)
a, 2a

t2 t This equation being quadratic in y, gives two values of y,
\ Equation of focal chord is (i.e. equation of PS) shows that every straight line will cut the parabola in two
2at - 0 points may be real, coincident or imaginary according as
y -0= 2 ( x - a)
at - a discriminant of Eq. ( iii ) > , = < 0
Þ y=
2t
( x - a) i.e. ( -4a ) 2 - 4 × m × 4ac >, =, < 0 or a - mc >, =, < 0
2
( t - 1)
or a >, =, < mc …(iv)
2
Þ (t - 1)y = 2tx - 2at
2tx - (t 2 - 1)y - 2at = 0
Þ
Condition of tangency
If d be the distance of this focal chord from the vertex (00
, )
of the parabola y 2 = 4ax , then
If the line Eq. (ii) touches the parabola Eq. (i), then Eq. (iii)
has equal roots
| 0 - 0 - 2at |
d= \ Discriminant of Eq. (iii) = 0
(2t )2 + (t 2 - 1)2
Þ ( -4a ) 2 - 4m × 4ac = 0
382 Textbook of Coordinate Geometry

a l n
Þ c= ,m ¹ 0 …(v) \ M=- and c = -
m m m
So, the line y = mx + c touches the parabola y 2 = 4ax if The line Eq. (i) will touch the parabola y 2 = 4ax , if
a a
c = (which is condition of tangency). c = Þ cM = a
M
m
æ n öæ l ö
Substituting the value of c from Eq. (v) in Eq. (ii), then Þ ç- ÷ç- ÷ = a
è m øè m ø
a
y = mx + , m ¹ 0 \ ln = am 2
m
Aliter :
a
Hence, the line y = mx + will always be a tangent to the Given line lx + my + n = 0 …(i)
m
and the parabola y 2 = 4ax …(ii)
parabola y 2 = 4ax .
n + my
a Substituting the value of x from Eq. (i) i.e. x = - in
The point of contact Substituting c = in Eq. (iii), then l
m Eq. (ii), then
a
æ ö (we should not substituting the value of y from Eq. (i), in
my 2 - 4ay + 4a ç ÷ = 0 Eq. (ii) since y is quadratic, substituting the value of x since
èm ø x is linear).
Þ m 2 y 2 - 4amy + 4a 2 = 0 æ n + my ö
y 2 = 4a ç - ÷
è l ø
Þ (my - 2a ) 2 = 0
Þ ly 2 + 4amy + 4an = 0 …(iii)
Þ my - 2a = 0
Since, Eq. (i) touches the parabola Eq. (ii), then roots of
2a Eq. (iii) must be coincident and condition for the same is
or y=
m B 2 = 4 AC ,
a i.e., ( 4am )2 = 4 × l 4an
Substituting this value of y in y = mx +
m Þ am 2 = ln
2a a \ ln = am 2
\ = mx +
m m
a a y Example 30 Show that the line x cos a + y sin a = p
Þ mx = or x =
m m2 touches the parabola y 2 = 4ax , if p cos a + a sin 2 a = 0
æ a 2a ö and that the point of contact is (a tan 2 a , - 2a tan a ) .
Hence, the point of contact is ç , ÷ , (m ¹ 0 ) this
èm 2 m ø Sol. The given line is
known as m-point on the parabola. x cos a + y sin a = p
Þ y = - x cota + p coseca
Remark Comparing this line with y = mx + c .
If m = 0, then Eq. (iii) gives \ m = - cota and c = p coseca
0 - 4 ay + 4 ac = 0 since, the given line touches the parabola
Þ y=c a
\ c = or cm = a
which gives only one value of y and so every line parallel to m
X-axis cuts the parabola only in one real point.
Þ ( p coseca )( - cot a ) = a
y Example 29 Prove that the straight line Þ a sin 2 a + p cos a = 0
lx + my + n = 0 touches the parabola y 2 = 4ax , if æ a 2a ö
and point of contact is ç 2 , ÷
ln = am 2 . èm m ø

Sol. The given line is lx + my + n = 0 æ a 2a ö


i.e. ç 2
,- ÷
l n è cot a cot a ø
or y =- x- …(i)
m m or (a tan 2 a , - 2a tan a )
Comparing this line with y = Mx + c
Chap 05 Parabola 383

x y y Example 32 Find the equations of the straight lines


y Example 31 Prove that the line + = 1 touches the
l m touching both x 2 + y 2 = 2a 2 and y 2 = 8ax .
parabola y 2 = 4a( x + b ), if m 2 (l + b ) + al 2 = 0. Sol. The given curves are
Sol. The given parabola is Y
y 2 = 4a( x + b ) …(i) ent
o n tang
Comm
Vertex of this parabola is ( -b, 0).
Now, shifting (0, 0) at ( -b, 0),
then, x = X + ( -b ) and y = Y + 0 X
O
or x + b = X and y = Y …(ii)
From Eq. (i),Y 2 = 4aX …(iii)
x y
and the line + =1
l m
X -b Y x 2 + y 2 = 2a 2 …(i)
reduces to + =1 2
l m and y = 8ax …(ii)
æ X - bö The parabola Eq. (ii) is y = 8ax 2
Y = m ç1 - ÷
è l ø
or y 2 = 4(2a )x
æ mö æ bö
Þ Y = ç - ÷ X + m ç1 + ÷ …(iv) \ Equation of tangent of Eq. (ii) is
è l ø è lø
2a
The line Eq. (iv) will touch the parabola Eq. (iii), if y = mx +
m
æ bö a
m ç1 + ÷ = or m 2 x - my + 2a = 0 …(iii)
è l ø æ mö
ç- ÷ It is also tangent of Eq. (i), then the length of perpendicular
è l ø
from centre of Eq. (i) i.e. (0, 0) to Eq. (iii) must be equal to
m2 æ bö the radius of Eq. (i) i.e. a 2.
Þ ç1 + ÷ = -a
l è lø | 0 - 0 + 2a | 4a 2
\ =a 2 Þ = 2a 2
\ m 2 (l + b ) + al 2 = 0 2 2
( m ) + ( -m ) 2 m4 + m2
Aliter :
Þ m4 + m2 - 2 = 0
The given line and parabola are
x y Þ ( m 2 + 2) ( m 2 - 1) = 0
+ =1 …(i)
l m Q m2 + 2 ¹ 0 [gives the imaginary values]
and y 2 = 4a( x + b ) …(ii) \ 2
m -1=0
respectively substituting the value of x from Eq. (i) Þ m = ±1
æ yö Hence, from Eq. (iii) the required tangents are
x = l ç1 - ÷
è mø x ± y + 2a = 0.
ì æ yö ü
in Eq. (ii), then y 2 = 4aíl ç1 - ÷ + bý

2
î
4al
è m ø þ
Equation of Tangent in
or y +
m
y - 4a(l + b ) = 0 …(iii)
Different Forms
Since, the line Eq. (i) touches the parabola Eq. (ii), then the
roots of Eq. (iii) are equal.
æ 4al ö
2 1. Point Form :
\ ç ÷ - 4 × 1 { -4a(l + b )} = 0
èm ø To find the equation of the tangent to the parabola
al 2
y 2 = 4ax at the point ( x 1 , y 1 ).
Þ + (l + b ) = 0
m2 (First Principal Method) Equation of parabola is
Þ al 2 + m 2 (l + b ) = 0 y 2 = 4ax …(i)
2 2
Þ m (l + b ) + al = 0 Let P º ( x 1 , y 1 ) and Q º ( x 2 , y 2 ) be any two points on
parabola (i), then
384 Textbook of Coordinate Geometry

)
, y1
P (x 1 2. Parametric Form :
T To find the equation of tangent to the parabola y 2 = 4ax
at the point (at 2 , 2at ) or ‘t’.
Since, the equation of tangent of the parabola y 2 = 4ax at
Q (x
2 , y2 ) ( x 1 , y 1 ) is yy 1 = 2a( x + x 1 ) …(i)
2
replacing x 1 by at and y 1 by 2at, then Eq. (i) becomes
y 12 = 4ax 1 …(ii)
y (2at ) = 2a( x + at 2 ) Þ ty = x + at 2
and y 22 = 4ax 2 …(iii)
Subtracting Eq. (ii) from Eq. (iii), then Remark
The equations of tangent of all standard parabolas at ‘t’.
y 22 - y 12 = 4a( x 2 - x 1 )
Equations of Parametric Tangent
y2 - y1 4a Parabolas coordinates ‘t’ at ‘t’
Þ = …(iv)
x2 - x1 y2 + y1 y 2 = 4ax (at 2, 2at ) ty = x + at 2
Equation of PQ is y 2 = - 4ax ( -at 2, 2at ) ty = - x + at 2
y - y1
y - y1 = 2 (x - x 1 ) …(v) x 2 = 4ay (2at, at 2 ) tx = y + at 2
x2 - x1
x 2 = - 4ay (2at, - at 2 ) tx = - y + at 2
From Eqs. (iv) and (v), then
4a
y - y1 = (x - x 1 ) …(vi)
y2 + y1 3. Slope Form :
Now, for tangent at P , Q ® P , i.e. x 2 ® x 1 and y 2 ® y 1 , To find the equation of tangent and point of contact in
then Eq. (vi) becomes terms of m(slope) to the parabola y 2 = 4ax .
4a
y - y1 = ( x - x 1 ) Þ yy 1 - y 12 = 2ax - 2ax 1 The equation of tangent to the parabola y 2 = 4ax at
2y 1
( x 1 , y 1 ) is yy 1 = 2a( x + x 1 ). …(i)
Þ yy 1 = 2ax + y 12 - 2ax 1 Since, m is the slope of the tangent, then
Þ yy 1 = 2ax + 4ax 1 - 2ax 1 [from Eq. (ii)] 2a 2a
m= Þ y1 =
Þ yy 1 = 2ax + 2ax 1 y1 m
\ yy 1 = 2a( x + x 1 ), Since, ( x 1 , y 1 ) lies on y 2 = 4ax , therefore
which is the required equation of tangent at ( x 1 , y 1 ). 4a 2
y 12 = 4ax 1 Þ = 4ax 1
Remarks m2
a
1. The equation of tangent at ( x1, y1 ) can also be obtained by \ x1 =
replacing x 2 by xx1, y 2 by yy1, x by
x + x1
, y by
y + y1
and xy by m2
2 2
xy1 + x1 y Substituting the values of x 1 and y 1 in Eq. (i), we get
and without changing the constant (if any) in the a
2 y = mx + …(ii)
equation of curve. This method is apply only when the m
equations of parabola is polynomial of second degree in x a
and y. Thus, y = mx + is a tangent to the parabola y 2 = 4ax ,
2. Equation of tangents of all standard parabolas at ( x1, y1 ).
m
where, m is the slope of the tangent.
Equations of Parabolas Tangent at( x1, y1 ) æ a 2a ö
The coordinates of the point of contact are ç , ÷ .
2
y = 4 ax yy1 = 2 a( x + x1 ) èm 2 m ø
Comparing Eq. (ii) with y = mx + c ,
2
y = - 4 ax yy1 = - 2 a( x + x1 ) a
c=
x 2 = 4 ay xx1 = 2 a( y + y1 )
m
2
which is condition of tangency.
x = - 4 ay xx1 = - 2 a( y + y1 )
when, y = mx + c is the tangent of y 2 = 4ax .
Chap 05 Parabola 385

Remark
The equation of tangent, condition of tangency and point of contact in terms of slope ( m) of all standard parabolas.
Equation of parabolas Point of contact in terms of Equation of tangent in terms of Condition of tangency
slope (m) slope (m)
y 2 = 4ax æ a 2a ö a a
ç 2, ÷ y = mx + c=
èm m ø m m

y 2 = - 4ax æ a 2a ö y = mx -
a
c =-
a
ç- 2 , ÷
è m mø m m
x 2 = 4ay (2am, am 2 ) y = mx - am 2 c = - am 2
x 2 = - 4ay (2am, - am 2 ) y = mx + am 2 c = am 2
(y - k ) 2 = 4a( x - h ) æ a 2a ö
y = mx - mh + k +
a
c + mh = k +
a
çh + 2 , k + ÷
è m mø m m
(y - k ) 2 = - 4a( x - h ) æ a 2a ö
y = mx - mh + k -
a
c + mh = k -
a
çh - 2 , k + ÷
è m mø m m
( x - h ) 2 = 4a(y - k ) (h + 2am, k + am 2 ) y = mx - mh + k - am 2 c + mh = k - am 2
( x - h ) 2 = - 4a(y - k ) (h + 2am, k - am 2 ) y = mx - mh + k + am 2 c + mh = k + am 2

Point of Intersection of Remarks

Tangents at any two Points on 1. The geometric mean of the x-coordinates of P and Q
(i.e. at12 ´ at22 = at1t2 ) is the x-coordinate of the point of

the Parabola intersection of tangents at P and Q on the parabola. If P and


Q are the ends points of focal chord, then x-coordinate of
Let the parabola be y 2 = 4ax point of intersection of tangents at P and Q is (– at1 t2 ).
2. The arithmetic mean of the y-coordinates of P and Q
let two points on the parabola are æi.e. 2at1 + 2at2 = a ( t + t ) ö is the y-coordinate of the point of
ç 2 ÷
P º (at 12 , 2at 1 ) and Q º (at 22 , 2at 2 ).
1
è 2 ø
intersection of tangents at P and Q on the parabola.
Equation of tangents at P (at 12 , 2at 1 )
and Q (at 22 , 2at 2 ) Remembering Method :
G O A [GOA rule]
are t 1 y = x + at 12 …(i)
and t 2 y = x + at 22 …(ii)
Y t 1)
2, 2a GM of at 12 and at 22 , AM of 2at 1 and 2at 2
at 1
P( 2at 1 + 2at 2
i.e. at 1 t 2 i.e. = a(t 1 + t 2 )
2
X′
O X
R
y Example 33 Find the equation of the common
(at1t2, a(t1 + t2)) tangents to the parabola y 2 = 4ax and x 2 = 4by .
Q (a
t2 2,
2at
Sol. The equation of any tangent in terms of slope (m) to the
2) parabola y 2 = 4ax is
Y′
a
On solving these equations, we get x = at 1 t 2 , y = a(t 1 + t 2 ) y = mx + …(i)
m
Thus, the coordinates of the point of intersection of If this line is also tangent to the parabola x 2 = 4ay , then Eq. (i)
tangents at
meets x 2 = 4by in two coincident points.
(at 12 , 2at 1 ) and (at 22 , 2at 2 ) are (at 1 t 2 , a(t 1 + t 2 )).
386 Textbook of Coordinate Geometry

Substituting the value of y from Eq. (i) in x 2 = 4by , we get Sol. Let the points P (at 12 , 2at 1 ) and Q (at 22 , 2at 2 ) on the parabola
æ aö y 2 = 4ax tangents at P and Q are
x 2 = 4b çmx + ÷
è mø t 1y = x + at 12 …(i)
4ab
Þ x 2 - 4bmx - =0 and t 2y = x + at 22 …(ii)
m
The roots of this quadratic are equal provided
Y
B 2 = 4 AC

Directrix
P
2 æ -4ab ö
i.e., ( -4bm ) = 4 × 1 ç ÷
è m ø
Þ 16b 2m 3 + 16ab = 0, m ¹ 0 90°
X
R
3
or m = -a /b
\ m = - a1/ 3 / b1/ 3 Q
Substituting the value of m in Eq. (i) the required equation is
a1/ 3 ab1/ 3
y = - 1/ 3 x - 1/ 3 Q Point of intersection of these tangents is (at 1t 2 , a(t 1 + t 2 ))
b a
1/ 3 Let this point is (h , k ),
a
Þ y = - 1/ 3 x - a 2 / 3b1/ 3 then, h = at 1t 2 …(iii)
b
and k = a (t 1 + t 2 ) …(iv)
\ a1/ 3 x + b1/ 3y + a 2 / 3b 2 / 3 = 0 1 1
Slope of tangents Eqs. (i) and (ii) are and , respectively.
t1 t2
y Example 34 The tangents to the parabola y 2 = 4ax
Since, tangents are perpendicular, then
make angle q 1 and q 2 with X-axis. Find the locus of
1 1
their point of intersection, if cotq 1 + cotq 2 = c . ´ = -1
t1 t 2
Sol. Let the equation of any tangent to the parabola y 2 = 4ax
is or t 1t 2 = - 1 …(v)
y = mx + (a / m ) …(i) From Eqs. (iii) and (v), we get
Let ( x 1, y1 ) be the point of intersection of the tangents to h = - a or h + a = 0
y 2 = 4ax , then Eq. (i) passes through ( x 1, y1 ). \ Locus of the point of intersection of tangents is
\ y1 = mx 1 + (a / m ) x +a=0
or m 2 x 1 - my1 + a = 0 which is directrix of y 2 = 4ax .

Let m1 and m 2 be the roots of this quadratic equation, then Aliter :


m1 + m 2 = y1 / x 1 and m1m 2 = a / x 1 Let the equation of any tangent to the parabola y 2 = 4ax is
or tan q 1 + tan q 2 = y1 / x 1 y = mx + a / m …(i)
and …(ii) 2
tan q 1 tan q 2 = a / x 1 Let the point of intersection of the tangents to y = 4ax
Now, cot q 1 + cot q 2 = c (given) then, Eq (i) passes through ( x 1, y1 ) .
1 1 \ y1 = mx 1 + a / m
Þ + =c
tan q 1 tan q 2 or m 2 x 1 - my1 + a = 0
tan q 1 + tan q 2 Let m1, m 2 be the roots of this quadratic equation then
Þ =c [from Eq. (ii)]
tan q 1 tan q 2 m1m 2 = a / x 1 = - 1
y1 / x 1 [since, tangents are perpendiculars]
Þ =c
a / x1 Þ a + x1 = 0
Þ y1 = ac \ Locus of the point of intersection of tangents is
The required locus is y = ac , x + a = 0 which is directrix of y 2 = 4ax .
which is a line parallel to X-axis.
Remark
y Example 35. Show that the locus of the points of Locus of the point of intersection of the perpendicular tangents
intersection of the mutually perpendicular tangents to to the parabola y 2 = 4ax is called the director circle. Its equation
a parabola is the directrix of the parabola. is x + a = 0, which is parabola’s own directrix.
Chap 05 Parabola 387

y Example 36 The tangents to the parabola y 2 = 4ax


, y1)
P (x 1
at P (at 12 , 2at 1 ) and Q (at 22 , 2at 2 ) intersect at R. Prove
ent
1 Tang
that the area of the DPQR is a 2 |(t 1 - t 2 )| 3 . T al
2 Norm
Sol. Equations of tangents at P (at 12 , 2at 1 ) and Q (at 22 , 2at 2 ) are
t 1y = x + at 12 …(i)
and t 2y = x + at 22 …(ii) The slope of the tangent at ( x 1 , y 1 ) = 2a / y 1
Y t 1)
2 , 2a Since, the normal at ( x 1 , y 1 ) is perpendicular to the
(at 1
P tangent at ( x 1 , y 1 ).
\ Slope of normal at ( x 1 , y 1 ) = - y 1 / 2a
X′ X Hence, the equation of normal at ( x 1 , y 1 ) is
A
y1
R y - y1 = - ( x - x 1 ).
(at1t2, a(t1+ t2)
2a

Q (a
t2 2, Remarks
Y′ 2at
2) 1. The equation of normal at ( x1, y1 ) can also be obtained by
this method
Since, point of intersect of Eqs. (i) and (ii) is
x - x1 y - y1
R(at 1t 2 , a(t 1 + t 2 )). = …(i)
a¢ x1 + hy1 + g hx1 + by1 + f
at 12 2at 1 1
1 a¢, b, g, f , h are obtained by comparing the given parabola with
\ Area of DPQR = at 22 2at 2 1
2 at t a(t + t ) 1 a¢ x 2 + 2hxy + by 2 + 2gx + 2fy + c = 0 …(ii)
1 2 1 2
and denominators of Eq. (i) can easily remembered by the
Applying R 2 ® R 2 - R1 and R 3 ® R 3 - R1 first two rows of this determinant
at 12 ½a¢ h g½
2at 1 1 i.e.
1 ½h b f ½
½ ½
= a(t 22 - t 12 ) 2a(t 2 - t 1 ) 0 ½g f c ½
2 at (t - t ) a(t - t ) 0
1 2 1 2 1
Since, first row a¢( x1 ) + h( y1 ) + g( 1)
Expanding with respect to first row and second row, h( x1 ) + b( y1 ) + f ( 1)
1 a(t 22 - t 12 ) 2a(t 2 - t 1 ) Here, parabola y 2 = 4 ax
=
2 at 1(t 2 - t 1 ) a(t 2 - t 1 ) or y 2 - 4 ax = 0 …(iii)
1 2 t + t1 2 Comparing Eqs. (ii) and (iii), then we get
= a (t 2 - t 1 )2 2
2 t1 1 a¢ = 0, b = 1, g = - 2 a, h = 0, f = 0
1 From Eq. (i), equation of normal of Eq. (iii) is
= a 2 (t 1 - t 2 )2 |(t 2 - t 1 )| x - x1 y - y1
2 =
1 0 + 0 - 2a 0 + y1 + 0
= a 2 (t 1 - t 2 )2 |(t 1 - t 2 )|
2 y1
or y - y1 = -
( x - x1 )
1 2a
= a 2 |(t 1 - t 2 )3 |.
2 2. Equations of normals of all standard parabolas at ( x1, y1 ).
Equations of Parabola Normal at ( x1, y1 )

Equations of Normals in
2 y1
y = 4 ax y - y1 = - ( x - x1 )
2a
Different Forms y 2 = - 4 ax y - y1 =
y1
( x - x1 )
2a
1. Point Form: To find the equation of the normal to
x 2 = 4 ay 2a
the parabola y 2 = 4ax at the point ( x 1 , y 1 ). y - y1 = - ( x - x1 )
x1
Since, the equation of the tangent to the parabola y 2 = 4ax x 2 = - 4 ay 2a
y - y1 = ( x - x1 )
at ( x 1 , y 1 ) is x1
yy 1 = 2a( x + x 1 )
388 Textbook of Coordinate Geometry

2. Parametric form : The equation of normal to the parabola y 2 = 4ax at


( x 1 , y 1 ) is
To find the equation of normal to the parabola y 2 = 4ax
y
at the point (at 2 , 2at ) or ‘t’. y - y 1 = - 1 (x - x 1 ) …(i)
2a
Since, the equation of normal of the parabola y 2 = 4ax at Since, m is the slope of the normal,
( x 1 , y 1 ) is y
y then, m = - 1 Þ y 1 = - 2am
y - y 1 = - 1 (x - x 1 ) …(i) 2a
2a
Replacing x 1 by at 2 and y 1 by 2at, then Eq. (i) becomes Since, ( x 1 , y 1 ) lies on y 2 = 4ax , therefore

y - 2at = - t ( x - at 2 ) y 12 = 4ax 1
or y + tx = 2at + at 3 Þ 4a 2m 2 = 4ax 1
\ x 1 = am 2
Remark
The equations of normals of all standard parabolas at ‘t’ On substituting the values of x 1 and y 1 in Eq. (i) we get
Equations of Parametric Normals y + 2am = m( x - am 2 )
Parabolas coordinates ‘t’ at ‘t’
\ y = mx - 2am - am 3 …(ii)
y 2 = 4 ax ( at 2, 2 at ) y + tx = 2 at + at 3
3
Thus, y = mx - 2am - am is a normal to the parabola
y 2 = - 4 ax ( -at 2, 2 at ) y - tx = 2 at + at 3
y 2 = 4ax , where m is the slope of the normal. The
x 2 = 4 ay (2 at, at 2 ) x + ty = 2 at + at 3
coordinates of the point of contact are (am 2 , - 2am )
2
x = - 4 ay (2 at, - at 2 ) x - ty = 2 at + at 3

On comparing Eq. (ii) with


y = mx + c
3. Slope form : \ c = - 2am - am 3
To find the Equation of normal, condition for
which is condition of normality when y = mx + c is the
normality and point of contact in terms of m (slope) to
normal of y 2 = 4ax .
the parabola y 2 = 4ax

Remark
The equations of normals, point of contact and condition of normality in terms of slope ( m) of all standard parabolas.

Point of contact in terms of Equation of normals in Condition of


Equation of parabolas
slope (m) terms of slope (m) normality
y 2 = 4 ax ( am2, - 2 am) y = mx - 2 am - am3 c = - 2 am - am3
y 2 = - 4 ax ( -am2, 2 am) y = mx + 2 am + am3 c = 2 am + am3
x 2 = 4 ay æ 2a a ö a
y = mx + 2 a + 2 c = 2a + 2
a
ç- , 2÷
è m m ø m m
x 2 = - 4 ay æ 2a a ö
y = mx - 2 a -
a
c = - 2a -
a
ç ,- 2÷
èm m ø m2 m2
( y - k )2 = 4 a( x - h) ( h + am2, k - 2 am) y - k = m( x - h) c = k - mh - 2 am - am3
- 2 am - am3
( y - k )2 = - 4 a( x - h) ( h - am2, k + 2 am) y - k = m( x - h) c = k - mh + 2 am + am3
+ 2 am + am3
( x - h)2 = 4 a( y - k ) æ 2a a ö y - k = m( x - h) c = k - mh + 2 a +
a
çh - , k + 2÷
è m m ø a m2
+ 2a + 2
m
Chap 05 Parabola 389

Y
t 1)
Point of Intersection of P (a
2 2a
t1 ,

Normals at any Two Points on


the Parabola X′
A
X

Let the parabola be y 2 = 4ax .


Q (a
t2 2,
Let the points on the parabola are Y′
2at
2)

P º (at 12 , 2at 1 ) and Q = (at 22 , 2at 2 ).


Since, it meet the parabola again at Q (at 22 , 2at 2 ), then
t 1)
2 , 2a
Y
(at 1 Eq. (i) passes through Q (at 22 , 2at 2 ).
P
\ 2at 2 = - at 1 t 22 + 2at 1 + at 13
R Þ 2a(t 2 - t 1 ) + at 1 (t 22 - t 12 ) = 0
A
X′ X
Þ a(t 2 - t 1 )[2 + t 1 (t 2 + t 1 )] = 0
Q a(t 2 - t 1 ) ¹ 0
Q

[Qt 1 and t 2 are different]


(a
t2
2
,2

\ 2 + t 1 (t 2 + t 1 ) = 0
at 2

Y′
)

Equations of normals at P (at 12 , 2at 1 ) and Q (at 22 , 2at 2 ) are 2


\ t 2 = -t1 -
t1
y = -t1x + 2at 1 + at 13 …(i)
and y = -t2x + 2at 2 + at 23 …(ii) Remarks
1. If normals at ‘t1’ and ‘t2’ meets the parabola y 2 = 4 ax at
On solving Eqs. (i) and (ii), we get
same point, then t1t2 = 2.
x = 2a + a(t 12 + t 22 + t 1 t 2 ) and y = - at 1 t 2 (t 1 + t 2 ) Proof Suppose normals meet at ‘ T ’, then
If R is the point of intersection, then 2 2
T = - t1 - = - t2 -
t1 t2
R º [2a + a (t 12 + t 22 + t 1 t 2 ), - at 1 t 2 (t 1 + t 2 )]
æ 1 1ö
(Remember) Þ ( t1 - t2 ) = 2 ç - ÷
è t2 t1 ø
Point of intersection of normals at t 1 and t 2 or t1t2 = 2 [Qt1 ¹ t2 ]
Parabola Equation of normal Point of intersection of 2. If the normals to the parabola y 2 = 4 ax at the points t1 and t2
at any point ‘t’ normals at t 1 and t 2 intersect again on the parabola at the point t3, then
y2 = 4 ax y + tx = 2at + at 3 ( 2a + a ( t12 + t1 t 2 + t 22 ), t3 = - ( t1 + t2 ) and the line joining t1 and t2 passes through a
- at1t2 (t1 + t2 )) fixed point ( - 2a, 0 ).
2 3
y = - 4 ax y - tx = 2at + at ( -2a - a ( t12 + t1 t 2 + t 22 ), y Example 37 Show that normal to the parabola
at1t2 (t1 + t2 )) y 2 = 8 x at the point (2, 4) meets it again at (18, –12).
x2 = 4 ay x + ty = 2at + at 3 ( - at1 t 2 ( t1 + t 2 ), 2a Find also the length of the normal chord.
+ a(t12 + t1t2 + t22 ))
Sol. Comparing the given parabola (i.e. y 2 = 8x ) with
2 3 ( at1 t 2 ( t1 + t 2 ), - 2a
x = - 4 ay x - ty = 2at + at y 2 = 4ax .
- a(t12 + t1t2 + t22 ))
\ 4a = 8 Þ a = 2
Since, normal at ( x 1, y1 ) to the parabola y 2 = 4ax is
Relation between ‘t 1 ’ and ‘t 2 ’ if Normal y1
y - y1 = - ( x - x1 )
at ‘t 1 ’ meets the Parabola Again at ‘t 2 ’ 2a
Here, x 1 = 2 and y1 = 4.
Let the parabola be y 2 = 4ax , equation of normal at
\ Equation of normal is
P (at 12 , 2at 1 ) is 4
y - 4 = - ( x - 2)
y = - t 1 x + 2at 1 + at 13 …(i) 4
390 Textbook of Coordinate Geometry

Y it meet the curve again Q say (at 22 , 2at 2 ).


)
P( 2, 4 2
\ t 2 = - t1 - …(ii)
t1
Y
X′ X
A )
2, 2at 1
(a t 1
P

Q (1
8, –
1 2)
Y′ X′ X
A
Þ y - 4 = -x +2
Þ x +y -6=0 …(i) θ
On solving Eq. (i) and y 2 = 8x , Q (at 2, 2at )
2 2
2
then, y = 8(6 - y ) Y′
2
Þ y + 8y - 48 = 0
Now, angle between the normal and parabola
Þ (y + 12)(y - 4 ) = 0 = Angle between the normal and tangent at Q
\ y = - 12 and y = 4 (i.e. t 2y = x + at 22 )
then, x = 18 and x = 2. If q be the angle, then
Hence, the point of intersection of normal and parabola are 1
(18, - 12) and (2, 4 ), therefore normal meets the parabola at -t 1 -
m1 - m 2 t2 tt +1
(18, - 12) and length of normal chord is distance between tanq = = =- 12
their points 1 + m1m 2 æ1ö t 2 - t1
1 + ( -t 1 ) ç ÷
èt 2 ø
= PQ = (18 - 2)2 + ( -12 - 4 )2 = 16 2
æ 2ö
t 1 ç -t 1 - ÷ + 1
y Example 38 Prove that the chord =-
è t1 ø
[from Eq. (ii)]
2
y - x 2 + 4a 2 = 0 is a normal chord of the -t 1 - - t 1
t1
parabola y 2 = 4ax . Also, find the point on the parabola
-t 12 - 1 t
when the given chord is normal to the parabola. =- =- 1
æ 1 + t 12 ö 2
Sol. We have, y - x 2 + 4a 2 = 0 - 2ç ÷
è t1 ø
i.e., y = x 2 - 4a 2 …(i)
tan f
Comparing the Eq. (i) with the equation y = mx + c , then = [from Eq. (i)]
2
m = 2, c = - 4a 2 æ1 ö
\ q = tan -1 ç tan f ÷
è2 ø
Since, -2am - am 3 = - 2a 2 - a( 2 )3
y Example 40 Prove that the normal chord to a
= - 2a 2 - 2a 2 = - 4a 2 = c
parabola y 2 = 4ax at the point whose ordinate is equal
Hence, the given chord is normal to the parabola y 2 = 4ax .
to abscissa subtends a right angle at the focus.
The coordinates of the points are (am 2 , - 2am ) i.e. Sol. Let the normal at P (at 12 , 2at 1 ) meet the curve at Q (at 22 , 2at 2 ).
(2a, - 2 2 a ) .
\ PQ is a normal chord
2
y Example 39 If the normal to a parabola y 2 = 4ax , and t 2 = - t1 - . …(i)
t1
makes an angle f with the axis. Show that it will cut
By given condition, 2at 1 = at 12
æ1 ö
the curve again at an angle tan -1 ç tan f ÷ . \ t 1 = 2 from Eq. (i), t 2 = - 3
è2 ø
then, P ( 4a, 4a ) and Q (9a, - 6a )
Sol. Let the normal at P (at 12 , 2at 1 ) be
but focus S(a, 0).
y = - t 1x + 2at 1 + at 13 . 4a - 0 4a 4
\ Slope of SP = = =
\ tan f = - t 1 = slope of the normal, …(i) 4a - a 3a 3
Chap 05 Parabola 391

t 1) This is a cubic equation in m, so it has three roots, say


Y 2
t1 , 2a
P (a m 1 , m 2 and m 3 .
\ m 1 + m 2 + m 3 = 0,
(2a - h )
X′
A S
X m 1m 2 + m 2m 3 + m 3m 1 = ,
a
k
m 1m 2m 3 = - …(ii)
Q (at 2, 2at ) a
2 2
Y′ Hence, for any given point P (h, k ), Eq. (i) has three real or
-6a - 0 6a 3 imaginary roots. Corresponding to each of these three
and slope of SQ = =- =-
9a - a 8a 4 roots, we have one normal passing through P (h, k ).
4 3 Hence, in total, we have three normals PA, PB and PC
Q Slope of SP ´ Slope of SQ = ´ - = -1
3 4 drawn through P to the parabola.
\ ÐPSQ = p / 2 Points A, B, C in which the three normals from P (h, k )
i.e. PQ subtends a right angle at the focus S. meet the parabola are called co-normal points.
y Example 41 If the normal to the parabola y 2 = 4ax Corollary 1 The algebraic sum of the slopes of three
concurrent normals is zero. This follows from Eq. (ii).
at point t 1 cuts the parabola again at point t 2 , prove
Corollary 2 The algebraic sum of ordinates of the feets of
that t 22 = 8.
three normals drawn to a parabola from a given point is
Sol. A normal at point t 1 cuts the parabola again at t 2 . Then, zero.
2 Let the ordinates of A, B, C be y 1 , y 2 , y 3 respectively, then
t 2 = - t1 - Þ t 12 + t 1t 2 + 2 = 0
t1
y 1 = - 2am 1 , y 2 = - 2am 2 and y 3 = - 2am 3
Since, t 1 is real , so (t 2 )2 - 4 × 1 × 2 ³ 0
\ Algebraic sum of these ordinates is
Þ t 22 ³ 8 y 1 + y 2 + y 3 = - 2am 1 - 2am 2 - 2am 3
= - 2a (m 1 + m 2 + m 3 )

Co-normal Points = - 2a ´ 0
=0
[from Eq. (ii)]

In general three normals can be drawn from a point to a Corollary 3 If three normals drawn to any parabola
parabola and their feet, points where they meet the y 2 = 4ax from a given point (h, k ) be real then h > 2a.
parabola are called conormal points.
When normals are real, then all the three roots of Eq. (i)
Let P (h, k ) be any given point and y 2 = 4ax be a parabola. are real and in that case
The equation of any normal to y 2 = 4ax is m 12 + m 22 + m 23 > 0 (for any values of m 1 , m 2 , m 3 )
y = mx - 2am - am 3
Þ (m 1 + m 2 + m 3 ) 2 - 2 (m 1m 2 + m 2m 3 + m 3m 1 ) > 0
Y
2 (2a - h )
Þ (0 ) 2 - > 0 Þ h - 2a > 0
A a
B \ h > 2a
X′ X Remark
O P (h, k)
For a = 1normals drawn to the parabola y 2 = 4 x from any point
C ( h, k ) are real, if h > 2.

Corollary 4 If three normals drawn to any parabola


Y′
y 2 = 4ax from a given point (h, k ) be real and distinct, then
If it passes through (h, k ), then 27ak 2 < 4 (h - 2a ) 3
3
k = mh - 2am - am
Let f (m ) = am 3 + m (2a - h ) + k
3
Þ am + m (2a - h ) + k = 0 …(i)
392 Textbook of Coordinate Geometry

\ f ¢ (m ) = 3am 2 + (2a - h ) y = mx - 2am - am 3

Two distinct roots of f ¢ (m ) = 0 are If it passes through (h , k ), then


k = mh - 2am - am 3
æ h - 2a ö æ h - 2a ö
a= ç ÷ and b = - ç ÷, Þ am 3 + m (2a - h ) + k = 0 …(i)
è 3a ø è 3a ø
Let the roots of Eq. (i) be m1, m 2 and m 3 .
Now, f (a ) f (b) < 0 Þ f (a ) f ( -a ) < 0 Then, from Eq. (i), m1 + m 2 + m 3 = 0 …(ii)
3 3 (2a - h )
Þ (aa + a (2a - h ) + k ) ( -aa - a (2a - h ) + k ) < 0 m1m 2 + m 2m 3 + m 3m1 = …(iii)
a
Þ k 2 - (aa 2 + (2a - h )) 2 a 2 < 0
k
2 and m1m 2m 3 = - …(iv)
2æ h - 2a ö (h - 2a ) a
Þ k -ç + (2a - h ) ÷ <0 But here, two of the three normals are given to be
è 3 ø 3a
coincident i.e. m1 = m 2 .
2
æ 4a - 2h ö æ h - 2a ö Putting m1 = m 2 in Eqs. (ii) and (iv), we get
Þ k2 - ç ÷ ç ÷ <0
è 3 ø è 3a ø 2m1 + m 3 = 0 …(v)
k
4 (h - 2a ) 3 and m12m 3 = - …(vi)
Þ k2 - < 0 Þ 27ak 2 - 4 (h - 2a ) 3 < 0 a
27a Putting m 3 = - 2m1 from Eq. (v) in Eq. (vi), we get
\ 27ak 2 < 4 (h - 2a ) 3 - 2m13 = -
k
a
Corollary 5 The centroid of the triangle formed by the feet
3 k
of the three normals lies on the axis of the parabola. Þ m1 =
2a
If A ( x 1 , y 1 ), B ( x 2 , y 2 ) and C ( x 3 , y 3 ) be vertices of DABC, Since, m1 is a root of Eq. (i).
then its centroid is
\ am13 + m1 (2a - h ) + k = 0
æ x1 + x2 + x 3 y1 + y2 + y 3 ö æ x1 + x2 + x 3 ö 1/ 3
ç , ÷ =ç , 0÷ . æk ö æk ö
è 3 3 ø è 3 ø Þ aç ÷+ç ÷ (2a - h ) + k = 0
è 2a ø è 2a ø
Since, y 1 + y 2 + y 3 = 0 (from corollary 2). Hence, the é 1/ 3
æk ö ù
centroid lies on the X-axis OX, which is the axis of the ê putting m1 = ç ÷ ú
êë è 2a ø ú
parabola also. û
1/ 3
x + x2 + x 3 1 æk ö 3k
Now, 1 = (am 12 + am 22 + am 23 ) Þ ç ÷ (2a - h ) = -
3 3 è 2a ø 2
a 2 k 27k 3
= (m 1 + m 22 + m 23 ) Þ (2a - h )3 = -
3 2a 8
a Þ 27ak 2 = 4 (h - 2a )3
= {(m 1 + m 2 + m 3 ) 2
3 Hence, the locus of (h , k ) is
- 2 (m 1m 2 + m 2m 3 + m 3m 1 )} 27ay 2 = 4 ( x - 2a )3 .
a é 2 ì2a - h üù 2h - 4a
= ê(0 ) - 2 í a ýú = 3 y Example 43 Find the locus of the point through
3 ë î þû which pass three normals to the parabola y 2 = 4ax
æ 2h - 4a ö such that two of them make angles a and b
\ Centroid of DABC is ç , 0÷ . respectively with the axis such that tan a tan b = 2 .
è 3 ø
Sol. Let (h , k ) be the point of intersection of three normals to
y Example 42 Show that the locus of points such the parabola y 2 = 4ax .
that two of the three normals drawn from them to The equation of any normal to y 2 = 4ax is
the parabola y 2 = 4ax coincide is 27ay 2 = 4 ( x - 2a ) 3 . y = mx - 2am - am 3
Sol. Let (h , k ) be the point of intersection of three normals to If it passes through (h1,k ), then
the parabola y 2 = 4ax . The equation of any normal to k = mh - 2am - am 3
y 2 = 4ax is Þ am 3 + m (2a - h ) + k = 0 …(i)
Chap 05 Parabola 393

Let roots of Eq. (i) be m1, m 2 , m 3 then from Eq. (i) Then, VA = 2a + am12 , VB = 2a + am 22 and
k VC = 2a + am 32 .
m1m 2m 3 = - …(ii)
a Given, VA , VB and VC are in AP.
Also m1 = tan a , m 2 = tan b and tan a tan b = 2 \ 2VB = VA + VC
\ m1m 2 = 2 …(iii) Þ 4a + 2am 22 = 2a + am12 + 2a + am 32
k
From Eqs. (ii) and (iii), 2m 3 = - Þ 2m 22 = m12 + m 32
a
k Þ 2m 22 = (m1 + m 3 )2 - 2m1m 3
or m3 = -
2a 2m1m 2m 3
Þ 2m 22 = (m1 + m 2 + m 3 - m 2 )2 -
Which being a root of Eq. (i) must satisfy it m2
i.e. am 33 + m 3 (2a - h ) + k = 0 2 æ kö
Þ 2m 22 = (0 - m 2 )2 - ç- ÷
æ kö k
3 m2 è a ø
Þ a ç- ÷ - (2a - h ) + k = 0
è 2a ø 2a [from Eqs. (iii) and (v)]
3 2k
k kh Þ m 23 = …(vi)
Þ - 2
-k + +k =0 a
8a 2a
(2a - h )
Þ k 2 - 4ah = 0 Now, from Eq. (iv), m 2 (m1 + m 3 ) + m 3m1 =
a
\ Required locus of (h , k ) is y 2 - 4ax = 0. m1m 2m 3 (2a - h )
Þ m 2 (m1 + m 2 + m 3 - m 2 ) + =
m2 a
y Example 44 If the three normals from a point to the
k (2a - h )
parabola y 2 = 4ax cut the axis in points whose Þ m 2 (0 - m 2 ) - =
am 2 a
distance from the vertex are in AP, show that the point
Þ - am 23 - k = m 2 (2a - h )
lies on the curve 27ay 2 = 2 ( x - 2a ) 3 .
Þ ( - am 23 - k )3 = m 23 (2a - h )3
Sol. Let (h , k ) be the point of intersection of three normals to
2k
the parabola y 2 = 4ax . The equation of any normal to Þ ( -2k - k )3 = (2a - h )3 [from Eq. (vi)]
a
y 2 = 4ax is y = mx - 2am - am 3 …(i) 2k
Þ - 27k 3 = - (h - 2a )3
Y a
Þ 27ak 2 = 2 (h - 2a )3
Hence, locus of (h , k ) is
B C 27ay 2 = 2 ( x - 2a )3 .
X′ A X
V
P (h , k )
y Example 45 The normals at P , Q , R on the parabola
Y′
y 2 = 4ax meet in a point on the line y = k. Prove that
If it passes through (h , k ) then
the sides of the DPQR touch the parabola
x 2 - 2ky = 0.
k = mh - 2am - am 3
Sol. Any normal to the parabola y 2 = 4ax is
Þ am 3 + m (2a - h ) + k = 0 …(ii)
Let roots of Eq. (ii) be m1, m 2 , m 3 then from Eq. (ii) y = mx - 2am - am 3 …(i)
m1 + m 2 + m 3 = 0 …(iii) Also, any point on the line y = k is ( x 1, k ).
(2a - h ) If Eq. (i) passes through ( x 1, k ) then k = mx 1 - 2am - am 3
m1m 2 + m 2m 3 + m 3m1 = …(iv)
a or am 3 + m (2a - x 1 ) + k = 0
k
and m1m 2m 3 = - …(v) If the roots of this equation are m1, m 2 , m 3 then we get
a
m1 + m 2 + m 3 = 0 …(ii)
Since, Eq. (i) cuts the axis of parabola viz. y = 0 at
(2a - x 1 )
(2a + am 2 , 0). m1m 2 + m 2m 3 + m 3m1 = …(iii)
a
\The normal through (h , k ) cut the axis at A (2a + am12 , 0), k
and m1m 2m 3 = - …(iv)
B (2a + am 22 , 0) and C (2a + am 32 , 0) and let V (0, 0) be the a
vertex of the parabola y 2 = 4ax .
394 Textbook of Coordinate Geometry

Also, coordinates of three points P , Q and R are


(am 12, - 2am1 ) (am 22 , - 2am 2 ) and (am 32 , - 2am 3 ), Circle Through Co-normal Points
respectively.
To find the equation of the circle passing through the three
\ The equation of the line PQ is
(conormal) points on the parabola, normals at which pass
( -2am 2 ) - ( -2am1 )
y - ( -2am1 ) = ( x - am12 ) through a given point (a, b).
am 22 - am12 Y
2 B
Þ y + 2am1 = - ( x - am12 ) A
(m 2 + m1 )
Þ y (m1 + m 2 ) + 2am1(m1 + m 2 ) = - 2x + 2am12 (α,β)
X
O
Þ y (m1 + m 2 ) + 2am1m 2 = - 2x
2am1m 2m 3
Þ y (m1 + m 2 + m 3 - m 3 ) + = - 2x
m3 C
2k
Þ y (0 - m 3 ) - = - 2x [from Eqs. (ii) and (iv)] Let A (am 12 , - 2am 1 ), B (am 22 , - 2am 2 ) and C (am 23 , - 2am 3 )
m3
be the three points on the parabola
Þ - ym 32 - 2k = - 2m 3 x
y 2 = 4ax
Þ ym 32 - 2m 3 x + 2k = 0,
which is a quadratic in m 3 . Since, point of intersection of normals is (a, b), then
Since, PQ will touch it, then am 3 + (2a - a ) m + b = 0 …(E)
B 2 - 4 AC = 0 \ m1 + m2 + m 3 = 0 …(i)
Þ ( -2x )2 - 4 × y × 2k = 0 (2a - a )
m 1m 2 + m 2m 3 + m 3m 1 = … (ii)
\ x 2 - 2ky = 0 a
y Example 46 Find the point on the axis of the b
and m 1m 2m 3 = - …(iii)
parabola 3y 2 + 4 y - 6 x + 8 = 0 from when three a
distinct normals can be drawn. Let the equation of the circle through A, B, C be
Sol. Given, parabola is 3y 2 + 4y - 6x + 8 = 0 x 2 + y 2 + 2 gx + 2 fy + c = 0 …(iv)
2
æ 4 ö If the point (am , - 2am ) lies on it, then
Þ 3 çy 2 + y ÷ = 6x - 8
è 3 ø (am 2 ) 2 + ( -2am ) 2 + 2 g (am 2 ) + 2 f ( -2 am ) + c = 0
ìï æ 2
2ö 4 üï Þ a 2m 4 + ( 4a 2 + 2ag ) m 2 - 4afm + c = 0 …(v)
Þ 3 í çy + ÷ - ý = 6x - 8
ïî è 3ø 9ï
þ This is a biquadratic equation in m. Hence, there are four
2
æ 2ö æ 4ö values of m, say m 1 , m 2 , m 3 and m 4 such that the circle
Þ 3 çy + ÷ = ç6x - 8 + ÷
è 3ø è 3ø pass through the points.
æ 2ö
2
æ 10 ö A (am 12 , - 2am 1 ), B (am 22 , - 2am 2 ), C (am 23 , -2am 3 ) and
\ çy + ÷ = 2 ç x - ÷
è 3ø è 9ø D (am 24 , - 2am 4 ).
2 10 \ m1 + m2 + m 3 + m 4 = 0 …(F)
Let y + =Y, x - =X
3 9
Þ 0 +m4 = 0 [from Eq. (i)]
Then, Y 2 = 2X
Þ m4 = 0
Comparing with Y 2 = 4aX 2
\ (am 4 , - 2am 4 ) = (0, 0 )
1
\ a= Thus, the circle passes through the vertex of the parabola
2
æ 2ö y 2 = 4ax from Eq. (iv),
any point on the axis of parabola is ç x, - ÷
è 3ø 0 +0 +0 +0 +c =0
10 \ c =0
and X > 2a Þ x - >1
9 From Eq. (v), a m + ( 4a + 2ag ) m 2 - 4afm = 0
2 4 2
19
Þ x> Þ am 3 + ( 4a + 2 g ) m - 4 f = 0 …(vi)
9
Chap 05 Parabola 395

Now, Eqs. (E) and (vi) are identical. Remark


4a + 2 g 4f This is likewise true for the pairs of chords AB, CD and AD, BC.
\ 1= =-
2a - a b Corollary 3. The circle through conormal point passes
\ 2 g = - (2a + a ), 2 f = - b/ 2 through the vertex (0, 0 ) of the parabola.
\ The equation of the required circle is Corollary 4. The centroid of four points; in which a circle
b intersects a parabola, lies on the axis of the parabola.
x 2 + y 2 - (2a + a ) x - y = 0
2 æ 4 2
4 ö
ç åam i å( -2am i ) ÷
[from Eq. (iv)] ç i =1 i =1 ÷
Corollary 1. The algebraic sum of the ordinates of the four Centroid = ç , ÷
4 4
points of intersection of a circle and a parabola is zero. ç ÷
ç ÷
Sum of ordinates è ø
= -2am 1 - 2am 2 - 2am 3 - 2am 4 æa a ö
= ç {( Sm 1 ) 2 - 2 Sm 1m 2 }, - ( Sm 1 ) ÷
= - 2a (m 1 + m 2 + m 3 + m 4 ) [from Eq. (F)] è4 2 ø
= - 2a ´ 0 = 0 æa æ 2 ( 4a 2 + 2ag ) ö ö
Corollary 2. The common chords of a circle and a parabola = çç ç 0 - ÷ , 0 ÷÷
è4 è a2 ø ø
are in pairs equally inclined to the axis of the parabola.
ψ = ( -2a - g, 0 )
Here y = 0, which is axis of the parabola y 2 = 4ax .
ψ
y Example 47 A circle cuts the parabola y 2 = 4ax at
Y right angles and passes through the focus, show that
A its centre lies on the curve y 2 (a + 2x ) = a (a + 3x ) 2 .
B
φ φ θ θ Sol. Let the circle x 2 + y 2 + 2gx + 2 fy + c = 0 …(i)
O X meet the parabola y 2 = 4ax at any point P (at 2 , 2at ) cutting
C it at right angles.
D We have to find locus of centre of circle Eq. (i),
i.e. (-g, - f )
Let A, B, C, D be the points of intersection of the circle and But given circle Eq. (i) passes through the focus (a , 0), then
the parabola with A (am 12 , - 2am 1 ), B (am 22 , - 2am 2 ) a 2 + 0 + 2ga + 0 + c = 0
C (am 23 , - 2am 3 ) and D (am 24 , - 2am 4 ) then equation of AC Þ c = - a 2 - 2ag …(ii)

and BD are
90°
y (m 1 + m 3 ) = - 2 x - 2am 1m 3 P
and y (m 2 + m 4 ) = - 2 x - 2am 2m 4 , respectively.
\ Slopes of the chords AC and BD are
2 2 S(a,0)
- and - , respectively.
m1 + m 3 m2 + m 4
2 Now, the circle and parabola intersect at P (at 2 , 2at ) at
\ Slope of AC = -
m1 + m 3 right angles.
2 Since, tangent at P (at 2 , 2at ) to the parabola y 2 = 4ax is
= [Qm 1 + m 2 + m 3 + m 4 = 0]
m2 + m 4 ty = x + at 2
æ 2 ö Hence, this tangent must pass through the centre ( - g , - f )
= - ç- ÷ = - Slope of BD of the circle
è m2 + m 4 ø
\ - ft = - g + at 2
\ Their slopes are equal in magnitude and opposite in
Þ ft = g - at 2 …(iii)
sign.
2
\ The chords of AC and BD are equally inclined to the axis. Also the point P (at , 2at ) lies on the circle (i), then
396 Textbook of Coordinate Geometry

a 2t 4 + 4a 2t 2 + 2agt 2 + 4aft + c = 0 æ a - 2g ö æ a - 2g ö
f ç ÷ = g -aç ÷
Hence, from Eqs. (ii) and (iii) when we put values for c and ft. è a ø è a ø
\ a 2t 4 + 4a 2t 2 + 2agt 2 + 4ag - 4a 2t 2 - a 2 - 2ag = 0
æ a - 2g ö
Þ a 2t 4 + 2agt 2 + 2ag - a 2 = 0 Þ f ç ÷ = ( 3g - a )
è a ø
Þ at 4 + 2gt 2 + 2g - a = 0 Þ f 2 ( a - 2g ) = a ( 3g - a ) 2
4 2
Þ a ( t - 1) + 2g ( t + 1) = 0 Þ ( - f )2 [a + 2 ( - g )] = a (a - 3g )2
2 2 2
Þ (t + 1) [a (t - 1) + 2g ] = 0 But t + 1 ¹ 0 Þ ( - f )2 [a + 2 ( - g )] = a [a + 3 ( - g )]2
a - 2g Hence, locus of the centre ( - g , - f ) is the curve
Þ a ( t 2 - 1) + 2g = 0 Þ t 2 =
a y 2 ( a + 2x ) = a ( a + 3x ) 2 .
Hence, from Eq. (iii),

Exercise for Session 2


1. If 2x + y + l = 0 is a normal to the parabola y 2 = - 8x , then the value of l is
(a) - 24 (b) - 16 (c) - 8 (d) 24
2. 2
The slope of a chord of the parabola y = 4ax which is normal at one end and which subtends a right angle at
the origin is
1 1
(a) (b) 2 (c) - (d) - 2
2 2

3. The common tangent to the parabola y 2 = 4ax and x 2 = 4ay is


(a) x + y + a = 0 (b) x + y - a = 0 (c) x - y + a = 0 (d) x - y - a = 0
4. 2 2 2
The circle x + y + 4lx = 0 which l ÎR touches the parabola y = 8x . The value of l is given by
(a) l Î (0, ¥) (b) l Î (-¥, 0) (c) l Î(1, ¥) (d) l Î(-¥, 1)
5. 2
If the normals at two points P and Q of a parabola y = 4ax intersect at a third point R on the curve, then the
product of ordinates of P and Q is
(a) 4a 2 (b) 2a 2 (c) - 4a 2 (d) 8a 2

6. The normals at three points P , Q, R of the parabola y 2 = 4ax meet in (h, k ). The centroid of DPQR lies on
(a) x = 0 (b) y = 0 (c) x = - a (d) y = a
7. The set of points on the axis of the parabola y 2 - 4x - 2y + 5 = 0 from which all the three normals to the
parabola are real, is
(a) (l, 0) ; l > 1 (b) (l, 1) ; l > 3 (c) (l, 2) ; l > 6 (d) (l, 3) ; l > 8
8. Prove that any three tangents to a parabola whose slopes are in harmonic progression enclose a triangle of
constant area.
9. A chord of parabola y 2 = 4ax subtends a right angle at the vertex. Find the locus of the point of intersection of
tangents at its extremities.
10. Find the equation of the normal to the parabola y 2 = 4x which is
(a) parallel to the line y = 2x - 5. (b) perpendicular to the line 2x + 6y + 5 = 0.
11. The ordinates of points P and Q on the parabola y 2 = 12x are in the ratio 1 : 2. Find the locus of the point of
intersection of the normals to the parabola at P and Q.
12. The normals at P , Q, R on the parabola y 2 = 4ax meet in a point on the line y = c . Prove that the sides of the
DPQR touch the parabola x 2 = 2cy .

13. The normals are drawn from (2l, 0) to the parabola y 2 = 4x . Show that l must be greater than 1. One normal is
always the X-axis. Find l for which the other two normals are perpendicular to each other.
Session 3
Pair of Tangents SS1 = T 2 , Chord of Contact, Equation of the Chord Bisected at
a Given Point, Diameter, Lengths of Tangent, Subtangent, Normal and Subnormal,
Some Standard Properties of the Parabola, Reflection Property of a Parabola,
Study of Parabola of the Form (ax 2 + by)2 + 2gx + 2fy + c = 0

Pair of Tangents SS1 = T 2 Let P ( x 1 , y 1 ) be any point outside the parabola. Let a
chord of the parabola through the point P ( x 1 , y 1 ) cut the
If y 12 - 4ax 1 > 0, then any point P ( x 1 , y 1 ) lies outside the parabola at Q and let R (h, k ) be any arbitrary point on the
parabola and a pair of tangents PQ , PR can be drawn to it line PQ ( R inside or outside).
from P. We find their equation as follows. Let Q divide PR in the ratio l : 1, then coordinates of Q are
Let T (h, k ) be any point on the pair of tangents PQ or PR æ lh + x 1 lk + y 1 ö
drawn from any external point P ( x 1 , y 1 ) to the parabola ç , ÷ [QPQ : QR = l : 1]
è l +1 l +1 ø
y 2 = 4ax .
Since, Q lies on parabola Eq. (i), then
Q 2
k)
T (h, æ lk + y 1 ö æ lh + x 1 ö
ç ÷ = 4a ç ÷
P (x è l +1 ø è l +1 ø
1,y)
1

R
R (h, k)
Equation of PT is
k - y1 Q
y - y1 = (x - x 1 )
h - x1
æ k - y1 ö æ hy 1 - kx 1 ö
Þ y =ç ÷ x +ç ÷
èh - x1 ø è h - x1 ø P (x1 , y1 )

which is tangent to the parabola


y 2 = 4ax Þ ( lk + y 1 ) 2 - 4a ( lh + x 1 ) ( l + 1) = 0
a Þ (k 2 - 4ah ) l2 + 2 [ky 1 - 2a (h + x 1 )]l
\ c=
m
+ (y 12 - 4ax 1 ) = 0 …(ii)
æ hy - kx 1 ö æ k - y 1 ö
Þ cm = a Þ ç 1 ÷ç ÷ =a Line PR will become tangent to parabola Eq. (i), then roots
è h - x1 ø èh - x1 ø
of Eq. (ii) are equal
Þ (k - y 1 ) (hy 1 - kx 1 ) = a (h - x 1 ) 2
\ 4 [ky 1 - 2a (h + x 1 )]2 - 4 (k 2 - 4ah ) (y 12 - 4ax 1 ) = 0
\ Locus of (h, k ), equation of pair of tangents is
or {ky 1 - 2a (h + x 1 )} 2 = (k 2 - 4ah ) (y 12 - 4ax 1 )
(y - y 1 ) ( xy 1 - x 1 y ) = a ( x - x 1 ) 2
Hence, locus of R (h, k ) i.e. equation of pair of tangents
Þ (y 2 - 4ax ) (y 12 - 4ax 1 ) = {yy 1 - 2a ( x + x 1 )} 2 from P( x 1 , y 1 ) is
\ SS 1 = T 2 {yy 1 - 2a ( x + x 1 )} 2 = (y 2 - 4ax ) (y 12 - 4ax 1 )
where S = y 2 - 4ax , S 1 = y 12 - 4ax 1 i.e. T 2 = SS 1 or SS 1 = T 2
and T = yy 1 - 2a ( x + x 1 ).
Remark
Aliter :
S = 0 is the equation of the curve, S1 is obtained from S by
Let the parabola be y 2 = 4ax …(i) replacing x by x1 and y by y1 and T = 0 is the equation tangent at
( x1, y1 ) to S = 0.
398 Textbook of Coordinate Geometry

Chord of Contact Let P º ( x 1, y1 )


and the tangents from P touch the parabola at Q (at 12 , 2at 1 )
The chord joining the points of contact of two tangents and R(at 22 , 2at 2 ) then P is the point of intersection of
drawn from an external point to a parabola is known as the tangents.
chord of contact of tangents drawn from external point. \ x 1 = at 1t 2 and y1 = a(t 1 + t 2 )
Theorem The chord of contact of tangents drawn from a x y
Þ t 1t 2 = 1 and t 1 + t 2 = 1 …(ii)
point ( x 1 , y 1 ) to the parabola y 2 = 4ax is a a
Now, QR = (at 12 - at 22 )2 + (2at 1 - 2at 2 )2
yy 1 = 2a ( x + x 1 ).
Proof Let PQ and PR be tangents to the parabola y 2 = 4ax = a 2 (t 1 - t 2 )2 [(t 1 + t 2 )2 + 4 ]

drawn from any external point P ( x 1 , y 1 ), then QR is called = | a || t 1 - t 2 | {(t 1 + t 2 )2 + 4 }


chord of contact of the parabola y 2 = 4ax .
= | a | {(t 1 + t 2 )2 - 4t 1t 2 } {(t 1 + t 2 )2 + 4 }
Y
′) æy 2 4x ö æy 2 ö
,y
(x′ = | a | ç 12 - 1 ÷ × ç 12 + 4 ÷
Q èa a ø èa ø
[from Eq. (ii)]
X′ X
A
(y12 - 4ax 1 ) (y12 + 4a )2
P Chord of contact =|a| ×
(x1, y1) |a| |a|
R (x′′, 1
y′′)
Y′
= (y12 - 4ax 1 )(y12 + 4a 2 )
|a|
Let Q º ( x ¢ , y ¢ ) and R º ( x ¢¢,y ¢¢ ) Aliter :
Equation of tangent PQ is Equation of QR is yy1 = 2a ( x + x 1 )
yy ¢ = 2a ( x + x ¢ ) …(i) yy - 2ax 1
Þ x= 1
2a
and equation of tangent PR is
The ordinates of Q and R are the roots of the equation
yy ¢¢ = 2a ( x + x ¢¢ ) …(ii)
æ yy - 2ax 1 ö
Since, lines Eqs. (i) and (ii) pass through ( x 1 , y 1 ), then y 2 = 4a ç 1 ÷
è 2a ø
y 1 y ¢ = 2a ( x 1 + x ¢ ) and y 1 y ¢¢ =2a ( x 1 + x ¢¢ ) Þ y 2 = 2 (yy1 - 2ax 1 ) … (i)
Hence, it is clear Q ( x ¢ , y ¢ ) and R ( x ¢¢,y ¢¢ ) lie on Þ 2
y - 2yy1 + 4ax 1 = 0
yy 1 = 2a ( x + x 1 )
\ k1 + k 2 = 2y1 and k1k = 4ax 1
which is chord of contact QR.
\ (k 2 - k1 ) = (k1 + k 2 )2 - 4k1k 2
y Example 48 Tangents are drawn from the point = ( 4y12 - 16ax 1 ) = 2 (y12 - 4ax 1 ) …(ii)
( x 1 , y 1 ) to the parabola y 2 = 4ax , show that the length
1 Y
of their chord of contact is ( y 12 - 4ax 1 )( y 12 + 4a 2 ). Q (h1, k1)
|a|
Sol. Given parabola is
y 2 = 4ax …(i) X′
A
X

Q (at12, 2at1) P (x R (h 2 , k 2 )
Y 1 , y1 )

) Y′
, y1
P (x 1 Since, Q (h1, k1 ) and R (h 2 , k 2 ) lie on the parabola y 2 = 4ax ,
A therefore
X′ X
k12 = 4ah1 and k 22 = 4ah 2
R (at22, 2at2)
Þ k 22 - k12 = 4a (h 2 - h1 )
Y′ (k 2 + k1 ) (k 2 - k1 ) = 4a (h 2 - h1 )
Chap 05 Parabola 399

Þ 2y1 (k 2 - k1 ) = 4a (h 2 - h1 ) Proof Since, equation of the parabola is


y (k - k 1 )
Þ (h 2 - h1 ) = 1 2 …(iii) y 2 = 4ax …(i)
2a
Let QR be the chord of the parabola whose mid-point is
Now, QR = (k 2 - k1 )2 + (h 2 - h1 )2
P ( x 1 , y 1 ).
y12 (k 2 - k1 )2 Q (x2, y2)
= (k 2 - k 1 )2 + 2
[from Eq. (iii)]
4a
(k 2 - k 1 )
= ( y 1 + 4a 2 )
2
2|a | P (x1, y1)
2 (y12 - 4ax 1 )
= (y12 + 4a 2 ) [from Eq. (ii)]
2 |a |
R
1 (x3, y3)
= (y12 - 4ax 1 ) (y12 + 4a 2 )
|a| Since, Q and R lie on parabola (i),
y 22 = 4ax 2 and y 23 = 4ax 3
y Example 49 Prove that the area of the triangle
formed by the tangents drawn from ( x 1 , y 1 ) to y 2 = 4ax \ y 23 - y 22 = 4a ( x 3 - x 2 )
and their chord of contact is ( y 12 - 4ax 1 ) 3/ 2 / 2a . y 3 - y2 4a
Þ =
Sol. Equation of QR (chord of contact) is x 3 - x2 y 3 + y2
yy1 = 2a ( x + x 1 ) 4a
= [QP ( x 1 , y 1 ) is mid-point of QR]
Þ yy1 - 2a ( x + x 1 ) = 0 2y 1
Q PM = Length of perpendicular from P ( x 1, y1 ) on QR
y 3 - y 2 2a
| y y - 2a ( x 1 + x 1 ) | |(y12 - 4ax 1 )| \ = = Slope of QR
= 1 1 = x 3 - x2 y1
(y12 + 4a 2 ) (y12 + 4a 2 )
2a
[Since, P ( x 1, y1 ) lies outside the parabola. So, y12 - 4ax 1 > 0] Equation of QR is y - y 1 = (x - x 1 )
y1
Q
Þ yy 1 - y 12 = 2ax - 2ax 1
Þ yy 1 - 2a ( x + x 1 ) = y 12 - 4ax 1
P(x
1, y1 ) [subtracting 2ax 1 from both sides]
\ T = S1,
R
where T = yy 1 - 2a ( x + x 1 ) and S 1 = y 12 - 4ax 1 .
1
Now, area of DPQR = QR × PM
2 y Example 50 Find the locus of the mid-points of the
=
1 1 (y 2 - 4ax 1 )
(y12 - 4ax 1 ) (y12 + 4a 2 ) 1
chords of the parabola y 2 = 4ax which subtend a right
2 |a| (y12 + 4a 2 ) angle at the vertex of the parabola.
= (y12 - 4ax 1 )3 / 2 / 2a, if a > 0 Sol. Let P (h , k ) be the mid-point of a chord QR of the parabola
y 2 = 4ax then equation of chord QR is
T = S1
Equation of the Chord Bisected or yk - 2a( x + h ) = k 2 - 4ah

at a Given Point Þ yk - 2ax = k 2 - 2ah


If A is the vertex of the parabola. For combined equation of
…(i)

Theorem The equation of the chord of the parabola AQ and AR making homogeneous of y 2 = 4ax with the help
y 2 = 4ax which is bisected at ( x 1 , y 1 ) is of Eq. (i).
yy 1 - 2a ( x + x 1 ) = y 12 - 4ax 1 \ y 2 = 4ax
æ yk - 2ax ö
or T = S1, Þ y 2 = 4ax ç 2 ÷
è k - 2ah ø
where, T = yy 1 - 2a ( x + x 1 ) and S 1 = y 12 - 4ax 1 .
400 Textbook of Coordinate Geometry

y 2 (k 2 - 2ah ) - 4akxy + 8a 2 x 2 = 0
Þ
Q
Diameter
The locus of the middle points of a system of parallel chords
90°
is called a diameter and in case of a parabola this diameter
(0, 0) A
P (h, k) is shown to be a straight line which is parallel to the axis of
the parabola.
R Theorem The equation of the diameter bisecting chords
Since, ÐQAR = 90° 2a
of slope m of the parabola y 2 = 4ax is y = .
\ Coefficient of x 2 + Coefficient of y 2 = 0 m
k 2 - 2ah + 8a 2 = 0 Proof Let y = mx + c be system of parallel chords to
Hence, the locus of P (h , k ) is y 2 - 2ax + 8a 2 = 0. y 2 = 4ax for different chords c varies, m remains constant.
Let the extremities of any chord PQ of the set be P ( x 1 , y 1 )
y Example 51 Show that the locus of the middle points and Q ( x 2 , y 2 ) and let its middle point be M (h, k ).
of normal chords of the parabola y 2 = 4ax is
Y
y 4 - 2a ( x - 2a ) y 2 + 8a 4 = 0. P
(x1 , y1 )

Sol. Equation of the normal chord at any point (at 2 , 2at ) of the y = mx + c
M
parabola y 2 = 4ax is X′ X
A
y + tx = 2at + at 3 …(i)

t, 2
2 at) Q (x2, y2)
R (a Y′

M (x1 , y1)
On solving equations
y 2 = 4ax and y = mx + c .
Q
æy -c ö
But if M ( x 1, y1 ) be its middle point its equation must be \ y 2 = 4a ç ÷
also è m ø
T = S1 \ my 2 - 4ay + 4ac = 0
Þ yy1 - 2a ( x + x 1 ) = y12 - 4ax 1 y + y 2 2a
4a
Þ yy1 - 2ax = y12 - 2ax 1 …(ii)
\ y1 + y2 = or 1 =
m 2 m
Q From Eqs. (i) and (ii) are identical, comparing, them [Q(h, k ) is the mid-point of PQ]
1 t 2at + at 3 2a
= = 2 Hence, locus of M (h, k ) is y = .
y1 -2a y1 - 2ax 1 m
2a Aliter :
From first two relations, t = - …(iii)
y1 Let (h, k ) be the middle point of the chord y = mx + c of the
t 2at + at 3 parabola y 2 = 4ax then
From last two relations, = 2
-2a y1 - 2ax 1 T = S 1 Þ ky - 2a( x + h ) = k 2 - 4ah
y12 - 2ax 1 2a 2a
Þ = 2a + at 2 slope = =m Þ k =
-2a k m
2
y12 - 2ax 1 æ -2a ö 2a
Þ = 2a + a ç ÷ [from Eq. (iii)] Hence, locus of the mid-point is y = .
-2a è y1 ø m
y12 - 2ax 1 2ay12 + 4a 3 Remarks
Þ =
-2a y12 1. The point in which any diameter meets the curve is called
the extremity of the diameter.
Þ y14 - 2ax 1y12 = - 4a 2y12 - 8a 4
2. Any line which is parallel to the axis of the parabola drawn
Þ y14 - 2a ( x 1 - 2a ) y12 + 8a 4 = 0 through any point on the parabola is called diameter and its
equation is y-coordinate of that point.
Hence, the locus of middle point ( x 1, y1 ) is
If point on diameter ( x1, y1 ), then diameter is y = y1.
y 4 - 2a ( x - 2a ) y 2 + 8a 4 = 0.
Chap 05 Parabola 401

Corollary 1. The tangent at the extremity of a diameter of Corollary 3. To find the equation of a parabola when the
a parabola is parallel to the system of chords it bisects. axes are any diameter and the tangent to the parabola at the
point where this diameter meets the curve.
y = mx + c
a , 2a
Diameter
Let the equation of the parabola be
m2 m
y 2 = 4ax …(i)
Let AB be the diameter of the parabola Eq. (i), then its
2a
Let y = mx + c (c variable) represents the system of parallel equation is y =
2a m
chords, then the equation of diameter of y 2 = 4ax is y = . Y
m
æ a 2a ö a , 2a
The diameter meets the parabola y 2 = 4ax at ç , ÷ m2 m T P
èm 2 m ø A θ θ
B
a Q M
and tangent is y = mx + which is parallel to y = mx + c .
m X
S N
Corollary 2. Tangents at the end of any chord meet on the
diameter which bisects the chords.
If extremities of the chord be P (at 12 , 2at 1 ) and Q (at 22 , 2at 2 )
Since, A is the extremity of the diameter
then its slope
2at 2 - 2at 1 æ a 2a ö
2 \ Coordinates of A is ç , ÷
m= Þ m= . èm 2 m ø
2 2 (t 2 + t 1 )
at 2 - at 1
where, m = tanq
y = mx + c 2 2at 1)
,
P (at 1 then, the equation of tangent AT at A is
a
y = mx + .
m
R Diameter
2a Now, let P be any point on the parabola Eq. (i), whose
(x1, y1) y=
m
coordinates referred to Vx and Vy are ( x , y ) and referred
Q to diameter AB and tangent AT are ( X , Y ).
(at22, 2at2)
then X = AQ and Y = QP [since, PQ || AT ]
\ Equation of diameter is Now, VN = VL + LN = VL + AM = VL + AQ + QM
y = 2a / m Þ y = a(t 1 + t 2 ) …(i) a
= + X + QP cosq
Now, tangents at P (at 12 , 2at 1 )
and Q (at 22 , 2at 2 )
meet at a m2
point [at 1 t 2 , a (t 1 + t 2 )] which lies on Eq. (i). a
or x= + X + Y cosq …(ii)
Aliter m2
Let equation of any chord PQ be y = mx + c . and PN = PM + MN = PM + AL
If tangents at P and Q meet at R ( x 1 , y 1 ), then PQ is the 2a
= QP sinq +
chord of contact with respect to R ( x 1 , y 1 ) . m
\ Equation of PQ is 2a
\ y = Y sinq + … (iii)
2a 2ax 1 m
yy 1 = 2 x ( x + x 1 ) or y = x+
y1 y1 From Eqs. (ii) and (iii) coordinates of P are
which is identical to y = mx + c æ a 2a ö
ç 2 + X + Y cos q, Y sinq + ÷
2a 2a èm mø
m= or y 1 = .
y1 m Now, P lies on Eq. (i).
2a 2
Hence, locus of R ( x 1 , y 1 ) is y = , which bisects the æ 2a ö æ a ö
chord PQ. m \ çY sinq + ÷ = 4a ç 2 + X + Y cos q ÷
è m ø è m ø
402 Textbook of Coordinate Geometry

4a 2 4a It meets the diameter through P i.e. X -axis or y = 0, then


Þ Y 2 sin2 q + + Y sinq Eq. (iii) reduces
2 m
m
0 = x + lt 12
2
4a
= + 4aX + 4aY cosq Þ x = - lt 12 = PB …(iv)
2
m
Similarly, - lt 22 = PC …(v)
Þ Y 2 sin2 q + 4a 2 cot 2 q + 4a cos qY 2
\ ( PA ) = ( -lt 1t 2 ) = l2t 12t 22
2
2 2
= 4a cot q + 4aX + 4aY cos q [Qm = tanq ] = ( -lt 12 ) ( -lt 22 ) [from Eqs. (iv) and (v)]
2 2
Þ Y sin q = 4aX = PB × PC

\ Y 2 = ( 4a cosec 2 q ) X ,
which is the required parabola referred to diameter and
tangent at the extremity of the diameter as axes.
Lengths of Tangent, Subtangent,
Remark
Normal and Subnormal
The quantity 4acosec 2q is called the parameter of the diameter Let the parabola y 2 = 4ax . Let the tangent and normal at
AQ. It is equal to length of the chord which is parallel to AT and
passes through the focus. P ( x , y ) meet the axis of parabola at T and G respectively
i.e. a cosec 2q = a( 1 + cot 2 q) = a + a cot 2 q and let tangent at P ( x , y ) makes angle y with the positive
a direction of X-axis.
= a+ 2
m Y
= a + VL = SP
2
1 )
But length of focal chord if P ( at 2, 2at ) is [QS ( a, 0 )] a æçt + ö÷ . x, y
è tø P(
2at - 0 2t 1 90°– ψ
\ tanq = = or t - = 2 cot q ψ
at 2 - a t2 - 1 t X′
T A N G
X
2 2
1 ïì 1 ïü
\ a æçt + ö÷ = a íæçt - ö÷ + 4 ý
è tø è t ø
îï ïþ
Y′
= a{( 2 cot q) 2 + 4}
Then, PT = Length of Tangent
= 4 acosec 2q = 4 × SP
PG = Length of Normal
y Example 52 If the diameter through any point P of a TN = Length of Subtangent
parabola meets any chord in A and the tangent at the and NG = Length of Subnormal
end of the chord meets the diameter in B and C, then If A (0, 0 ) is the vertex of the parabola.
prove that PA 2 = PB ×PC .
Q PN = y
Sol. The equation of the parabola referred to the diameter
through P and tangent at P as axes is
\ PT = PN cosec y = y cosec y
y 2 = 4 lx …(i) PG = PN cosec (90° - y ) = y sec y
where, l = a cosec 2q [from previous corollary] TN = PN cot y = y cot y
Let QR be any chord of the parabola Eq. (i). Let the and NG = PN cot(90° - y ) = y tan y
extremities Q and R be ( lt 12 , 2lt 1 ) and ( lt 22 , 2lt 2 ). 2a
where, tan y = =m [slope of tangent at P ( x , y )]
Then, the equation of QR is y
y (t 1 + t 2 ) - 2x - 2lt 1t 2 = 0 … (ii)
It meets the diameter through P i.e. X -axis or y = 0, then y Example 53 Find the length of tangent, subtangent,
Eq. (ii) reduces normal and subnormal to y 2 = 4ax at (at 2 , 2at ) .
0 - 2x - 2l t 1t 2 = 0 Sol. Q Equation of tangent of (at 2 , 2at ) of parabola y 2 = 4ax is
Þ x = - lt 1t 2 = PA ty = x + at 2
Now, tangent at Q is t 1y = x + lt 12 …(iii)
1
Slope of this tangent m =
t
Chap 05 Parabola 403

Let tangent makes angle y with positive direction of X -axis and adding Eqs. (i) and (ii), we get
1
tan y = ( x + a ) (1 + t 2 ) = 0 Þ 1 + t 2 ¹ 0
t
then t = cot y \ x + a = 0, which is directrix.
\ Length of tangent at (2) The portion of a tangent to a parabola intercepted
(at 2 , 2at ) = 2at cosecy between the directrix and the curve subtends a right
angle at the focus.
= 2at (1 + cot 2 y ) = 2at (1 + t 2 )
The equation of the tangent to the parabola y 2 = 4ax at
Length of normals at
P (at 2 , 2at ) is
(at 2 , 2at ) = 2at sec y = 2at (1 + tan 2 y )
ty = x + at 2 …(i)
= 2a (t 2 + t 2 tan 2 y ) = 2a (t 2 + 1)
Let Eq. (i) meet the directrix x + a = 0 at Q,
Length of subtangent at (at 2 , 2at ) = 2at cot y
æ at 2 - a ö
= 2at 2 then coordinates of Q are ç - a, ÷ , also focus S
è t ø
Length of subnormal at (at 2 , 2at ) = 2at tan y is (a , 0 ).
= 2a Y
(at,2 2at)
P

Some Standard Properties Z


Q

V
90°
S (a, 0)
X

of the Parabola x+a=0

Directrix
(1) The tangents at the extremities of a focal chord
intersect at right angles on the directrix. 2at - 0
\ Slope of SP =
The extremities of a focal chord PQ are at 2 - a
æa 2a ö 2t
P º (at 2 , 2at ) and Q º ç , - ÷ = = m1 (say)
èt 2 t ø 2
t -1
P
T at 2 - a
-0
90°
t t2 - 1
X
and slope of SQ = = = m2 (say)
S (a, 0) -a -a -2 t
Q
\ m 1m 2 = - 1
i.e. SP is perpendicular to SQ i.e. Ð PSQ = 90°
Let parabola is y 2 = 4ax
(3) The tangent at any point P of a parabola bisects
\ Tangent at P (at 2 , 2at ) is the angle between the focal chord through P and the
perpendicular from P to the directrix.
ty = x + at 2 …(i)
Let the tangent at P (at 2 , 2at ) to the parabola y 2 = 4ax
1
Its slope = m 1 (say) meets the axis of the parabola i.e. X-axis or y = 0 at T. The
t
equation of tangent to the parabola y 2 = 4ax at P (at 2 , 2at )
æa 2a ö
and tangent at Q ç , - ÷ is is
èt 2 t ø
ty = x + at 2
y a
- =x + …(ii) Y
t t2 (at2, 2at)
P
2 M θ
Þ - ty = xt + a θ
Its slope - t = m2 (say) θ
X
T Z V S (a, 0)
Q m 1m 2 = - 1
\ Ð PTQ = 90° x+a=0
Directrix
404 Textbook of Coordinate Geometry

For coordinates of T solve it with y = 0. \ SP = PM = a + at 2


\ T ( -at 2 , 0 ) SG = VG - VS = 2a + at 2 - a
\ ST = SV + VT = a + at 2 = a (1 + t 2 ) = a + at 2
Also, SP = PM = a + at 2 = a (1 + t 2 ) and ST = VS + VT = a + at 2
\ SP = ST , i.e. ÐSTP = ÐSPT Hence, SP = SG = ST
But ÐSTP = Ð MPT [alternate angles] (6) If S be the focus and SH be perpendicular to the
ÐSPT = Ð MPT tangent at P, then H lies on the tangent at the vertex
(4) The foot of the perpendicular from the focus on any and SH 2 = OS × SP , where O is the vertex of the
tangent to a parabola lies on the tangent at the vertex. parabola.
Equation of tangent at P (at 2 , 2at ) Let P (at 2 , 2at ) be any point on the parabola
On the parabola y 2 = 4ax y 2 = 4ax … (i)
2 2
is ty = x + at then, tangent at P (at , 2at ) to the parabola Eq. (i) is
2
Þ x - ty + at = 0 …(i) ty = x + at 2
Now, the equation of line through S (a, 0 ) and It meets the tangent at the vertex i.e. x = 0.
perpendicular to Eq. (i) is Y
x+a=0 P (at2, 2at)
tx + y = l
M
Since, it passes through (a, 0 ). H
X
\ ta + 0 = l O S (a, 0)

\ Equation tx + y = ta or t 2 x + ty - at 2 = 0 …(ii)
Directrix
By adding Eqs. (i) and (ii), we get
x (1 + t 2 ) = 0 \ Coordinates of H is (0, at )
Þ x =0 [Q1 + t 2 ¹ 0 ] and SP = PM = a + at 2 Þ OS = a
Hence, the point of intersection of Eq. (i) and (ii) lies on and SH = (a - 0 ) 2 + (0 - at ) 2 = a 2 + a 2 t 2
x = 0 i.e. on Y-axis (which is tangent at the vertex of a
parabola). or (SH ) 2 = a {a (1 + t 2 )} = OS × SP.
(5) If S be the focus of the parabola and tangent and
normal at any point P meet its axis in T and G Reflection Property of Parabola
respectively then ST = SG = SP .
The tangent ( PT ) and normal ( PN ) of the parabola
Let P (at 2 , 2at ) be any point on the parabola y 2 = 4ax , y 2 = 4ax
then equation of tangent and normal at P (at 2 , 2at ) are
at P are the internal and external bisectors of ÐSPM and
ty = x + at 2 and y = - tx + 2at + at 2 , respectively. BP is parallel to the axis of the parabola and
Since, tangent and normal meet its axis in T and G. Ð BPN = ÐSPN .
Y
\ Coordinates of T ¢ and G are ( -at 2 , 0 ) and (2a + at 2 , 0 ) P Light ray
M β α B
respectively. nt β α
No

Ta nge Reflected ray


Y
rm

S
al

x+a=0 2 X′ X
P (at , 2at) T A N
M Light ray
90° Light ray
Light ray
X′ X
T Z V S G Y′
(a, 0)
All rays of light coming from the positive direction of
Directrix X-axis and parallel to the axis of the parabola after
Y′
reflection pass through the focus of the parabola.
Chap 05 Parabola 405

y Example 54 A ray of light is coming along the line i.e. (slope of ax + by + l = 0) ´ (slope of
y = b from the positive direction of X-axis and strikes xf 1 ( l) + yf 2 ( l) + f 3 ( l) = 0 ) = -1
a concave mirror whose intersection with the a f ( l)
xy-plane is a parabola y 2 = 4ax . Find the equation Þ - ´- 1 = -1
b f 2 ( l)
of the reflected ray and show that it passes through
the focus of the parabola. Both a and b are positive. Þ af 1 ( l) + bf 2 ( l) = 0 …(ii)
Now, substitute the value of l in Eq. (i) from Eq. (ii).
Sol. Given parabola is y 2 = 4ax
Multiply and divide (a 2 + b2 ) in LHS of Eq. (i)
Y ) b4a , b )
2

2
P y=b
α
æ ax + by + l ö
α Normal i.e. (ax + by + l) = (a + b ) ç
2 2 2 ÷
ç a 2 + b2 ÷
2α α 180°–α è ø
X′ X
A S 180°–2α
(a, 0)
and RHS of Eq. (i) by (a 2 + b2 )
i.e. xf 1 ( l) + yf 2 ( l) + f 3 ( l)
Y′
æ b2 ö æ xf ( l) + yf ( l) + f ( l) ö
æ b2 ö
Equation of tangent at P ç , b ÷ is yb = 2a ç x + ÷, = (a 2 + b2 ) ç 1 2 3 ÷
è 4a ø è 4a ø ç 2
(a + b ) 2 ÷
è ø
2a
Slope of tangent is .
b Then, Eq. (i) reduce in the form
b 2
Hence, slope of normal = - = tan (180° - a ). æ ax + by + l ö æ ö
2a ç ÷ = 4r ç bx - ay + m ÷
b ç a 2 + b2 ÷ ç (a 2 + b2 ) ÷
\ tana = è ø è ø
2a
\ Slope of reflected ray = tan (180° - 2a ) which is of the form Y 2 = 4rX
= - tan2a
ax + by + l bx - ay + m 1
ì b ü Y= ,X = and 4r = .
ì 2 tan a ü ïï 2 × ï 4ab
2
(a + b ) 2 2
(a + b ) 2
(a + b2 )
2
= -í = - 2a ï = -
2 ý í 2 ý
î 1 - tan a þ ï1 - b ï ( 4a 2 - b 2 ) 1
îï 4a þï
2 Latusrectum is 4r = .
(a + b2 )
2
Hence, equation of reflected ray is
4ab æ b2 ö Axis is Y = 0 or ax + by + l = 0.
y -b= - ç x - ÷
( 4a 2 - b 2 ) è 4a ø Equation of tangent at vertex is
Þ (y - b ) ( 4a 2 - b 2 ) = - ( 4ax - b 2 ) X = 0 or bx - ay + m = 0.
which obviously passes through the focus S (a , 0). Vertex is the point of intersection of
X = 0 and Y = 0

Study of Parabola of the Form i.e. bx - ay + m = 0 and ax + by + l = 0.


Equation of directrix is X + r = 0.
(ax + by ) 2 +2 gx + 2 fy + c = 0 Equation of latusrectum is X - r = 0.
Given equation can be written as Focus Since, axis and latusrectum intersect at the focus S
its coordinates are detained by solving
(ax + by ) 2 = - 2 gx - 2 fy - c
X - r = 0 and Y = 0.
Now, add an arbitrary constant l in the square root of the
second degree terms. Then the equation will be of the form y Example 55. Find the length of latusrectum of the
i.e. (ax + by + l) 2 = xf 1 ( l) + yf 2 ( l) + f 3 ( l) …(i) parabola (a 2 + b 2 ) ( x 2 + y 2 ) = (bx + ay - ab ) 2 .
Now, choose l such that the lines Sol. Given equation may be written as
ax + by + l = 0 and xf 1 ( l) + yf 2 ( l) + f 3 ( l) = 0 a 2 x 2 + a 2y 2 + b 2 x 2 + b 2y 2 = b 2 x 2 + a 2y 2 + a 2b 2
are perpendicular + 2abxy - 2a 2by - 2ab 2 x
406 Textbook of Coordinate Geometry

Þ a 2 x 2 - 2abxy + b 2y 2 = - 2ab 2 x - 2a 2by + a 2b 2 Þ (x 2 + y 2 ) =


| bx + ay - ab |
P (x, y)
M
æ ab ö (a 2 + b 2 )

bx+ay–ab = 0
2
Þ (ax - by ) = - 2ab çbx + ay - ÷
è 2ø
Þ ( x - 0) 2 + ( y - 0) 2
ab N
Since, ax - by = 0 and bx + ay - = 0 are perpendicular. | bx + ay - ab | S (0, 0)
2 =
ab ö (a 2 + b 2 )
2 æ
æ ö
2 ç ax - by ÷
ç bx + ay - ÷ which is of the form SP = PM .
2
\ (a + b ) 2 2
= - 2ab (a + b ) ç 2 ÷
ç a2 + b2 ÷ 2 2
1
è ø ç (a + b ) ÷ Since, distance from focus S to ( bx + ay - ab = 0) = ( 4r )
è ø 2
1 ab Y
ab ö , y)
2 æ Þ ( 4r ) = P (x
æ ax - by ö -2ab ç bx + ay - ÷ 2 2
a +b 2 N
Þ ç ÷ = ç 2 ÷
ç a2 + b2 ÷ (a 2 + b 2 ) 2 2 2ab
è ø ç (a + b ) ÷ \ 4r = X
è ø O M
(a 2 + b 2 )
which is of the form Y 2 = - 4 rX .
2ab Remark
Therefore, the latusrectum = 4r = .
(a 2 + b 2 ) Consider the equation of parabola is y 2 = 4 ax.

Aliter : i.e. ( MP ) 2 = (Latusrectum) NP.


Given equation may be written as i.e. if P is any point on the given parabola, then
(bx + ay - ab )2 (the distance of P from its axis) 2 = (Latusrectum)
x2 + y2 =
(a 2 + b 2 ) (The distance of P from the tangent at its vertex).

Exercise for Session 3


1. If m1, m 2 are slopes of the two tangents that are drawn from (2, 3) to the parabola y 2 = 4x , then the value of
1 1
+ is
m1 m 2
2 3
(a) –3 (b) 3 (c) (d)
3 2

2. The angle between the tangents drawn from the origin to the parabola y 2 = 4a( x – a ) is
1
(a) 90° (b) 30° (c) tan-1 æç ö÷ (d) 45°
è 2ø

3. If (a, b) is the mid-point of chord passing through the vertex of the parabola y 2 = 4x , then
(a) a = 2b (b) 2a = b (c) a 2 = 2b (d) 2a = b 2

4. The diameter of the parabola y 2 = 6x corresponding to the system of parallel chords 3x - y + c = 0 is


(a) y - 1 = 0 (b) y - 2 = 0 (c) y + 1 = 0 (d) y + 2 = 0

5. From the point ( -1, 2) tangent lines are drawn to the parabola y 2 = 4x , the area of triangle formed by chord of
contact and the tangents is given by
(a) 8 (b) 8 3 (c) 8 2 (d) None of these

6. For parabola x 2 + y 2 + 2xy - 6x - 2y + 3 = 0, the focus is


(a) (1, - 1) (b) (–1, 1) (c) (3, 1) (d) None of these

7. The locus of the mid-point of that chord of parabola which subtends right angle on the vertex will be
(a) y 2 - 2ax + 8a 2 = 0 (b) y 2 = a (x - 4a ) (c) y 2 = 4a (x - 4a ) (d) y 2 + 3ax + 4a 2 = 0
Chap 05 Parabola 407

8. A ray of light moving parallel to the X-axis gets reflected from a parabolic mirror whose equation is
( y - 2)2 = 4( x + 1). After reflection, the ray must pass through the point
(a) (–2, 0) (b) (–1, 2) (c) (0, 2) (d) (2, 0)

9. 2
Prove that the locus of the point of intersection of tangents to the parabola y = 4ax which meet at an angle a is
( x + a )2 tan2 a = y 2 - 4ax .

10. Find the locus of the middle points of the chords of the parabola y 2 = 4ax which pass through the focus.

11. From the point P( -1, 2) tangents are drawn to the parabola y 2 = 4x . Find the equation of the chord of contact.
Also, find the area of the triangle formed by the chord of contact and the tangents.

Shortcuts and Important Results to Remember


1 Second degree terms in the equation of a parabola 9 If the normals to the parabola y 2 = 4ax at the points t1 and
should make perfect squares. t 2 intersect again on the parabola at the point
2 If l1 and l 2 are the lengths of segments of a focal chord ‘t 3 ’, then t1t 2 = 2; t 3 = -(t1 + t 2 ) and the line joining t1 and t 2
4l l passes through a fixed point (-2 a, 0).
then the latusrectum of the parabola is 1 2 .
l1 + l 2 10 If the length of a focal chord of y 2 = 4ax at a distance b
3 If a be the inclination of a focal chord with axis of the from the vertex is c then b2c = 4a3 .
parabola then its length is 4acosec 2 a.
11 From an external point only one normal can be drawn.
4 If tangents of y 2 = 4ax at P(t1 ) and Q(t 2 ) meets at R, then
12 If a normal to y 2 = 4ax makes an angle q with the axis of
1
area of DPQR is a2 (t1 - t 2 )3 . y 2 = 4ax then it will cut the curve again at an angle of
2
æ tan q ö
5 The foot of the perpendicular from the focus on any tan -1 ç ÷.
è 2 ø
tangent to a parabola lies on the tangent at the vertex.
13 The orthocentre of any triangle formed by the three
6 The area of the triangle formed by three points on a
tangents to a parabola y 2 = 4ax ‘t1’, ‘t 2 ’ and ‘t ’3 lies on the
parabola is twice the area of the triangle formed by the
tangents, at these points. directrix and has the coordinates
(- a, a(t1 + t 2 + t 3 + t1t 2t 3 )).
7 The equation of the common tangent to the parabolas
x 2 = 4ay and y 2 = 4ax is x + y + a = 0. 14 Normals at the end points of the latusrectum of a parabola
y 2 = 4ax intersect at right angle on the axis of the parabola
8 If the chord joining t1, t 2 and t 3 , t 4 pass through a point
and their point of intersection is (3a, 0 ).
(c, 0) on the axis, then
t1t 2 = t 3t 4 = - c / a. 15 A line ray parallel to axis of the parabola after reflection
passes through the focus.
JEE Type Solved Examples :
Single Option Correct Type Questions
n This section contains 10 multiple choice examples. l Ex. 3 Two parabolas have the same focus. If their
Each example has four choices (a), (b), (c) and (d) out of directrices are the X-axis and the Y -axis, respectively, then
which ONLY ONE is correct. the slope of their common chord is
4 3
l Ex. 1 A ray of light travels along a line y = 4 and strikes (a) ± 1 (b) (c) (d) None of these
3 4
the surface of curves y 2 = 4( x +y ), then the equation of the Sol. (a) Let the focus be (a, b )
line along which the reflected ray travels is Then, equations are ( x - a ) 2 + (y - b ) 2 = y 2 and
(a) x = 0 (b) x = 2 ( x - a ) 2 + (y - b ) 2 = x 2
(c) x + y = 4 (d) 2x + y = 4
If S1 º ( x - a ) 2 + (y - b ) 2 - y 2 = 0
Sol. (a) The given curve is (y - 2 ) 2 = 4( x + 1 )
and S 2 º ( x - a ) 2 + (y - b ) 2 - x 2 = 0
Y
\Equation of common chord S1 - S 2 = 0 gives
y=4 x2 -y 2 = 0 or y =± x
(0,4)
Hence, slope of common chord is ± 1.
(–1,2) (0,2)
l Ex. 4 Let us define a region R in xy-plane as a set of
points ( x , y ) satisfying [ x 2 ] = [y ] (where [ x ] denotes greatest
X
(0,0) integer £ x), then the region R defines
(a) a parabola whose axis is horizontal
The focus is ( 0, 2 ). (b) a parabola whose axis is vertical
The point of intersection of the curve and y = 4 is ( 0, 4 ). From
the reflection property of parabola the reflected ray passes (c) integer point of the parabola y = x 2
through the focus. (d) None of the above
Therefore, x = 0 is the reflected ray. Sol. (d)Q[ x 2 ] = [y ]
Y
If 0 £ y < 1,
l Ex. 2 A parabola is drawn with focus at (3, 4 ) and vertex
then, [y ] = 0 3
at the focus of the parabola y 2 - 12 x - 4y + 4 = 0. The \ [x 2 ] = 0
equation of the parabola is
0 £ x2 <1 2
(a) x 2 - 6 x - 8y + 25 = 0 (b) y 2 - 8 x - 6y + 25 = 0
Þ x Î( - 1, 1 )
(c) x 2 - 6 x + 8y - 25 = 0 (d) x 2 + 6 x - 8y - 25 = 0 for 1 £ y < 2,
1
2 2
Sol. (a) y - 12 x - 4y + 4 = 0 Þ (y - 2 ) = 12 x then [y ] = 1
S (3,4) X′ X
\ [x 2 ] = 1 –2 – √3 – √2 –1 O 1 √2 √3 2

Þ 1 £ x2 <2 Y′

Þ x Î( - 2, - 1 ] È [1, 2 )
for 2 £ y < 3, then [y ] = 2

A (3,2)
then, [ x 2 ] = 2 Þ2 £ x 2 < 3
Its vertex is ( 0, 2 ) and a = 3, \ x Î( - 3, - 2 ] È [ 2, 3 )
its focus = (3, 2 ). The graph of the region will not only contain of the parabola
Hence, for the required parabola; focus is (3, 4 ), vertex = (3, 2 ) y = x 2 but [ x 2 ] = [y ] contain points within the rectangles of side
and a = 2,
1, 2, ; 1, 2 - 1; 1, 3 - 2 etc.
Hence, the equation of the parabola is
( x - 3 ) 2 = 4(2 ) (y - 2 ) Hence, a, b, c are incorrects.

or x 2 - 6 x - 8y + 25 = 0
Chap 05 Parabola 409

l Ex. 5 The minimum area of circle which touches the 2m


\ m3 = -
l
parabolas y = x 2 +1 and x = y 2 + 1 is
æ - 2m ö 2 æ - 2m ö ö÷
(a)
9p
sq units (b)
9p
sq units \ D º ça æç ÷ , - 2a ç ÷
è è l ø è l øø
16 32
9p 9p æ 4am 2 4am ö
(c) sq units (d) sq units Þ Dºç 2 , ÷
8 4 è l l ø
Sol. (b) The parabolas y = x 2 + 1 and x = y 2 + 1 are symmetrical
about y = x. l Ex. 7 If d is the distance between the parallel tangents
Therefore, the tangent at point A is parallel to y = x
with positive slope to y 2 = 4 x and
dy
Y
then
dx
= 2x =1
y=x x 2 + y 2 - 2 x + 4y - 11 = 0, then
1 5 (a) 10 < d < 20 (b) 4 < d < 6
or x = ,y =
2 4 A (c) d < 4 (d) None of these
æ 1 5ö Sol. (c) Tangent to the parabola y 2 = 4 x having slope m is
\ Aºç , ÷ B
è2 4ø 1
X′ X y = mx +
O
and æ 5 1ö m
B ºç . ÷
è 4 2ø Y′ and tangent to the circle ( x - 1 ) 2 + (y + 2 ) 2 = 4 2 having slope m is
2 2
1 1 æ1 5 ö æ 5 1ö 3 2 y + 2 = m( x - 1 ) + 4 (1 + m 2 )
Hence, Radius (r ) = AB = ç - ÷ +ç - ÷ =
2 2 è2 4ø è 4 2ø 8 1
4 (1 + m 2 ) - m - 2 -
9p
2 m
\ Area = pr = sq units \Distance between tangents (d ) =
32 (1 + m 2 )

l Ex. 6 Let the line lx + my =1 cut the parabola y 2 = 4ax (1 + m 2 )


2
in the points A and B. Normals at A and B meet at point C. = 4- -
(1 + m 2 ) m
Normal from C other than these two meet the parabola at D,
then the coordinate of D are As m>0 (given)
æ 4am 4a ö we get d<4
(a) (a, 2a ) (b) ç 2 , ÷
è l l ø
æ 2am 2 2a ö æ 4am 2 4am ö
l Ex. 8 Two parabolas C and D intersect at two different
(c) ç 2 , ÷ (d) ç 2 , ÷ points, where C is y = x 2 - 3 and D is y = kx 2 . The
è l l ø è l l ø
intersection at which the x value is positive is designated
Sol. (d) Let A º (am12, - 2am1 ) and B º (am22, - 2am2 ) point A , and x = a at this intersection the tangent line l at A
Now, A and B lie on lx + my = 1 to the curve D intersects curve C at point B, other than A. If
Þ l (am12 ) + m( - 2am1 ) = 1 ...(i) x-value of point B is 1, then a is equal to
and l (am22 ) + m( - 2am2 ) = 1 ...(ii) (a) 1 (b) 2 (c) 3 (d) 4
Sol. (c) C : y = x 2 - 3 and D : y = kx 2
Subtracting Eq. (ii) from Eq. (i), then
la(m12 - m22 ) - 2am(m1 - m2 ) = 0 Solving C and D, then
kx 2 = x 2 - 3 ...(i)
Þ a(m1 - m2 ) ¹ 0
3
\ l (m1 + m2 ) - 2m = 0 or x2 = ,
2m 1 -k
Þ m1 + m2 = ...(iii)
l 3k
then, y=
Let D º (am32, - 2am3 ) and C º (h, k ) 1 -k

\ Equation of normal in terms of slope æ 3 3k ö


\ Aºç , ÷ (given x-value of A is positive)
y = Mx - 2aM - aM 3 è 1 -k 1 -k ø
3
then aM - (h - 2a ) M + k = 0 æ 3 ö
and a= ç ÷
\ m1 + m2 + m3 = 0 è 1 -k ø
2m then A º (a, ka 2 ) º (a, a 2 - 3 ) [from Eq. (i)]
Þ + m3 = 0
l
410 Textbook of Coordinate Geometry

tangent ‘l’ at A to the curve D is Hence, the corresponding point on the parabola is (1, 2 )
y + a2 -3 \ ( x1 - x 2 ) 2 + (3 + (1 - x12 ) - 4 x 2 ) 2 = ( x1 - x 2 ) 2 + (y1 - y 2 ) 2
= kx × a
2
= Distance between (1, 2 ) and ( 0, 3 ) – radius
æ 3ö
Þ y + a 2 - 3 = 2ax ç1 - 2 ÷ [from Eq. (i)] …(ii) = (1 + 1 ) - 1 = ( 2 - 1 )
è a ø
\ B º (1, - 2 ) (a ¹ 1) or ( x1 - x 2 ) 2 + (3 + (1 - x12 ) - 4 x 2 ) 2 = ( 2 - 1 ) 2
æ 3ö
From Eq. (ii), - 2 + a 2 - 3 = 2a ç1 - 2 ÷ \ min× [( x1 - x 2 ) 2 + (3 + (1 - x12 ) - 4 x 2 ) 2 ] = 3 - 2 2
è a ø
Þ a 3 - 5a = 2a 2 - 6 l Ex. 10 The condition that the parabolas y 2 = 4c ( x - d )
3 2
Þ a - 2a - 5a + 6 = 0
and y 2 = 4ax have a common normal other than X-axis
Þ (a - 1 ) (a + 2 ) (a - 3 ) = 0
(a > 0, c > 0 ) is
\ a =3 (Qa ¹ 1, a ¹ - 2)
(a) 2a < 2c + d (b) 2c < 2a + d
(c) 2d < 2a + c (d) 2d < 2c + a
l Ex. 9 min [( x 1 - x 2 ) 2 + (3 + (1 - x 12 ) - 4 x 2 ) 2 ], " Sol. (a) Normals of parabolas y 2 = 4ax and y 2 = 4c ( x - d ) in terms
x 1 , x 2 Î R, is of slope are
(a) 4 5 + 1 (b) 3 - 2 2 (c) 5 + 1 (d) 5 - 1 y = mx - 2am - am 3 …(i)
Sol. (b) Let y1 = 3 + (1 - x12 ) and y 2 = 4x 2 and y = m ( x - d ) - 2cm - cm 3
…(ii)
or x12 2
+ (y1 - 3 ) = 1 and y 22 = 4x 2 Subtracting Eqs. (ii) from (i), then
2 2
Thus, ( x1, y1 ) lies on the circle x + (y - 3 ) = 1 and ( x 2, y 2 ) lies md - 2am - am 3 + 2cm + cm 3 = 0
on the parabola y 2 = 4 x. m¹0
Thus, the given expression is the shortest distance between \ d - 2a - am 2 + 2c + cm 2 = 0
the curve x 2 + (y - 3 ) 2 = 1 and y 2 = 4 x. Þ (a - c )m 2 = d - 2a + 2c
Now, the shortest distance always occurs along the common d - 2a + 2c
normal to the curves and normal to the circle passes through Þ m2 =
(a - c )
the centre of the circle.
d
Normal to parabola y 2 = 4 x is y = mx - 2m - m 3. It passes Þ -2 > 0
a -c
through centre of circle ( 0, 3 ).
Therefore, 3 = - 2m - m 3 Þm 3 + 2m + 3 = 0 which has only one Þ d > 2a - 2c
real value m = - 1. Þ 2a < 2c + d

JEE Type Solved Examples :


More than One Correct Option Type Questions
n
This section contains 5 multiple choice examples. Each Sol. (a, b, c, d) Let P (at 2, 2 at ) be a point on the parabola y 2 = 4ax
example has four choices (a), (b), (c) and (d) out of which with focus S (a, 0 )
a
MORE THAN ONE may be correct. Now, mid-point of focal radii SP is M æç (t 2 + 1 ), at ö÷.
è2 ø
l Ex. 11 The locus of the mid-point of the focal radii of a a 2
Let x = (t + 1 ) and y = at
2
variable point moving on the parabola y 2 = 4ax is a
a æy 2 ö
parabola whose or x = ç 2 + 1÷
2 èa ø
(a) latusrectum is half the latusrectum of the original a
parabola Þ y 2 = 2ax - a 2 or y 2 = 2a æç x - ö÷
è 2ø
æa ö a
(b) vertex is ç , 0÷ which is a parabola with vertex æç , 0 ö÷ and latusrectum is 2a,
è2 ø è2 ø
a a a a
(c) directrix is Y -axis directrix is x - = i.e. x = 0 (Y ) and focus æç + , 0 ö÷ i.e. (a, 0 ).
2 2 è2 2 ø
(d) focus has the coordinates (a, 0)
Chap 05 Parabola 411

l Ex. 12 If P1P2 and Q 1Q 2 , two focal chords of a parabola Also, line (i) touches the circle x 2 + (y - 2 ) 2 = 4, then
2
y = 4ax at right angles, then | 2 + am 2 |
=2
(a) area of the quadrilateral P1Q 1P2Q 2 is minimum when (1 + m 2 )
the chords are inclined at an angle p / 4 to the axis of Þ 4 + a 2m 4 + 4am 2 = 4 + 4m 2
the parabola.
4 - 4a
(b) minimum area is twice the area of the square on the \ m2 = and m 2 = 0
latusrectum of the parabola. a2
(c) minimum area of quadrilateral P1Q 1P2Q 2 cannot be Put 4a = 1 for y = x 2, x ³ 0, then m 2 = 48
found. and put 4a = - 1 for y = - x 2, x < 0, then m 2 = 80
(d) minimum area is thrice the area of the square on the \Common tangents are
latusrectum of the parabola.
y = 0, y = 4 3 x - 12 and y = 4 5 x + 20
Sol. (a, b) Let coordinates of P1 are (at 2, 2 at ), then coordinates of P2
a 2a
are æç 2 , - ö÷. l Ex. 14 Let V be the vertex and L be the latusrectum of
èt t ø
Y P the parabola x 2 = 2y + 4 x - 4. Then, the equation of the
P1
parabola whose vertex is at V , latusrectum L / 2 and axis is
Q1 perpendicular to the axis of the given parabola.
(a) y 2 = x - 2 (b) y 2 = x - 4
α
X′ X (c) y 2 = 2 - x (d) y 2 = 4 - x
A S
Sol. (a, c) Given parabola is x 2 = 2y + 4 x - 4
P2
Þ ( x - 2 ) 2 = 2y
Q2
Y′ Vertex of the parabola is (2, 0) and length of latusrectum = 2
Let focal chord P1P2 makes an angle a with X -axis, then \ V (2, 0 ) and L = 2
æ 2a ö Y
2at - ç - ÷
è t ø 2
tan a = =
a 1
at 2 - 2 t- axis of
t t parabola
X′ X
1 (2,0)
Þ t - = 2 cot a … (i)
t
2
æ 1ö ìæ 1ö 2 ü Y′
Now, P1P2 = a çt + ÷ = aí çt - ÷ + 4ý
è tø è tø
î þ Length of latusrectum of required parabola = L / 2 = 1
= a { 4 cot 2 a + 4 } = 4a cosec 2a [from Eq. (i)] \Equation of the required parabola is (y - 0 ) 2 = ± 1( x - 2 )
Similarly, Q1Q2 = 4a cosec 2(90 - a ) = 4a sec 2a Þ y 2 = x - 2 or y 2 = - x + 2
1
\Area of quadrilateral P1Q1P2Q2 = ( P1P2 ) (Q1Q2 )
2 l Ex. 15 Consider a circle with its centre lying on the
= 8a sec a cosec a = 32a 2 cosec 2a
2 2 2
focus of the parabola y 2 = 2ax such that it touches the
\Minimum area = 32a 2 = 2 (latusrectum) 2 and is inclined at directrix of the parabola. Then a point of intersection of the
a=p /4 (Q cosec 2 a = 1) circle and the parabola is
æa ö æa ö
(a) ç , a ÷ (b) ç , - a ÷
l Ex. 13 The equation of the line that touches the curves è2 ø è2 ø
y = x | x | and x 2 + (y - 2 ) 2 = 4, where x ¹ 0 , is æ a ö
(c) ç - , a ÷
æ a ö
(d) ç - , - a ÷
(a) y = 4 5x + 20 (b) y = 4 3 x - 12 è 2 ø è 2 ø

(c) y = 0 (d) y = - 4 5x - 20 Sol. (a, b) Given parabola is y 2 = 2ax …(i)


a a
\ Focus and equation of directrix are S æç , 0 ö÷ and x = -
ì x 2, x ³ 0 è 2 ø 2
Sol. (a, b, c)Qy = x | x | = í 2 respectively.
î- x , x < 0 2
a
\Equation of circle is ç x - ö÷ + (y - 0 ) 2 = a 2
æ … (ii)
Q Equation of tangent in terms of slope (m ) of x 2 = 4ay is è 2ø
y = mx - am 2 … (i) (Q radius = distance between focus and directrix)
412 Textbook of Coordinate Geometry

From Eqs. (i) and (ii), From Eq. (i),


2
æ aö 3a 2 y 2 =a2
ç x - ÷ + 2ax = a 2 Þ x 2 + ax - =0
è 2ø 4 \ y =±a
a 3a 3a a
or x = ,x ¹- (Q for x = - , y = imaginary) Hence, required point of intersection are æç , ± a ö÷.
2 2 2 è2 ø

JEE Type Solved Examples :


Paragraph Based Questions
n This section contains 2 Solved Paragraphs based upon Therefore, 4 + 2 l = 0 or l = - 2
each of the paragraph 3 multiple choice question. Each Thus, the required circle is
of these questions has four choices (a), (b), (c) and (d) out of ( x - 1 ) 2 + (y - 2 ) 2 - 2( x - y + 1 ) = 0
which ONLY ONE is correct.
or x 2 + y 2 - 4 x - 2y + 3 = 0
Paragraph I Y 0
(Q. Nos. 16 to 18) 1=
y+
x–
Tangents are drawn to the parabola y 2 = 4 x at the point P (1,2) P
which is the upper end of latusrectum.
16. Image of the parabola y 2 = 4 x in the tangent line at X
A O
the point P is S (1, 0)
(a) ( x + 4 )2 = 16y (b) ( x + 2)2 = 8 (y - 2)
(c) ( x + 1)2 = 4 (y - 1) (d) ( x - 2)2 = 2(y - 2)

17. Radius of the circle touching the parabola y 2 = 4 x at Its radius is ( 4 + 1 - 3 ) = 2.


the point P and passing through its focus is 18. (a) Area bounded by AOPA = Area of DAOB + Area of OPBO
(a) 1 (b) 2 (c) 3 (d) 2 Y

18. Area enclosed by the tangent line at P, X-axis and the


M P(1, 2)
parabola is
2 4 B
(a) sq units (b) sq units
3 3
X
14 16 A O
(c) sq units (d) sq units
3 3
Sol. Upper end of latusrectum is P (1, 2 )
\The equation of tangent at P (1, 2 ) is
2
y × 2 = 2 (x + 1) Þ x - y + 1 = 0 1 2y 2 1 éy 3 ù 8 2
= ´1 ´1 + ò dy - ´ 1 ´ 1 = ê ú = = sq unit
16. (c) Any point on the given parabola is (t 2, 2t ). The image of 2 0 4 2 12
ë û0 12 3
(h, k ) of the point (t 2, 2t ) on x - y + 1 = 0 is given by
Paragraph II
h - t 2 k - 2 t - 2(t 2 - 2 t + 1 )
= = (Q. Nos. 19 to 21)
1 -1 1+1
Þ h = 2 t - 1 and k = t 2 + 1 Let C 1 and C 2 be respectively, the parabolas x 2 = y - 1 and
æh + 1ö
2
y 2 = x - 1. Let P be any point on C 1 and Q be any point on
or k =ç ÷ + 1 Þ (h + 1 ) 2 = 4(k - 1 )
è 2 ø C 2 . Let P1 and Q 1 be the reflections of P and Q , respectively
The required equation of image is ( x + 1 ) 2= 4(y - 1 ). with respect to the line y = x .
17. (c)Q Focus is S(1, 0) and P is (1, 2). 19. P1 and Q 1 lie on
Equation of circle touching the parabola at (1, 2) is (a) C1 and C 2 respectively (b) C 2 and C1 respectively
( x - 1 ) 2 + (y - 2 ) 2 + l( x - y + 1 ) = 0 it passes through (1, 0 ). (c) Cannot be determined (d) None of these
Chap 05 Parabola 413

20. If the point P( l, l2 + 1) and Q(m 2 + 1, m), then P1 and \ P1 and Q1 are ( l2 + 1, l)
Q 1 are and (m, m 2 + 1 ) (Ans. 20(b))
2 2 2 2
(a) ( l + 1, l) and (m + 1, m) (b) ( l + 1, l ) and (m, m + 1) 2
Also, P1 and Q1 lie on y = x - 1
(c) ( l, l2 + 1) and (m, m 2 + 1) (d) ( l, l2 +1) and (m 2 + 1, m)
and x 2 = y - 1.
21. Arithmetic mean of PP1 and QQ 1 is always Hence, P1 and Q1 lie on C 2 and C1 , respectively. (Ans. 19(b))
less than 21. (a)Q A is mid-point of PP1 and B is mid-point of QQ1.
1 3 1
(a) PQ (b) PQ (c) 2PQ (d) PQ \ PA = PP1
2 2 2
Sol. Since, the reflection of a point ( p, q ) with respect to line y = x is 1
and QB = QQ1 ...(i)
(q, p ). 2
Let P( l, l2 + 1 ) and Q(m 2 + 1, m) be points on C1 and C 2, Þ PC ³ PA ...(ii)
respectively. and QC ³ QB ...(iii)
\ Reflection of P( l, l2 + 1 ) with respect to line y = x is On adding Eqs. (ii) and (iii), then
P1( l2 + 1, l) and reflection of Q(m 2 + 1, m) with respect to line PC + QC ³ PA + QB
y = x is Q1(m, m 2 + 1 ). 1 1
= PP1 + QQ1 [from Eq. (i)]
x2 =y –1 Y
2 2
C1
Q1 æ PP + QQ1 ö
y=x =ç 1 ÷
è 2 ø
P B æ PP + QQ1 ö
(0, 1)
\ PC + QC ³ ç 1 ÷
C è 2 ø
C2
A Q Þ PQ ³ (AM of PP1 and QQ1 )
O P1
X
(1, 0)
y2=x–1

JEE Type Solved Examples :


Single Integer Answer Type Questions
n
This section contains 2 solved examples. The answer to at 2t 3 a(t 2 + t 3 ) 1 t t t +t 1
1 a2 2 3 2 3
each example is a single digit integer, ranging from 0 to Area of DPQR = | at 3t1 a(t 3 + t1 ) 1 | = | t 3t1 t 3 + t1 1 |
2 at t a(t + t ) 1 2 tt t +t 1
9 (both inclusive). 12 1 2 12 1 2

a2
l Ex. 22 Points A, B, C lie on the parabola y 2 = 4ax . The =
2
| St 2t 3(t 3 - t 2 ) |
tangents to the parabola at A, B, C taken in pairs intersect at
a2
points P , Q , R, then, the ratio of the areas of the DABC and = | (t1 - t 2 ) (t 2 - t 3 ) (t 3 - t1 ) |
2
DPQR is
1
Sol. (2) Let (at12, 2at1 ), (at 22, 2at 2 ) and (at 32, 2at 3 ) be the points A, B = ´ Area of DABC
2
and C respectively. Area of DABC
\ =2
at12 2at1 1 1 t1 t12 Area of DPQR
1
\Area of DABC = at 2 2at 2 1 | = a | 1 t 2 t 22
2 2
2
at 32 2at 3 1 1 t 3 t 32 l Ex. 23 If the orthocentre of the triangle formed by the
= a 2 | (t1 - t 2 ) (t 2 - t 3 ) (t 3 - t1 ) | points t 1 , t 2 , t 3 on the parabola y 2 = 4ax is the focus, the
Coordinates of P , Q and R are (at 2t 3, a(t 2 + t 3 )), (at 3t1, a(t 3 + t1 )), value of | t 1t 2 + t 2 t 3 + t 3 t 1 | is
(at1t 2, a(t1 + t 2 )) respectively, then
414 Textbook of Coordinate Geometry

Sol. (5) Q SA is perpendicular to BC[S is focus (a, 0)] A(at12, 2at1)


F E
æ 2at1 - 0 ö æ 2at 3 - 2at 2 ö
Þ ç 2 ÷ç ÷ = -1 S
è at1 - a ø è at 32 - at 22 ø (a,0)
æ 2t1 ö æ 2 ö B C
Þ ç 2 ÷ç ÷ = -1 D
è t1 - 1 ø è t 3 + t 2 ø (at22, 2at2) (at32, 2at3)

Þ 4t1 + t12(t 2 + t 3 ) = t 2 + t 3 …(i) On subtracting Eq. (ii) from Eq. (i), then
4 + (t1t 2 + t 2t 3 + t 3t1 ) = - 1 Þ t1t 2 + t 2t 3 + t 3t1 = - 5
Similarly, 4t 2 + t 22(t 3 + t1 ) = t 3 + t1 …(ii)
\ | t1t 2 + t 2t 3 + t 3t1 | = 5

JEE Type Solved Examples :


Matching Type Questions
n This section contains one solved example. Example 24 (B) Equation of tangent to parabola y 2 = 4 x at P (t 2, 2 t ) is
has three statements (A, B and C) given in Column I and ty = x + t 2
five statements (p, q, r, s and t) in Column II. Any given
Given t 2 Î[1, 4 ]
statement in Column I can have correct matching with
one or more statement(s) given in Column II. 1
\Area of DAPN = ( AN ) ( PN )
2
l Ex. 24 Match the following. Y
)
(t2 , 2t
Column I Column II P

(A) If PQ is any focal chord of the parabola y2 = 32x (p) 2 X′ X


and length of PQ can never be less than l units, A (–t2,0) O N
then l is divisible by
(B) A tangent is drawn to the parabola y2 = 4 x at the (q) 3 Y′
point ‘P’ whose abscissa lies in the interval [1, 4 ] .
1
If maximum possible area of the triangle formed = (2 t 2 ) (2 t )
by the tangent at ‘P’, ordinate of the point ‘P’ and 2
the X-axis is l sq units, then l is divisible by 3
Þ D = 2t 3 = 2(t 2 ) 2
(C) The normal at the ends of the latusrectum of the (r) 4
parabola y2 = 4 x meet the parabola again at A and Q t 2 Î[1, 4 ]
A¢. If length AA¢ = l unit, then l is divisible by Þ D max occurs, when t 2 = 4
(s) 6 3
Þ D max = 2( 4 ) 2 = 16 sq units
(t) 8
\ l = 16
Sol. (A) ® (p, r, t); (B) ® (p, r, t); (C) ® (p, q, r, s)
(C) Given parabola y 2 = 4 x. Now, the ends of latusrectum are
(A) Let P (at 2, 2at ) be any point on the parabola y 2 = 4ax and P(1, 2 ) and Q(1, - 2 ) or P (t 2, 2t ) and Q (t12, 2t1 ), where t = 1, t1 = - 1.
S (a, 0 ) be the focus, then the other end of focal chord through 2
æ a 2a We know that the other end of normal is given by t 2 = - t -
P will be Q ç 2 , - ö÷. t
èt t ø
2
Þ A(t 22, 2t 2 ) and A ¢(t 32, 2t 3 ),
1
Then, length of focal chord PQ = a æçt + ö÷ where t 2 = - 3, t 3 = 3
è tø
2 or A(9, - 6 ) and A¢(9, 6 )
1 æ 1ö
Q t+ ³ 2 Þ a çt + ÷ ³ 4a \ AA¢ = 12 units
t è tø
\ l = 12
or PQ ³ 32 [Q 4a = 32]
\ l = 32
JEE Type Solved Examples :
Statement I and II Type Questions
n Directions (Ex. Nos. 25 and 26) are Assertion-Reason On differentiating w.r.t. t, we get
type questions. Each of these question contains two 3t 2 + (2 - l ) > 0, for l < 2
statements :
Statement I (Assertion) and The cubic Eq. (i) has only one real root.
Statement II (Reason) \ Statement I is true.
Each of these examples also has four alternative choices, Hence, both statements are true but Statement II is not correct
only one of which is the correct answer. explanation for Statement I.
You have to select the correct choice
(a) Statement I is true, Statement II is true; Statement II is a l Ex. 26 Statement I If there exist points on the circle
correct explanation for statement I.
x + y 2 = l2 from which two perpendicular tangents can be
2
(b) Statement I is true, Statement II is true; Statement II is not a
1
correct explanation for Statement I. drawn to the parabola y 2 = 2 x , then l ³ .
(c) Statement I is true, Statement II is false. 2
(d) Statement I is false, Statement II is true. Statement II Perpendicular tangents to the parabola meet
at the directrix.
l Ex. 25 Statement I Through the point ( l, l +1), l < 2, Sol. (a) Statement II is true as it is property of parabola. Equation
1
there cannot be more than one normal to the parabola y 2 = 4 x . of directrix of parabola y 2 = 2 x is x = - .
2
Statement II The point ( l, l +1) cannot lie inside the æ 1 ö
Any point on directrix is ç - , y ÷ , now this point exists on the
parabola y 2 = 4 x . è 2 ø
Sol. (b) Let S = y 2 - 4 x circle, then
1
\ S1 = ( l + 1 ) 2 - 4 l = ( l - 1 ) 2 ³ 0 + y 2 = l2
4
\ Point ( l, l + 1 ) cannot lie inside the parabola y 2 = 4 x.
1
Þ y 2 = l2 - ³ 0
\ Statement II is true. 4
Now, equation of normal at (t 2, 2 t ) is y + tx = 2t + t 3 passes 1
\ l³
through ( l, l + 1 ). 2
Þ l + 1 + tl = 2t + t 3 Hence, both statements are true and Statement II is correct
explanation for Statement I.
Þ t 3 + (2 - l )t - ( l + 1 ) = 0 …(i)

Subjective Type Examples


n
In this section, there are 16 subjective solved On comparing the coefficients of x and y in Eq. (i) and (ii),
examples. 2a k
we get =
h 2a
l Ex. 27 The two parabolas y 2 = 4a ( x - l ) and Þ hk = 4a 2
x 2 = 4a (y - l1 ) always touch one another, l and l1 being Hence, the locus is xy = 4a 2,
variable parameters. Prove that the point of contact lies on which is independent of l and l1.
2
the curve xy = 4a .
l Ex. 28 Show that the area formed by the normals to
Sol. Let (h, k ) be the common point and if the parabolas touch 2
y = 4ax at the points t 1 , t 2 , t 3 is
each other, then the tangents at (h, k ) should be identical.
Their equations are 1 2
a |(t 1 - t 2 ) (t 2 - t 3 )(t 3 - t 1 )| l2 , where l = (t 1 + t 2 + t 3 ).
ky = 2a( x - l + h ) 2
Þ 2ax - ky = 2a(l - h ) …(i) Sol. Equation of normals at P , Q, R are
and hx = 2a(y - l1 + k ) y = - t1x + 2at1 + at13
Þ hx - 2ay = 2a(k - l1 ) …(ii)
416 Textbook of Coordinate Geometry

y = - t 2x + 2at 2 + at 23, y = - t 3x + 2at 3 + at 33, respectively. l Ex. 29 Prove that the two parabolas y 2 = 4ax and
2
(at1 , 2at1) y 2 = 4c ( x - b ) cannot have common normal, other than the
P
N axis unless b / (a - c ) > 2.
L
Sol. Given parabolas y 2 = 4ax and y 2 = 4c( x - b ) have common
M
Q normals. Then equation of normals in terms of slopes are
2
(at2 , 2at2) y = mx - 2am - am 3
R (at 2,2 at )
3 3 and y = m( x - b ) - 2cm - cm 3
Subtracting them two by two, coordinates of L, M and N are respectively then normals must be identical, compare the
L º {2a + a(t12 + t1t 2 + t 22 ), - at1t 2(t1 + t 2 )} coefficients
M º {2a + a(t12 + t1t 3 + t 32 ), - at1t 3(t1 + t 3 )} 2am + am 3
1=
N º {2a + a(t 22 + t 2t 3 + t 32 ), - at 2t 3(t 2 + t 3 )} mb + 2cm + cm 3

\ Area of DLMN Þ m [(c - a )m 2 + (b + 2c - 2a )] = 0,


½2a + a(t12 + t1t 2 + t 22 ) -at1t 2(t1 + t 2 ) 1½ m¹0 [Qother than axis]

= | 2a + a(t12 + t1t 3 + t 32 ) -at1t 3(t1 + t 3 ) 1½| 2a - 2c - b
2½ 2 2 -at 2t 3(t 2 + t 3 ) 1½ and m2 = ,
½2a + a(t 2 + t 2t 3 + t 3 ) ½ c -a
On applying R2 ® R2 - R1 and R3 ® R3 - R1
2(a - c ) - b
½ 2 + (t12 + t1t 2 + t 22 )
2 -t1t 2(t1 + t 2 ) 1½ m=±
a c -a
= |½ (t 3 - t 2 )(t1 + t 2 + t 3 ) t1(t 3 - t 2 )(t1 + t 2 + t 3 ) 0½|
½
2 (t - t )(t + t + t ) t (t - t )(t + t + t ) 0½
½ 3 1 1 2 3 2 3 1 1 2 3 ½ æ b ö
Þ m = ± ç-2 - ÷
Expanding by last column è c -a ø
a2 1 t b
= |(t 3 - t1 )(t 3 - t 2 )|(t1 + t 2 + t 3 ) 2 |½ 1½| \ -2 - >0
2 ½1 t 2½ c -a
1 2
= a |(t1 - t 2 )(t 2 - t 3 )(t 3 - t1 )|(t1 + t 2 + t 3 ) 2 b
2 Þ -2 + >0
1 a -c
= a 2 |(t1 - t 2 )(t 2 - t 3 )(t 3 - t1 )| l2 b
2 Þ >2
a -c
Aliter
Equations of sides of DLMN formed by the normals are
y + t1x - (2at1 + at13 ) = 0
l Ex. 30 If on a given base BC, a triangle is described such
that the sum of the tangents of the base angles is m, then
y + t 2x - (2at 2 + at 23 ) = 0
prove that the locus of the opposite vertex A is a parabola.
y + t 3x - (2at 3 + at 33 ) = 0
Sol. Let the given point B and C be (0, 0) and (a, 0 ), respectively.
2
½t1 1 -(2at1 + at13 ) ½ Given, tan a + tan b = m (constant)
1 ½t 1 -(2at + at 3 )½
\ Required area = 2 2 2
2 | C1C 2C 3 | ½t 1 ½ Y
½3 -(2at 3 + at 33 )½
2 A(x,y)
½t1 1 2 t1 + t13 ½
a2 ½t 1 2 t + t 3½
= 2 2 2
| 2(t 2 - t 3 )(t 3 - t1 )(t1 - t 2 )| ½t 1 3½
½ 3 2 t 3 + t 3½
2
ì t 1 2 t ½t 1 t13½ü α β
X
a2 ï½ 1 1½ 1
3 ï
X′
B (0,0)
í½t 2 1 2 t 2½+½t 2 1 t 2 ½ý
= D C (a,0)
2|(t 2 - t 3 )(t 3 - t1 )(t1 - t 2 )| ï t 1 2 t ½t 1 3½ï Y′
½ 3½ t3 ½
î 3 ½3 þ y y
a2 Þ + =m
= x a -x
2|(t 2 - t 3 )(t 3 - t1 )(t1 - t 2 )|
ay
{ 0 - (t1 - t 2 )(t 2 - t 3 )(t 3 - t1 )(t1 + t 2 + t 3 )} 2 Þ =m
1 x(a - x )
= a 2 |(t1 - t 2 )(t 2 - t 3 )(t 3 - t1 )|(t1 + t 2 + t 3 ) 2
2 \ ay = amx - mx 2
1 2
= a |(t1 - t 2 )(t 2 - t 3 )(t 3 - t1 )| l2. which is the equation of parabola.
2
Chap 05 Parabola 417

l Ex. 31 A parabolic mirror is kept along y 2 = 4 x and two l Ex. 33 A parabola of latusrectum 4a , touches a fixed
light rays parallel to its axis are reflected along one straight equal parabola , the axes of the two curves being parallel;
line. If one of the incident rays is at 3 units distance from the prove that the locus of the vertex of the moving curve is a
axis, then find the distance of the other incident ray from the parabola of latusrectum 8a.
axis. Sol. Let the given parabola is
Sol. Let the incident rays be PA and QB. y 2 = 4ax …(i)
After reflection, both rays passes through the focus S(1, 0 ). If the vertex of moving parabola is ( a, b ), then equation of
Y A moving parabola is
P
(y - b ) 2 = - 4a ( x - a ) …(ii)
2
y
X′ X On substituting the value of i.e. x = in Eq. (ii),
O S(1,0) 4a
Q

Y′
(0,0)
Therefore, AB is a focal chord.
1 -2ö
Let A be (t 2, 2t ), then B be æç 2 , ÷. (α, β)
èt t ø
Given, 2t = 3 \ t = 3 / 2
2 4
Hence, distance of B from the axis of parabola is - = units.
t 3 æy 2 ö
then (y - b ) 2 = -4a ç - a ÷
è 4a ø
l Ex. 32 Prove that the length of the intercept on the Þ y 2 - 2 by + b 2 = - y 2 + 4aa
normal at the point P (at 2 , 2at ) of a parabola y 2 = 4ax made
2y 2 - 2 by + b 2 - 4aa = 0 …(iii)
by the circle on the line joining the focus and point P as
Since, two parabolas (i) and (ii) touch each other.
diameter is a (1 + t 2 ) . Hence, roots of Eq. (iii) are equal i.e. Discriminant = 0
Sol. Let the normal at P (at 2, 2at ) cut the circle in K and the axis Þ “B 2 - 4 AC = 0”
of parabola at G then PK is required intercept.
\ ( -2 b ) 2 = 4 × 2 × (b 2 - 4aa )
SP = PM = a + at 2
) Þ 4b 2 = 32aa
Y 2, 2at
t
P (a or b 2 = 8aa
M θ
φ θ the required locus is y 2 = 8ax
90° K
X which has latusrectum double that of given parabola.
T A S (a,0) G
φ
l Ex. 34 The normal at point P on a given parabola meet
the axis of parabola at Q. Then prove that a line through Q
Directrix (x+a=0) and perpendicular to this normal always touches a fixed
Since angle in a semi-circle being right angle. parabola whose length of latusrectum is same as that of
\ Ð SKP = 90° given parabola.
Sol. Let the equation of parabola is
and normal at P (at 2, 2at ) is
y 2 = 4ax. …(i)
y = - tx + 2at + at 3
2
Let P (at , 2at )
Þ tx + y - 2at - at 3 = 0 …(i)
Normal at P is y + tx = 2at + at 3
\ SK is the perpendicular distance from S (a, 0 ) to the normal (i),
Q Normal meet the axis of parabola (i.e. X-axis), then
| at + 0 - 2at - at 3|
then SK = =a |t | 1 + t2 Q(2a + at 2, 0 ).
t2 + 1
Now, equation of the line through Q and perpendicular to the
\ In DSPK , ( Pk ) 2 = (SP ) 2 - (SK ) 2 normal is
1
= a 2(1 + t 2 ) 2 - a 2t 2 (1 + t 2 ) = a 2(1 + t 2 ) y - 0 = ( x - 2a - at 2 )
t
\ PK = a (1 + t 2 )
418 Textbook of Coordinate Geometry

Þ ty = x - 2a - at 2 Points P and Q lie on y 2 = 4ax then


Þ ty = ( x - 2a ) - at 2 ( KP ) 2 sin 2 q = 4a(d + KP cos q) …(i)
2 2
which is clearly tangent to the parabola and ( KQ ) sin q = 4a(d - KQ cos q) …(ii)
y 2 = 4a (2a - x ). 2 2 2
4a cos q + (16a cos q + 16ad sin q)
From Eq. (i), KP =
2 sin 2 q
l Ex. 35 TP and TQ are any two tangents to a parabola
-4a cos q + (16a 2 cos2 q + 16ad sin 2 q)
and the tangent at a third point R cuts them in P ¢ and Q ¢, From Eq. (ii), KQ =
TP ¢ TQ ¢ 2 sin 2 q
prove that + =1. 1 1 2a cos2 q + d sin 2 q
TP TQ \ 2
+ 2
=
( KP ) ( KQ ) 2ad 2
Sol. Let parabola be y 2 = 4ax and coordinates of P and Q on this
parabola are P º (at12, 2at1 ) and Q º (at 22, 2at 2 ) ; T is the point of For d = 2a
intersection of tangents at t1 and t 2. 1 1 1
\ + =
( KP ) 2 ( KQ ) 2 d 2
) 1
P (t 1 = 2 , which is independent of q.
4a
P′

T A
l Ex. 37 If the distribution of weight is uniform, then the
R (t3)
rope of the suspended bridge takes the form of parabola. The
height of the supporting towers is 20 m, the distance between
Q ′ Q (t2) these towers is 150 m and the height of the lowest point of
the rope from the road is 3 m. Find the equation of the
\ Coordinates of T º {at1, t 2, a (t1 + t 2 )}
parabolic shape of the rope considering the floor of the
Similarly P ¢ º {at 3t1, a (t 3 + t1 )}
bridge as X-axis and the axis of the parabola as Y-axis. Find
Q ¢ º {at 2t 3, a (t 2 + t 3 )} the height of that tower which supports the rope and is at a
Let TP ¢ : TP = l :1 distance of 30 m from the centre of the road .
t -t TP ¢ t 3 - t 2
\ l= 3 2 Þ = Sol. Here MZ is the road let V is lowest point of the rope given
t1 - t 2 TP t1 - t 2 VZ = 3 m.
TQ ¢ t1 - t 3 S
Similarly, = 150 m
TQ t1 - t 2 P (75, 20)
TP ¢ TQ ¢
\ + =1
TP TQ
20 m
l Ex. 36 Prove that on the axis of any parabola there is a
certain point ‘K’ which has the property that, if a chord PQ
3 V
1 1
of parabola be drawn through it, then + is the M Road Z
2
PK QK 2
Let MZ is X-axis and let MVS is Y-axis.
same for all positions of the chord . Taking X and Y -axes as shown in the given figure, the equation
x -d y - 0 of parabola is of the form
Sol. Any line passing through K is = =r
cos q sin q
x 2 = 4ay
\ Coordinates of P and Q are
if V is origin
(d + KP cos q, KP sin q)
1
and (d - KQ cos q, - KQ sin q). Coordinates of P is ìí (150 ), 20 - 3üý i.e. (75 , 17 ) P lies on
î2 þ
Y
P Eq. (i), then
(75 ) 2 = 4a (17 )
θ 5625
X′ X or 4a =
A K (d, 0) 17
2 5625
Q From Eq. (i), x = y
17
Y′
Chap 05 Parabola 419

Now, shifting the origin (0, 0) to M ( 0, - 3 ), l Ex. 39 A variable chord PQ of the parabola y = 4 x 2
then equation of parabola in new form is
subtends a right angle at the vertex. Find the locus of the
5625
x2 = (y - 3 ) …(ii) points of intersection of the normals at P and Q.
17
Sol. Parametric point on the parabola
Again, let the required height of supporting tower at a distance
of 30 m from the centre of road be h metres, then the coordinates y = 4 x 2 is (t, 4t 2 )
of the top of this tower are (30, h ) referred to given axes Let P (t1, 4t12 ) and Q (t 2, 4t 22 ),
5625
from Eq. (ii), (30 ) 2 = (h - 3 ) if A is vertex of the parabola, then
17
Slope of AP ´ Slope of AQ = - 1 [QAP^ AQ ]
900 ´ 17 ö
\ h = æç ÷ + 3 = 5.72 m \ 4t1 ´ 4t 2 = - 1
è 5625 ø
1
\ t1t 2 = -
l Ex. 38 Tangent is drawn at any point ( x 1 , y 1 ) on the 16
Equations of normals at P and Q are
parabola y 2 = 4ax . Now tangents are drawn from any point
x + 4t1y = t1 + 16t13 …(i)
on this tangent to the circle x 2 + y 2 = a 2 such that all the
and x + 4t 2y = t 2 + 16t 23 …(ii)
chords of contact pass through a fixed point ( x 2 , y 2 ) . Prove
2 Let the normals (i) and (ii) intersect at (h, k ), then
æ x1 ö æ y 1 ö on solving Eqs. (i) and (ii), then we get
that 4 ç ÷ + ç ÷ = 0.
è x2 ø èy 2 ø 1 + 16 (t12 + t 22 + t1t 2 )
k=
Sol. Tangent at ( x1, y1 ) on the parabola y 2 = 4ax 4
\ yy1 = 2a ( x + x1 ) 1 + 16 (t12 + t 22 ) + 16t1t 2
Þ k=
æ 2a ( x ¢ + x1 ) ö 4
Any point on this tangent will be ç x ¢, ÷ 1 + 16 ( (t1 + t 2 ) 2 - 2t1t 2 ) - 1
è y1 ø = [Q16t1t 2 = - 1 ]
4
æ 2a ( x ¢ + x1 ) ö
Equation of chord of contact of the point ç x ¢, ÷ w.r.t. ì 1ü
è y1 ø = 4 í(t1 + t 2 ) 2 + ý
î 8þ
2a ( x ¢ + x )
circle x 2 + y 2 = a 2 is xx ¢ + y × 1
=a2 1
y1 k = 4 (t1 + t 2 ) 2 + …(iii)
2
Þ xx ¢y1 + 2ayx ¢ + 2ayx1 = a 2y1 Again h = - 16t1t 2(t1 + t 2 )
Þ (2ayx1 - a 2y1 ) + x ¢ ( xy1 + 2ay ) = 0 Þ h = t1 + t 2 …(iv)
which is family of straight lines passing through point of On eliminating t1 and t 2 from Eqs. (iii) and (iv), we get
intersection of 1
k = 4h 2 +
2ayx1 - a 2y1 = 0 and xy1 + 2ay = 0 2
ay1 Þ 2k = 8h 2 + 1
\ y=
2 x1 Hence, locus of (h, k ) is 2y = 8 x 2 + 1.
2ay æ 2a ö æ ay1 ö
and x =- = ç- ÷ ç ÷
y1 è y1 ø è 2 x1 ø l Ex. 40 Equilateral triangles are circumscribed to the
a2 parabola y 2 = 4ax . Prove that their angular points lie on the
x =-
x1 conic (3 x + a ) ( x + 3a ) = y 2 .
The fixed point is ( x 2, y 2 ). Sol. Let coordinates of P and Q are (at12, 2at1 ) and (at 22, 2at 2 ),
2
a ay1 respectively.
\ x2 = - and y 2 =
x1 2 x1
2 2
æ x ö æy ö æ x ö æ y1 ö P
\ 4ç 1÷ + ç 1÷ = 4ç 21 ÷ + ç ÷
è x2 ø è y 2 ø è -a / x1 ø è ay1 / 2 x1 ø
4 x12 4 x12 A 60°
=- + 2 =0
a2 a
2 Q
æ x ö æy ö
Hence, 4 ç 1 ÷ + ç 1 ÷ = 0.
è x2 ø è y 2 ø
420 Textbook of Coordinate Geometry

Tangents at P and Q are t1y = x + at12 and t 2y = x + at 22, This is taken as directrix, then
respectively. SP = PM
1 1
Slope of tangents at P and Q are and , respectively. Þ BS = BM Þ(h - a ) 2 + (k - 0 ) 2 = (a cos q - b ) 2 …(iv)
t1 t2
and AS = AN
Let one angular point of equilateral triangle is A. Þ (h + a ) 2 + (k - 0 ) 2 = (a cos q + b ) 2 …(v)
\ A º (at1t 2, a(t1 + t 2 ))
On adding Eqs. (iv) and (v), then
1 1
- a 2 cos2 q + b 2 = h 2 + k 2 + a 2 …(vi)
t1 t 2 t -t
Given, tan60° = = 2 1 On subtracting Eq. (iv) from Eq. (v), then
1 1 1 + t1t 2
1+ × h
t1 t 2 4ab cos q = 4ah or cos q = …(vii)
2 2 2 b
Þ ( 3 ) (1 + t1t 2 ) = (t 2 - t1 )
From Eqs. (vi) and (vii), we get
Þ 3 (1 + t1t 2 ) 2 = (t1 + t 2 ) 2 - 4t1t 2 a 2h 2
+ b 2 = h2 + k 2 + a 2
For locus of A, put at1t 2 = x and a(t1 + t 2 ) = y , then b2
2 2
x y x
3 æç1 + ö÷ = æç ö÷ - 4 Þ h2
(b 2 - a 2 )
+ k2 =b2 -a2
è a ø èa ø a a2
Þ y 2 = 3 x 2 + 10ax + 3a 2 h2 k2
Þ 2
+ 2 =1
\ (3 x + a ) ( x + 3a ) = y 2
a b -a2
Hence, the locus of S (h, k ) is
l Ex. 41 A parabola is drawn to pass through A and B, the x2 y2
+ = 1.
ends of diameter of a given circle of radius a and to have a a2 b2 -a2
directrix a tangent to a concentric circle of radius b, the axes
being AB and the perpendicular diameter. Prove that the l Ex. 42 Two straight lines are at right angles to one
x2 y2 another and one of them touches y 2 = 4a ( x + a ) and the
locus of the focus of the parabola is + = 1.
b2 b2 -a2 other y 2 = 4b( x + b ) . Prove that the point of intersection of
Sol. Let AB be taken along X -axis, it mid-point O as origin and a the lines lies on the line x + a + b = 0.
line through O perpendicular to AB as Y -axis.
Sol. We know that any tangent in terms of slope (m) of the
Y a
parabola y 2 = 4ax is y = mx + .
m
Replacing x by x + a, we get
a
y = m( x + a ) + which is tangent to
m
y 2 = 4a( x + a ) …(i)
X′ A O X 2
Similarly, tangent in terms of slope of y = 4b( x + b ) is
B
s b
N

y = m1( x + b ) + …(ii)
m1
Given tangents are perpendicular, we have
M

1
Y′
mm1 = - 1 or m1 = -
m
(x + b )
Equation of circle on AB as diameter is then Eq. (ii) becomes y = - - bm
m
x2 + y 2 =a2 …(i) On subtracting Eqs. (i) and (ii), then
The equation of a circle of radius b and concentric to circle (i) is æ 1ö
0 = (x + a + b ) çm + ÷
x2 + y 2 =b2 …(ii) è mø
Let S (h, k ) be the focus of the parabola which passes through 1
Þ m+ ¹0
A( - a, 0 ) and B(a, 0 ). m
Let equation of tangent to circle (ii) be Hence, x +a +b =0
x cos q + y sin q = 6. …(iii)
#L Parabola Exercise 1 :
Single Option Correct Type Questions
n This section contains 30 multiple choice questions. 8. If a ¹ 0 and the line 2bx + 3cy + 4d = 0 passes through
Each question has four choices (a), (b), (c) and (d) out of the points of intersection of the parabolas y 2 = 4ax
which ONLY ONE is correct and x 2 = 4ay , then
1. A common tangent is drawn to the circle (a) d 2 + ( 2b + 3c )2 = 0 (b) d 2 + (3b + 2c )2 = a 2
x 2 + y 2 = a 2 and the parabola y 2 = 4bx . If the angle 2 2
(c) d + ( 2b - 3c ) = 0 (d) d 2 + ( 2b + 3c )2 = a 2
p
which this tangent makes with the axis of x is , then
the relationship between a and b is (a , b > 0 ) 4 9. A parabola y = ax 2 + bx + c crosses the X-axis at (a, 0 )
and (b, 0 ) both to the right of the origin. A circle also
(a) b = 2a (b) a = b 2 (c) c = 2a (d) a = 2c
passes through these two points. The length of a
2. The equation of parabola whose vertex and focus lie tangent from the origin to the circle is
on the axis of x at distances a and a 1 from the origin bc b c
respectively, is (a) (b) ac 2 (c) (d)
a a a
(a) y 2 = 4 (a1 - a ) x (b) y 2 = 4 (a1 - a ) ( x - a )
(c) y = 4 (a1 - a ) ( x - a1 ) (d) y 2 = 4aa1 x
2 10. Two mutually perpendicular tangent of the parabola
y 2 = 4ax meet the axis at P1 and P2 . It S is the focus of
3. If parabolas y 2 = ax and
1 1
25 [( x - 3 ) 2 + (y + 2 ) 2 ] = (3 x - 4y - 2 ) 2 are equal, the parabola, then + is equal to
SP1 SP2
then the value of a is
(a) 3 (b) 6 (c) 7 (d) 9 1 1 2 4
(a) (b) (c) (d)
4a a a a
4. ABCD and EFGC are squares and the curve y = l x
passes through the origin D and the points B and F. 11. If the normal to the parabola y 2 = 4ax at P meets the
FG curve again at Q and if PQ and the normal at Q make
The ratio is angles a and b respectively with the X-axis, then
BC
Y tan a (tan a + tan b) has the value equal to
1
(a) - 2 (b) - 1 (c) - (d) 0
E F 2
A B 12. If the normals to the parabola y 2 = 4ax at three
points P , Q and R meet at A and S is the focus, then
X SP × SQ × SR is equal to
D C G
(a) (SA ) 2 (b) (SA ) 3 (c) a (SA ) 2 (d) a (SA ) 3
3 +1 3 +1 5 +1 5 +1
(a) (b) (c) (d) 13. The length of the shortest normal chord of the
4 2 4 2
parabola y 2 = 4ax is
5. Let A and B be two points on a parabola y 2 = x with
(a) 2a 27 (b) 9a (c) a 54 (d) 18a
vertex V such that VA is perpendicular to VB and q is
the angle between the chordVA and the axis of the 14. The largest value of a for which the circle x 2 + y 2 = a 2
| VA | falls totally in the interior of the parabola
parabola. The value of is
| VB | y 2 = 4 ( x + 4 ) is
(a) tan q (b) cot 2 q (c) tan 3 q (d) cot 3 q 4 6
(a) 4 3 (b) 4 (c) (d) 2 3
6. The vertex of the parabola whose parametric 7
equation is x = t 2 - t + 1, y = t 2 + t + 1, t Î R, is 15. From a point (sin q, cos q ), if three normals can be
1 1 drawn to the parabola y 2 = 4ax , then the value of a is
(a) (1, 1) (b) (2, 2) (c) (3, 3) (d) æç , ö÷ 1 1
è 2 2ø (a) æç , 1ö÷ (b) é - , 0ö÷
è2 ø êë 2 ø
7. The circle x 2 + y 2 + 2 px = 0, p Î R, touches the 1 -1 1
parabola y 2 = 4 x externally, then (c) é , 1ù (d) æç , 0ö÷ È æç0, ö÷
êë 2 úû è 2 ø è 2ø
(a) p > 0 (b) p < 0 (c)p > 1 (d) p > 2
422 Textbook of Coordinate Geometry

16. If two different tangents of y 2 = 4 x are the normals 24. Consider the parabola y 2 = 4 x . Let A º ( 4, - 4 ) and
to x 2 = 4 by, then B º ( 9, 6 ) be two fixed points on the parabola. Let C be
1 1 a moving point on the parabola between A and B
(a) | b | < (b) | b | <
2 2 2 such that the area of the triangle ABC is maximum.
1 1 Then the coordinates of C are
(c) | b | > (d) | b | >
1
2 2 2 (a) æç , 1ö÷ (b) (3, - 2 3 )
è4 ø
17. The shortest distance between the parabolas
(c) (3, 2 3 ) (d) ( 4, 4 )
2y 2 = 2 x - 1 and 2 x 2 = 2y - 1 is
(a)
1
(b)
1
(c) 2 2 (d) 4 25. Through the vertex O of the parabola y 2 = 4ax , two
2 2 2 chords OP and OQ are drawn and the circles on OP
18. Normals at two points ( x 1 , y 1 ) and ( x 2 , y 2 ) of the and OQ as diameters intersect at R. If q 1 , q 2 and f are
the angles made with the axis by the tangents at P
parabola y 2 = 4 x meet again on the parabola, where
and Q on the parabola and by OR, then the value of
x 1 + x 2 = 4, then | y 1 + y 2 | is equal to cot q 1 + cot q 2 is
(a) 2 (b) 2 2 (c)4 2 (d) 8 2 (a) - 2 tan f (b) 2 tan f
(c) 0 (d) 2 cot f
19. A line is drawn from A ( - 2, 0 ) to intersect the curve
y 2 = 4 x at P and Q in the first quadrant such that 26. AB is a double ordinate of the parabola y 2 = 4ax .
1 1 1 Tangents drawn to the parabola at A and B meet the
+ < . Then the slope of the line is always Y -axis at A1 and B1 respectively. If the area of
AP AQ 4
1 1 trapezium AA1B1B is equal to 24a 2 , then the angle
(a) < (b) > (c) > 2 (d) > 3 subtended by A1B1 at the focus of the parabola is
3 3
equal to
20. An equilateral triangle SAB is inscribed in the (a) tan -1 2 (b) tan -1 3 (c) 2 tan -1 2 (d) 2 tan -1 3
parabola y 2 = 4ax having its focus at S. If chord AB n
1
lies towards the left of S, then the side length of this 27. If the 4th term in the expansion of æç px + ö÷ , n Î N
triangle is è xø
5 2
(a) a ( 2 - 3 ) (b) 2a ( 2 - 3 ) is and three normals to the parabola y = x are
2
(c) 4a ( 2 - 3 ) (d) 8a ( 2 - 3 ) drawn through a point (q, 0 ), then
21. Let C be a circle with centre (0, 1) and radius unity. P (a) q = p (b) q > p (c) q < p (d) pq = 1
is the parabola y = ax 2 . The set of values of a for 28. The set of points on the axis of the parabola
which they meet at a point other than origin is y 2 - 4 x - 2y + 5 = 0 from which all the three normals
1 1 1 1
(a) (0, ¥ ) (b) æç0, ö÷ (c) æç , ö÷ (d)æç , ¥ ö÷ to the parabola are real is
è 2ø è 4 2ø è2 ø (a) (k , 0) ; k > 1 (b) (k , 1) ; k > 3
2 (c) (k , 2) ; k > 6 (d) (k , 3 ) ; k > 8
22. Let S be the focus of y = 4 x and a point P be moving
on the curve such that its abscissa is increasing at the 29. The triangle formed by the tangent to the parabola
rate of 4 units/s. Then the rate of increase of the y = x 2 at the point whose abscissa is x 0 ( x 0 Î[1, 2 ]),
projection of SP on x + y =1 when p is at (4, 4) is the Y -axis and the straight line y = x 02 has the
3 greatest area if x 0 is equal to
(a) - 2 (b) - (c) - 1 (d) 2
2 (a) 0 (b) 1 (c) 2 (d) 3
2
23. If P is a point on the parabola y = 3 ( 2 x - 3 ) and M is 30. The set of points ( x , y ) whose distance from the line
the foot of perpendicular drawn from P on the y = 2 x + 2 is the same as the distance from (2, 0) is a
directrix of the parabola, then the length of each side parabola. This parabola is congruent to the parabola
of the equilateral triangle SMP, where S is the focus in standard from y = kx 2 for some k which is equal to
of the parabola, is 4 12 5 5
(a) 2 (b) 4 (c) 6 (d) 8 (a) (b) (c) (d)
5 5 4 12
Chap 05 Parabola 423

#L Parabola Exercise 2 :
More than One Correct Option Type Questions
n This section contains 15 multiple choice questions. 37. If a point P on y 2 = 4 x , the foot of the perpendicular
Each question has four choices (a), (b), (c) and (d) out of
from P on the directrix and the focus form an
which MORE THAN ONE may be correct.
equilateral triangle, then the coordinates of P may be
31. Equation of the common tangent to the circle (a) (3, - 2 3 ) (b) ( - 3, 2 3 )
x 2 + y 2 = 50 and the parabola y 2 = 40 x can be (c) (3, 2 3 ) (d) ( - 3, - 2 3 )
(a) x + y - 10 = 0 (b) x - y + 10 = 0
(c) x + y + 10 = 0 (d) x - y - 10 = 0 38. The locus of the foot of the perpendicular from the
2
32. Let PQ be a chord of the parabola y = 4 x . A circle focus on a tangent to the parabola y 2 = 4ax is
(a) the directrix (b) the tangent at the vertex
drawn with PQ as a diameter passes through the
(c) x = a (d) x = 0
vertex V of the parabola. If area of DPVQ = 20 unit 2 ,
then the coordinates of P are 39. The extremities of latusrectum of a parabola are (1, 1)
(a) (16,8) (b) (16, - 8 ) and (1, -1). Then the equation of the parabola can be
(c) ( - 16, 8 ) (d) ( - 16, - 8 ) (a) y 2 = 2x - 1 (b) y 2 = 1 - 2x
(c) y 2 = 2x - 3 (d) y 2 = 2x - 4
33. Let y 2 = 4ax be a parabola and x 2 + y 2 + 2bx = 0 be a
circle. If parabola and circle touch each other 40. If from the vertex of a parabola y 2 = 4ax a pair of
externally, then chords be drawn at right angles to one another and
(a) a > 0, b < 0 (b) a > 0, b > 0 with these chords as adjacent sides a rectangle be
(c) a < 0, b > 0 (d) a < 0, b < 0 made, then the locus of the further angle of the
34. Tangent is drawn at any point ( x 1 , y 1 ) other than the rectangle is
(a) an equal parabola
vertex on the parabola y 2 = 4ax . If tangents are (b) a parabola with focus at (8a, 0)
drawn from any point on this tangent to the circle (c) a parabola with directrix as x - 7a = 0
x 2 + y 2 = a 2 such that all the chords of chords of (d) not a parabola
contact pass through a fixed point ( x 2 , y 2 ), then 41. If two chords drawn from the point (4, 4) to the
y parabola x 2 = 4y are divided by the line y = mx in the
(a) x 1, a, x 2 are in GP (b) 1 , a, y 2 are in GP
2 ratio 1 : 2, then
y1 x 1
(c) - 4, , are in GP (d) x 1x 2 + y 1y 2 = a 2 (a) m Î ( - ¥, - 3 ) (b) m Î ( - ¥, - 3 - 1)
y2 x2
(c) m Î ( 3 , ¥ ) (d) m Î ( 3 - 1, ¥ )
35. Let P, Q and R are three co-normal points on the
42. Through a point P( - 2, 0 ), tangents PQ and PR are
parabola y 2 = 4ax . Then the correct statement(s) is/at
(a) algebraic sum of the slopes of the normals at P , Q and R
drawn to the parabola y 2 = 8 x . Two circles each
vanishes passing through the focus of the parabola and one
(b) algebraic sum of the ordinates of the points P , Q and R touching at Q and the other at R are drawn. Which of
vanishes the following point ( s ) with respect to the triangle
(c) centroid of the triangle PQR lies on the axis of the PQR lie ( s ) on the common chord of the two circles?
parabola (a) centroid (b) orthocentre
(d) Circle circumscribing the triangle PQR passes through (c) incentre (d) circumcentre
the vertex of the parabola
43. The set of points on the axis of the parabola
36. Let P be a point whose coordinates differ by unity 1
and the point does not lie on any of the axes of (y - 2 ) 2 = 4 æç x - ö÷ from which three distinct
è 2ø
reference. If the parabola y 2 = 4 x + 1 passes through
normals can be drawn to the parabola are
P, then the ordinate of P may be (a) (3, 2) (b) (1, 2)
(a) 3 (b) - 1 (c) (4, 2) (d) (5, 2)
(c) 5 (d) 1
424 Textbook of Coordinate Geometry

44. Three normals are drawn from the point (14, 7) to the 45. A quadrilateral is inscribed in a parabola, then
curve y 2 - 16 x - 8y = 0. Then the coordinates of the (a) the quadrilateral may be cyclic
feet of the normals are (b) diagonals of the quadrilateral may be equal
(a) (3, - 4 ) (b) (8, 16) (c) all possible pairs of adjacent sides may be perpendicular
(c) (0, 0) (d) (2, 2) (d) None of the above

#L Parabola Exercise 3 :
Paragraph Based Questions
n This section contains 8 paragraphs based upon each of Paragraph III
the paragraph 3 multiple choice questions have to be (Q. Nos. 52 to 54)
answered. Each of these questions has four choices (a), (b),
(c) and (d) out of which ONLY ONE is correct. Consider a parabola (P ) x 2 - 4 xy + 4y 2 - 32 x + 4y + 16 = 0.
Paragraph I 52. The focus of the parabola (P ) is
(Q. Nos. 46 to 48) (a) ( 2, 1) (b) ( - 2, 1) (c) ( - 2, - 1) (d) ( 2, - 1)
Consider a parabola P touches coordinate axes at ( 4, 0 ) and 53. Length of latusrectum of the parabola (P ) is
(0, 3 ).
3 6 12 24
(a) (b) (c) (d)
46. If focus of parabola P is (a , b ), then the value of b - a is 5 5 5 5
1 3 4 12
(a) (b) (c) (d) 54. Equation of directrix of parabola (P ) is
25 25 25 25
(a) x - 2y - 4 = 0 (b) 2x + y - 3 = 0
47. Length of latus rectum of parabola P is (c) x - 2y + 4 = 0 (d) 2x + y + 3 = 0
72 144
(a) (b) Paragraph IV
125 125
288 576 (Q. Nos. 55 to 57)
(c) (d)
125 125 If l and m are variable real numbers such that
48. Equation of directrix of parabola P is 5l 2 - 4lm + 6m 2 + 3l = 0, then the variable line lx + my = 1
(a) 4 x + 3y = 0 (b) 3 x + 4y = 12 always touches a fixed parabola, whose axis is parallel to the
(c) 3 x + 4y = 0 (d) 4 x + 3y = 12 X -axis.
Paragraph II 55. If (a , b ) is the vertex of the parabola, then the value of
(Q. Nos. 49 to 51) | a - b | is
Let C be the locus of the circumcentre of a variable triangle (a) 2 (b) 3
having sides Y -axis, y = 2 and ax + by =1, where (a , b ) lies on (c) 4 (d) 5
the parabola y 2 = 4 lx . 56. If (c , d ) is the focus of the parabola, then the value of
49. For l = 2, the product of coordinates of the vertex of 2 | d - c | is
(a) 1 (b) 2 (c) 4 (d) 8
the curve C is
(a) - 8 (b) - 6 (c) 6 (d) 8 57. If ex + f = 0 is directrix of the parabola and e, f are
1 prime numbers, then the value of | e - f | is
50. For l = , the length of smallest focal chord of the (a) 2 (b) 4
32
(c) 6 (d) 8
curve C is
8 Paragraph V
(a) (b) 2 (c) 4 (d) 8
3 (Q. Nos. 58 to 60)
51. The curve C is symmetrical about the line C 1 is a curve y 2 = 4 x , C 2 is curve obtained by rotating C 1 ,
3 3 120° in anti-clockwise direction C 3 is reflection of C 2 with
(a) x = - (b) y = -
2 2 respect to y = x and S1 , S 2 , S3 are focii of C 1 , C 2 and C 3 ,
3 3 respectively, where O is origin.
(c) x = (d) y =
2 2
Chap 05 Parabola 425

58. If (t 2 , 2 t ) are parametric form of curve C 1 , then the 63. Let D be the minimum area bounded by the tangent
parametric form of curve C 2 is and the coordinate axes, then the value of 8D is
1 1 (a) 1 (b) 2 (c) 4 (d) 8
(a) æç (t 2 + 2 3 t ), ( 3 t 2 + 2t )ö÷
è2 2 ø Paragraph VII
1 1
(b) æç ( - t 2 + 2 3 t ), ( 3 t 2 + 2t )ö÷ (Q. Nos. 64 to 66)
è2 2 ø A parabola (P ) touches the conic
1 1
(c) æç ( - t 2 + 2 3 t ), ( - 3 t 2 + 2t )ö÷ x 2 + xy +y 2 - 2 x - 2y + 1 = 0
è2 2 ø at the points when it is cut by the line x + y + 1 = 0.
1 1
(d) æç ( - t 2 + 2 3 t ), ( - 3 t 2 - 2t )ö÷ 64. If equation of parabola (P ) is
è2 2 ø
ax 2 + 2hxy + by 2 + 2 gx + 2 fy + c = 0, then the
59. Area of DOS 2 S3 is value of | a + b + c + f + g + h | is
1 1 (a) 8 (b) 10 (c) 12 (d) 14
(a) (b)
8 4
1 65. The length of latusrectum of parabola (P ) is
(c) (d) 1
2 (a) 2 (b) 3 2 (c) 5 2 (d) 7 2

60. If S1 ( x 1 , y 1 ), S 2 ( x 2 , y 2 ) and S3 ( x 3 , y 3 ), then the value of 66. If (a , b ) is the vertex of the parabola (P ), then the
Sx 12 + Sy 12 is value of | a - b | is
1 3
(a) 2 (b) 3 (a) 0 (b) (c) 1 (d)
2 2
(c) 4 (d) 5
Paragraph VI Paragraph VIII
(Q. Nos. 67 to 69)
(Q. Nos. 61 to 63)
y = 3 x is tangent to the parabola 2y = ax 2 + b.
Tangent to the parabola y = x 2 + ax + 1 at the point of
intersection of the Y -axis also touches the circle x 2 + y 2 = c 2 . 67. The minimum value of a + b is
It is known that no point of the parabola is below X-axis. (a) 2 (b) 4 (c) 6 (d) 8
2 68. If ( 2 , 6 ) is the point of contact, then the value of 2a is
61. The value of 5 c when a attains its maximum value is
(a) 1 (b) 3 (a) 2 (b) 3 (c) 4 (d) 5
(c) 5 (d) 7 69. If b =18, then the point of contact is
62. The slope of the tangent when C is maximum, is (a) (1, 3 ) (b) ( 2, 6 )
(a) - 1 (b) 0 (c) 1 (d) 2 (c) (3, 9) (d) (6, 18 )

#L Parabola Exercise 4 :
Single Integer Answer Type Questions
n
This section contains 10 questions. The answer to each 72. The tangents and normals are drawn at the
question is a single digit integer, ranging from 0 to 9 (both extremities of the latusrectum of the parabola
inclusive). y 2 = 4 x . The area of quadrilateral so formed is l sq
70. Two tangents are drawn from the point ( - 2, -1) to the units, the value of l is
parabola y 2 = 4 x . If q is the angle between these 73. Three normals are drawn from the point (a , 0 ) to the
tangents, then the value of tanq is parabola y 2 = x . One normal is the X-axis. If other
71. If the distances of two points P and Q from the focus of two normals are perpendicular to each other, then
a parabola y 2 = 4 x are 4 and 9 respectively, the the value of 4a is
distance of the point of intersection of tangents at P
and Q from the focus is
426 Textbook of Coordinate Geometry

74. AB is the chord of the parabola y 2 = 6 x with the 77. Radius of the largest circle which passes through the
vertex at A. BC is drawn perpendicular to AB meeting focus of the parabola y 2 = 4 x and contained in it, is
the axis at C. The projection of BC on the axis is l 78. If the circle ( x - 6 ) 2 + y 2 = r 2 and the parabola
units, then the value of l is y 2 = 4 x have maximum number of common chords,
75. The parabolas y = x 2 - 9 and y = lx 2 intersect at then the least integral value of r is
points A and B. If length of AB is equal to 2a and if 79. The slope of the line which belongs to the family of
la 2 + m = a 2 , then the value of m is lines (1 + a ) x + (a - 1)y + 2(1 - a ) = 0 and makes
76. Let n be the number of integral points lying inside shortest intercept on x 2 - 4y + 4 = 0 is
the parabola y 2 = 8 x and circle x 2 + y 2 = 16, then the
sum of the digits of number n is

#L Parabola Exercise 5 :
Matching Type Questions
n This section contains 3 questions. Each question has four
statements (A, B, C and D) given in Column I and four (B) If the tangents drawn from the point (0, 2) to (q) 1
statements (p, q, r and s) in Column II. Any given the parabola y2 = 4 ax are inclined at an angle
statement in Column I can have correct matching with 3p
, then the values of a are
one or more statement(s) given in Column II. 4
80. Match the following. (C) If two distinct chords of a parabola y2 = 4 ax (r) 2
Column I Column II passing through (a, 2a) are bisected on the line
x + y =1, then the length of latusrectum can be
(A) The number of common chords of the (p) Prime number
parabola x = y2 - 6 y + 11and (D) If the focus of the parabola x 2 - ay + 3 = 0 is (s) 3
y = x 2 - 6x + 11is (0, 2) and if two values of a are a1, a2 such that
a
(B) AB is a chord of the parabola y2 = 4 x (q) Composite a1 > a2, then the value of 1 is
a2
with vertex A, BC is drawn number
perpendicular to AB meeting the axis 82. Match the following.
at C. The projection of BC on the
axis of the parabola is Column I Column II
2 2
(A) The common chord of the circle x + y = 5 (p) (1, 2)
(C) The maximum number of common (r) Perfect number
normals of y2 = 4 ax and the parabola 6 y = 5x 2 + 7x will passes
and x 2 = 4 by is through the point (s)

(D) If the locus of the middle of point of (s) Even number (B) Tangents are drawn from point (2, 3) to the (q) (4 , 4 )
contact of tangents drawn to the parabola y2 = 4 x. Then, the points
parabola y2 = 8x and the foot of of contact are
perpendicular drawn from its focus to (C) From a point P on the circle x 2 + y2 = 5, the (r) (- 2, 1)
the tangents is a conic, then the length equation of chord of contact to the parabola
of latusrectum of this conic is y2 = 4 x is y = 2(x - 2). Then,
the coordinates of point P will be
81. Match the following.
(D) P(4 , - 4 ) and Q are points on the parabola (s) (9, - 6)
Column I Column II
y2 = 4 x such that the area of DPOQ is 6 sq
(A) If the parabola x 2 = ay makes an intercept of (p) -2 units, where O is the vertex. Then, the
length 40 on the line y - 2x = 1, then the coordinates of Q may be
values of a are
#L Parabola Exercise 6 :
Statement I and II Type Questions
n Directions (Q. Nos. 83 to 90) are Assertion-Reason type 86. Statement I The conic ax + by =1 represents a
questions. Each of these questions contains two
statements:
parabola.
Statement I (Assertion) and Statement II Conic
Statement II (Reason) ax 2 + 2hxy + by 2 + 2 gx + 2 fy + c = 0 represents a
Each of these questions also has four alternative choices,
only one of which is the correct answer. You have to select parabola, if h 2 = ab.
the correct choice as given below :
87. Statement I The lines from the vertex to the two
(a) Statement I is true, Statement II is true; Statement II is a
correct explanation for Statement I extremities of a focal chord of the parabola y 2 = 4ax
(b) Statement I is true, Statement II is true; Statement II is not a are perpendicular to each other.
correct explanation for Statement I
Statement II If extremities of focal chord of a
(c) Statement I is true, Statement II is false
parabola are (at 12 , 2at 1 ) and (at 22 , 2at 2 ), then t 1t 2 = - 1.
(d) Statement I is false, Statement II is true
83. Statement I The equation of the common tangent to 88. Statement I Length of focal chord of a parabola
the parabolas y 2 = 4 x and x 2 = 4y is x + y + 1 = 0. y 2 = 8 x making an angle of 60° with X-axis is 32/3.

Statement II Both the parabolas are reflected to Statement II Length of focal chord of a parabola
each other about the line y = x . y 2 = 4ax making an angle a with X-axis is
84. Statement I Two perpendicular normals can be 4a sec 2 (a / 2 ).
5
drawn from the point æç , - 2 ö÷ to the parabola 89. Statement I Straight line x + y = l touch the
è2 ø
(y + 2 ) 2 = 2( x - 1). parabola y = x - x 2 , if k =1.

Statement II Two perpendicular normals can be Statement II Discriminant of ( x - 1) 2 = x - x 2 is


drawn from the point (3a , 0 ) to the parabola y 2 = 4ax . zero.
a 90. Statement I Length of latusrectum of parabola
85. Statement I The line y = mx + is tangent to the (3 x + 4y + 5 ) 2 = 4( 4 x + 3y + 2 ) is 4.
m
parabola y 2 = 4ax for all values of m. Statement II Length of latusrectum of parabola
Statement II A straight line y = mx + c that intersects y 2 = 4ax is 4a.
the parabola y 2 = 4ax one point is a tangent line.

Parabola Exercise 7 :
Subjective Type Questions
n
In this section, there are 15 subjective questions. 93. Prove that the normal chord to a parabola at the
2
91. If the tangent to the parabola y = 4ax meets the axis point whose ordinate is equal to the abscissa
subtends a right angle at the focus.
in T and tangent at the vertex A in Y and the
rectangleTAYG is completed, show that the locus of 94. Find the shortest distance between the parabola
G is y 2 + ax = 0. y 2 = 4 x and circle x 2 + y 2 - 24y + 128 = 0.

92. If incident ray from point ( -1, 2 ) parallel to the axis of 95. Show that the locus of a point that divides a chord of
the parabola y 2 = 4 x strikes the parabola, find the slope 2 of the parabola y 2 = 4 x internally in the ratio
equation of the reflected ray. 1 : 2 is a parabola. Find the vertex of this parabola.
428 Textbook of Coordinate Geometery

96. Show that the locus of the points of intersection of 101. A family of chords of the parabola y 2 = 4ax is drawn
tangents to y 2 = 4ax , which intercept a constant so that their projections on a straight line inclined
length d on the directrix is (y 2 - 4ax )( x + a ) 2 = d 2 x 2 . equally to both the axes are all of a constant length c;
prove that the locus of their middle points is the
97. Through the vertex O of the parabola y 2 = 4ax two curve (y 2 - 4ax ) (y + 2a ) 2 + 2a 2 c 2 = 0.
chords OP and OQ are drawn and the circle on OP
and OQ as diameters intersect in R. If q 1 and q 2 are 102. The normals at P , Q , R are concurrent and PQ meets
the angles made with the axes by the tangents at P the diameter through R on the directrix x = - a . Prove
and Q to the parabola and f is the angle made by OR that PQ touches [or PQ envelopes] the parabola
with the axis of the parabola, then prove that y 2 + 16a ( x + a ) = 0.
cotq 1 + cotq 2 + 2 tan f = 0.
103. If the normals to the parabola y 2 = 4ax at three
98. Three normals with slopes m1 , m 2 and m3 are drawn points P , Q and R meet at A and S be the focus, prove
from a point P not on the axis of the parabola that SP × SQ × SR = a (SA ) 2 .
y 2 = 4 x . If m1m 2 = a, results in the locus of P being a
104. From a point A common tangents are drawn to the
part of the parabola, find the value of a.
circle x 2 + y 2 = (a 2 / 2 ) and the parabola y 2 = 4ax .
99. Find the locus of centres of a family of circles passing Find the area of the quadrilateral formed by the
through the vertex of the parabola y 2 = 4ax and common tangents, the chords of contact of the point
cutting the parabola orthogonally at the other point A, with respect to the circle and the parabola.
of intersection. 105. Prove that the any three tangents to a parabola
100.TP andTQ are tangents to the parabola y = 4ax . The 2 whose slopes are in harmonic progression enclose a
normals at P and Q intersect at R on the curve. Prove triangle of constant area.
that the circle circumscribing the DTPQ lies on the
parabola 2y 2 = a ( x - a ).

#L Parabola Exercise 8 :
Questions Asked in Previous 13 Year’s Exams
n
This section contains questions asked in IIT-JEE, AIEEE, (a) ( x + y ) 2 = ( x - y - 2) (b) ( x - y ) 2 = ( x + y + 2)
JEE Main & JEE Advanced from year 2005 to 2017. (c) ( x - y ) 2 = 4( x + y - 2) (d) ( x - y ) 2 = 8( x + y - 2)
2
106. Tangent to the curve y = x + 6 at a point (1, 7)
109. The equations of the common tangents to the
touches the circle x 2 + y 2 + 16 x + 12y + c = 0 at a
parabolas y = x 2 and y = - ( x - 2 ) 2 is/are
point Q. Then the coordinates of Q are
[IIT-JEE 2005, 3M] (a) y = 4( x - 1) (b) y = 0 [IIT-JEE 2006, 5M]
(a) ( - 6, - 11) (b) ( - 9, - 13 ) (c) y = - 4( x - 1) (d) y = - 30x - 50
(c) ( - 10, - 15) (d) ( - 6, - 7 )
110. The locus of the vertices of the family of parabolas
107. Let P be a point (1, 0) and Q a point on the locus a3x 2 a 2x
y 2 = 8 x . The locus of mid-point of PQ is y= + - 2a is [AIEEE 2006, 4.5 M ]
[AIEEE 2005, 3M] 3 105 2 3
(a) xy = (b) xy =
(a) x 2 - 4y + 2 = 0 (b) x 2 + 4y + 2 = 0 64 4
2 2 35 64
(c) y + 4 x + 2 = 0 (d) y - 4 x + 2 = 0 (c) xy = (d) xy =
16 105
108. The axis of a parabola is along the line y = x and the
111. Angle between the tangents to the curve
distance of its vertex from origin is 2 and that from
y = x 2 - 5 x + 6 at the points (2, 0) and (3, 0) is
its focus is 2 2. If vertex and focus both lie in the
first quadrant, the equation of the parabola is
[AIEEE 2006, 4 .5 M ]
(a) p / 3 (b) p / 2
[IIT-JEE 2006, 3M] (c) p / 6 (d) p / 4
Chap 05 Parabola 429

112. Consider the circle x 2 + y 2 = 9 and the parabola 117. The tangent PT and the normal PN to the parabola
y 2 = 8 x . They intersect at P and Q in the first and y 2 = 4ax at a point P on it meet its axis at pointsT
fourth quadrants, respectively. Tangents to the circle and N, respectively. The locus of the centroid of the
at P and Q intersect the X-axis at R and tangents to DPTN is a parabola whose [IIT-JEE 2009, 4M ]
the parabola at P and Q intersect the X-axis at S. æ 2a ö
(a) vertex is ç , 0÷ (b) directrix is at x = 0
(i) The ratio of the areas of the DPQS and DPQR is è3 ø
(a) 1 : 2 (b) 1 : 2 2a
(c) latusrectum is (d) focus is (a, 0)
(c) 1 : 4 (d) 1 : 8 3

(ii) The radius of the circumcircle of the DPRS is 118. Let A and B be two distinct points on the parabola
(a) 5 (b) 3 3 y 2 = 4 x . If the axis of the parabola touches a circle of
(c) 3 2 (d) 2 3 radius r having AB as its diameter, The slope of the
line joining A and B can be [IIT-JEE 2010, 3M ]
(iii) The radius of the incircle of the DPQR is
1 1 2 2
[IIT-JEE 2007, (4 + 4 + 4) M] (a) - (b) (c) (d) -
(a) 4 (b) 3 r r r r
(c) 8/3 (d) 2 119. If two tangents drawn from a point P to the parabola
x 2 y 2 = 4 x are at right angles, the locus of P is
113. Statement I The curve y = - + x + 1 is symmetric [AIEEE 2010, 4M ]
2 (a) 2x + 1 = 0 (b) x = - 1
with respect to the line x =1 because (c) 2x - 1 = 0 (d) x = 1
Statement II A parabola is symmetric about its axis. 120. Consider the parabola y 2 = 8 x . Let D1 be the area of
[IIT-JEE 2007, 3M ] the triangle formed by the end points of its
(a) Statement I is true, Statement II is true; Statement II is a æ1 ö
correct explanation for Statement I latusrectum and the point P ç , 2 ÷ on the parabola
è2 ø
(b) Statement I is true, Statement II is true; Statement II is
not a correct explanation for Statement I and D 2 be the area of the triangle formed by drawing
(c) Statement I is true, Statement II is false tangent at P and at the end points of the latusrectum.
(d) Statement I is false, Statement II is true D
Then, 1 is [IIT-JEE 2011, 4M ]
D2
114. The equation of a tangent to the parabola y 2 = 8 x is
y = x + 2. The point on this line from which the other 121. Let ( x , y ) be any point on the parabola y 2 = 4 x . Let P
tangent to the parabola is perpendicular to the given
be the point that divides the line segment from (0, 0)
tangent is [AIEEE 2007, 3M ] to ( x , y ) in the ratio 1 : 3. Then, the locus of P is
(a) ( - 1, 1) (b) (0, 2) [IIT-JEE 2011, 3M ]
(c) (2, 4) (d) ( - 2, 0) 2 2
(a) x = y (b) y = 2x
115. Consider the two curves 2
(c) y = x (d) x 2 = 2y
C 1 :y 2 = 4 x , C 2 : x 2 + y 2 - 6 x + 1 = 0, then
[IIT-JEE 2008, 3M ] 122. Let L be a normal to the parabola y 2 = 4 x . If L passes
(a) C 1 and C 2 touch each other only at one point through the point (9, 6), then L is given by
[IIT-JEE 2011, 4M ]
(b) C 1 and C 2 touch each other exactly at two points
(a) y - x + 3 = 0 (b) y + 3 x - 33 = 0
(c) C 1 and C 2 intersect (but do not touch) at exactly
(c) y + x - 15 = 0 (d) y - 2x + 12 = 0
two points
(d) C 1 and C 2 neither intersect nor touch each other 123. The shortest distance between line y - x =1 and curve
116. A parabola has the origin as its focus and the line x = y 2 is [AIEEE 2011, 4M ]
x = 2 as the directrix. The vertex of the parabola is at 3 2 8
[AIEEE 2008, 3M ] (a) (b)
8 3 2
(a) (0, 2) (b) (1, 0)
(c) (0, 1) (d) (2, 0) 4 3
(c) (d)
3 4
430 Textbook of Coordinate Geometery

124. Let S be the focus of the parabola y 2 = 8 x and let PQ 131. Let O be the vertex and Q be any point on the
be the common chord of the circle parabola x 2 = 8y . If the point P divides the line
x 2 + y 2 - 2 x - 4y = 0 and the given parabola. The segment OQ internally in the ratio 1 : 3, then the
locus of P is [JEE Main 2015, 4 M]
area of the DPQS is [IIT-JEE 2012, 4M ]
(a) x 2 = y (b) y 2 = x
Paragraph (c) y 2 = 2x (d) x 2 = 2y
(Q. Nos. 125 and 126)
2
132. If the normals of the parabola y 2 = 4 x drawn at the
Let PQ be a focal chord of the parabola y = 4ax . The end points of its latusrectum are tangents to the
tangent to the parabola at P and Q meet at a point lying on circle ( x - 3 ) 2 + (y + 2 ) 2 = r 2 , then the value of r 2 is
the line y = 2 x + a, a > 0. [JEE Advanced 2013, 3+3 M] [JEE Advanced 2015, 4M]

125. If chord PQ subtends an angle q at the vertex of 133. Let the curve C be the mirror image of the parabola
y 2 = 4ax , them tanq is equal to y 2 = 4 x with respect to the line x + y + 4 = 0. If A and
2 2 2 2 B are the points of intersection of C with the line
(a) 7 (b) - 7 (c) 5 (d) - 5 y = - 5, the distance between A and B is
3 3 3 3
[JEE Advanced 2015, 4M]
126. Length of chord PQ is
(a) 7a (b) 5a (c) 2a (d) 3a 134. Let P and Q be distinct points on the parabola
y 2 = 2 x such that a circle with PQ as diameter passes
127. The slope of the line touching the parabolas y 2 = 4 x through the vertex O of the parabola. If P lies in the
and x 2 = - 32y is [JEE Main 2014, 4 M]
first quadrant and the area of the DOPQ is 3 2, then
which of the following is (are) the coordinates of P?
(a) 1/8 (b) 2/3
[JEE Advanced 2015, 4M]
(c) 1/2 (d) 3/2
128. The common tangent to the circle x 2 + y 2 = 2 and (a) ( 4, 2 2 ) (b) ( 9, 3 2 )

the parabola y 2 = 8 x touch the circle at the points æ1 1 ö


(c) ç , ÷ (d) (1, 2 )
P , Q and the parabola at the points R, S. Then, the è4 2ø
area of the quadrilateral PQRS is 135. Let P be the point on the parabola y 2 = 8 x , which is
[JEE Advanced 2014, 3 M] at a minimum distance from the centre C of the circle
(a) 3 (b) 6 (c) 9 (d) 15 x 2 + (y + 6 ) 2 = 1, the equation of the circle passing
Paragraph through C and having its centre at P, is
(Q. Nos. 129 and 130) [JEE Main 2016, 4 M]
2 2
(a) x + y - 4 x + 8y + 12 = 0
Let a , r , s and t be non-zero real numbers. Let P (at 2 2at ),
æ a -2a ö (b) x 2 + y 2 - x + 4y - 12 = 0
2 2
Qç , ÷ , R (ar , 2ar ) and S (as , 2as ) be distinct points x
èt 2 t ø (c) x 2 + y 2 - + 2y - 24 = 0
4
on the parabola y 2 = 4ax . Suppose that PQ is the focal
(d) x 2 + y 2 - 4 x + 9y + 18 = 0
chord and lines QR and PK are parallel, where K is the
point ( 2a , 0 ). [JEE Advanced 2014, (3 + 3) M] 136. The circle C 1 : x 2 + y 2 = 3 with centre at O, intersects
the parabola x 2 = 2y at the point P in the first
129. The value of r is
quadrant. Let the tangent to the circle C 1 at P
1 t2 +1 1 t2 -1
(a) - (b) (c) (d) touches other two circles C 2 and C 3 at R 2 and R3 ,
t t t t respectively. Suppose C 2 and C 3 have equal radii 2 3
130. If st =1, then the tangent at P and the normal at S to and centres Q 2 and Q 3 , respectively. If Q 2 and Q 3 lie
the parabola meet at a point whose ordinate is on the Y -axis, then [JEE Advanced 2016, 4M]
(t 2 + 1) 2 a(t 2 + 1) 2 (a) Q 2 Q 3 = 12
(a) (b) (b) R 2 R 3 = 4 6
2t 3 2t 3
a(t 2 + 1) 2 a(t + 2) 2
2 (c) area of DOR 2 R 3 is 6 2
(c) (d)
t3 t3 (d) area of DPQ 2 Q 3 is 4 2
Chap 05 Parabola 431

137. Let P be the point on the parabola y 2 = 4 x which is at (a) 4( 2 + 1)


the shortest distance from the centre S of the circle (b) 2( 2 + 1)
x 2 + y 2 - 4 x - 16y + 64 = 0. Let Q be the point on the (c) 2( 2 - 1)
circle dividing the line segment SP internally. Then, (d) 4( 2 - 1)
(a) SP = 2 5 [JEE Advanced 2016, 4M] 139. If a chord, which is not a tangent of the parabola
(b) SQ : QP = ( 5 + 1) : 2 y 2 =16 x has the equation 2x + y = p, and mid-point
(c) the x-intercept of the normal to the parabola at P is 6 (h, k ), then which of the following is (are) possible
1
(d) the slope of the tangent to the circle at Q is value(s) of p, h and k ? [JEE Advanced 2017, 4M]
2 (a) p = 2, h = 3, k = - 4
138. The radius of a circle, having minimum area, which (b) p = - 1, h = 1, k = - 3
touches the curve y = 4 - x 2 and the lines, y = | x | is (c) p = - 2, h = 2, k = - 4
(d) p = 5, h = 4, k = - 3
[JEE Main 2017, 4M]

Answers
Exercise for Session 1 25. (a) 26. (c) 27. (b) 28. (b) 29. (c) 30. (d)
1. (a) 2. (d) 3. (c) 4. (b) 5. (c) 6. (c) 31. (b,c) 32. (a,b) 33. (b,d) 34. (b,c,d) 35. (a,b,c, d) 36. (a, c)
7. (b) 8. (c) 9. (c) 10. (b) 11. (d) 12. (b) 37. (a,c) 38. (b,d) 39. (a,c) 40. (a,c) 41. (b,c,d) 42. (a,b,c,d)
13. 16x2 + 9 y2 + 24xy - 256x - 142 y + 849 = 0 43. (a,c,d) 44.(a,b,c) 45. (a,b) 46. (d) 47. (d) 48. (c)
49. (b) 50. (c) 51. (d) 52. (d) 53. (c) 54. (d)
14. 4x2 + y2 - 4xy + 104x + 148 y - 124 = 0
55. (b) 56. (b) 57. (d) 58. (d) 59. (b) 60. (b)
-7 5 -17 5 ö 5 11
15. æç , ö÷ ; æç , ÷; y = ; x + = 0; 3 61. (a) 62. (b) 63. (b) 64. (c) 65. (d) 66. (a)
è 2 2ø è 2 2ø 2 4
67. (c) 68. (b) 69. (d) 70. (3) 71. (6) 72. (8)
16. Parabola 17. Parabola 73. (3) 74. (6) 75. (9) 76. (8) 77. (4) 78. (5)
2 68 5
19. x = y2 - 5 y + ; 20. 4 79. (0)
5 5 2 80. (A) ® (q,r,s); (B) ® (q,s); (C) ® (p); (D) ® (q)
81. (A) ® (p,q); (B) ® (p,r); (C) ® (q,r,s); (D) ® (s)
Exercise for Session 2
82. (A) ® (p,r); (B) ® (p,q); (C) ® (r); (D) ® (p,s)
1. (d) 2. (b, d) 3. (a) 4. (a) 5. (d) 6. (d)
83. (a) 84. (a) 85. (a) 88. (c) 87. (d) 88. (c)
7. (b) 9. x + 4a = 0 10. (a) y = 2x - 12 (b) y = 3x - 33
89. (c) 90. (d) 92. x = 1
12 3
11. y2 = (x - 6 ) 2 13. l = 8 2
343 2 94. 4( 5 - 1) 95. æç , ö÷ 98. (2)
è9 9ø
Exercise for Session 3 99. 2 y2 (2 y2 + x2 - 12ax) = ax (3x - 4a)2
1. (b) 2. (a) 3. (d) 4. (a) 5. (c) 6. (c) 15a2
7. (a) 8. (c) 10. y2 = 2a (x - a) 11. y = x - 1, 8 2 sq units 104. sq units 106. (d) 107. (d) 108. (d) 109. (a, b)
4
110.(a) 111. (b) 112. [i] (c) [ii] (b) [iii] (d) 113. (a)
Chapter Exercise 114.(d) 115. (b) 116. (b) 117. (a,d) 118. (c,d) 119. (b)
1. (a) 2. (b) 3. (b) 4. (d) 5. (d) 6. (a) 120. (2) 121. (c) 122. (a,b,d)123. (a) 124. 4 sq units
7. (a) 8. (a) 9. (d) 10. (b) 11. (a) 12. (c) 125. (d) 126. (b) 127. (c) 128. (d) 129. (d) 130. (b)
13. (a) 14. (d) 15. (d) 16. (a) 17. (a) 18. (c)
131. (d) 132. (2) 133. (4) 134. (a,d)
19. (d) 20. (c) 21. (d) 22. (a) 23. (c) 24. (a)
135. (a) 136. (a,b,c) 137. (a,c,d) 138. (d) 139. (a)
Solutions
1. Equation of tangent of y 2 = 4bx is
or

or

or
k12 - l2k1 - l4 = 0

k1 =

k1
l2
FG
=

=
l2 ± ( l4 + 4 l4 )

1+ 5
2
5+1
2
[Q k1 > 0]

BC 2
b
y = mx + …(i) 5. Coordinates of A and B are
m
p (VA cos q, VA sin q) and (VB sin q, - VB cos q) respectively
Here, m = tan = 1
4 Q A and B lie on y 2 = x, then (VA sin q) 2 = VA cos q
From Eq. (i), y = x + b
Y
For common tangent y = x + b is also tangent of circle
A
x 2 + y 2 = a 2, then
|0-0+b|
=a θ
1+1 X′ X
V 90°– θ
Þ b =a 2 [Q a > 0, b > 0]
2. The coordinates of vertex and focus of required parabola are
(a, 0 ) and (a1, 0 ) respectively. Therefore, the distance between B
Y′
the vertex and the focus is AS = a1 - a . So, the length of
latusrectum is 4 (a1 - a ). Thus, the equation of the required cos q
parabola is Þ VA =
sin 2 q
(y - 0 ) 2 = 4 (a1 - a ) ( x - a )
sin q
or y 2 = 4 (a1 - a ) ( x - a ). Similarly, VB =
cos2 q
3. The parabolas are equal if the lengths of their latusrectum are | VA |
equal. \ = cot 3 q
| VB |
The length of latusrectum of y 2 = ax is a
6. Q x = t 2 - t + 1, y = t 2 + t + 1
The equation of second parabola can be written as
\ x + y = 2t 2 + 2 and y - x = 2t
æ 3 x - 4y - 2 ö
( x - 3 ) 2 + (y + 2 ) 2 = ç ÷
è 5 ø or 2 ( x + y - 2 ) = 4 t 2 = (2t ) 2 = (y - x ) 2
Here, focus is (3, - 2 ) and the equation of directrix is or (x - y )2 = 2 (x + y - 2)
3 x - 4y - 2 = 0. 2
æx -yö æ x + y - 2ö
\Length of latusrectum= 2 ´ Distance between focus and or ç ÷ = 2ç ÷
è 2 ø è 2 ø
directrix
x -y x + y -2
½ 9 - 4 ´ - 2 - 2½ For vertex, = 0 and =0
= 2½ ½= 6 2 2
½ (9 + 16 ) ½
or x = y and x + y = 2
Thus, the two parabolas are equal if a = 6. \ x =y =1
4. Let DC = CB = BA = AD = k Hence, vertex is (1, 1).
\ Coordinates of B are (k, k ), 7. It is clear from figure
which lie on y =l x Y
\ k=l k
\ k = l2 y2=4x
2
\ BC = k = l …(i)
X′ X
(–p,0) (0,0)
Also, let CG = GF = FE = EC = k1
\Coordinates of F are ( l2 + k1, k1 ),
which lie on y =l x
Y′
Then k1 = l ( l2 + k1 )
Þ k12 = l4 + l2k1 - p < 0 or p > 0
Chap 05 Parabola 433

8. The point of intersection of the parabolas y 2 = 4ax and meet the curve again at Q, then
x 2 = 4ay are (0, 0) and ( 4a, 4a ) but a ¹ 0. 2
t 2 = - t1 - …(i)
t1
Now, 2bx + 3cy + 4d = 0 passes through (0, 0) and ( 4a, 4a ).
Here tan a = - t1 and tan b = - t 2, from Eq. (i)
Therefore, d = 0 and 2b ( 4a ) + 3c ( 4a ) = 0 i.e.,
2
2b + 3c = 0 [Q a ¹ 0] \ - tan b = tan a +
tan a
or d 2 + (2b + 3c ) 2 = 0
Þ tan a (tan a + tan b ) = - 2
9. Q (OT ) 2 = OA × OB 12. Let A º ( a, b)
= a×b The equation of normal at (at 2, 2at )
Y
y + tx = 2at + at 3 …(i)
( a, b ) lie on Eq. (i), then
(0,c)
at 3 + (2a - a ) t - b = 0 …(ii)
X′ o X Let t1, t 2 and t 3 be the roots of Eq. (ii), then
B(β,0)
A(α,0) T at 3 + (2a - a ) t - b = a (t - t1 ) (t - t 2 ) (t - t 3 ) …(iii)
Let P º (at12, 2at1 ), Q º (at 22, 2at 2 ) and R º (at 32, 2at 3 )
Y′ Since, the focus is S (a, 0 )
\ SP = a (1 + t12 ), SQ = a (1 + t 22 )
or OT = a b …(i)
and SR = a (1 + t 32 )
Parabola cuts X -axis at a and b.
On putting t = i = - 1 in Eq. (iii), we get
\a, b are the roots of ax 2 + bx + c = 0
c - ai + (2a - a ) i - b
\ ab = …(ii) = a (i - t1 ) (i - t 2 ) (i - t 3 )
a
From Eqs. (i) and (ii), we get or | (a - a ) i - b | = a | i - t1 | | i - t 2 | i - t 3 |
c Þ (a - a ) 2 + b 2
OT =
a = a (1 + t12 ) (1 + t 22 ) (1 + t 32 )
10. Q Two perpendicular tangents meet a point on directrix.
or a ((a - a ) 2 + b 2 ) = a (1 + t12 ) × a (1 + t 22 ) × a (1 + t 32 )
Now, equations of tangents at (at12, 2at1 ) and (at 22, 2 at 2 ) are
a (SA ) 2 = SP × SQ × SR
t1y = x + at12 and t 2y = x + at 22, respectively.
or SP × SQ × SR = a (SA ) 2
\ P1 º ( - at12, 0 ), P2 º ( - at 22, 0 )
13. Let AB be a normal chord, where A º (at12, 2at1 ) and
Þ SP1 = a (1 + t12 ), SP2 = a (1 + t 22 ) and t1t 2 = - 1
2
1 1 1 1 B º (at 22, 2at 2 ), we have t 2 = - t1 -
\ + = + t1
SP1 SP2 a (1 + t12 ) a (1 + t 22 )
Now, AB = (at12 - at 22 ) 2 + (2at1 - 2at 2 ) 2
1 1 é 1ù
= + ê\ t 2 = - t ú
a (1 + t12 ) æ 1ö ë 1û
= a | t1 - t 2 | (t1 + t 2 ) 2 + 4
a ç1 + 2 ÷
è t1 ø ½ æ 1ö½ æ 4 ö
= a ½ 2 çt1 + ÷ ½ ç 2 + 4 ÷
1 t12 1 ½ è t ø
1 ½ è t1 ø
= + =
a (1 + t12 ) a (1 + t12 ) a (t12 + 1 ) 3
= 4a
11. Let P º (at12, 2at1 ) and Q º (at 22, 2at 2 ) t14
Y
(t12 + 1 ) 3
P
( AB ) 2 = 16a 2
t14

α β d ( AB ) 2 ì (1 + t12 ) 2 2 ü
X′ X
= 16a 2 í 5
(t1 - 2 )ý
A dt1 î t1 þ
2
d ( AB )
For = 0 Þ t1 = 2
dt1
Y′ Q For which ( AB ) 2 is minimum, thus
Equation of normal at P is 16a 2 (2 + 1 ) 3
ABmin = = 2a 27
y + t1 x = 2at1 + at13 4
434 Textbook of Coordinate Geometry

14. On solving \ Shortest distance = AB


x 2 + y 2 = a 2 and y 2 = 4 ( x + 4 ) æ 3 1ö æ1 3 ö
2 2
= ç - ÷ +ç - ÷
Þ x 2 + 4 (x + 4) = a 2 è 4 2ø è2 4ø
or x 2 + 4 x + 16 - a 2 = 0 æ1 1ö 1 1
= ç + ÷ = =
If the circle and parabola touch each other, then è 16 16 ø 8 2 2
D = 0 Þ16 - 4 × 1 × (16 - a 2 ) = 0 18. We know that normals at (at12, 2at1 ) and (at 22, 2at 2 ) meet again
Þ a 2 = 12 or a = 2 3 on the parabola. Then t1t 2 = 2
Here, a =1
15. sin q > | 2a | [Q h > 2a]
and x1 = at12 = t12 and y1 = 2at1 = 2t1
1
Þ 0 < | 2a | < 1 or 0 < | a | < x 2 = at 22 = t 22 and y 2 = 2at 2 = 2t 2
2
æ 1 ö æ 1ö Given x1 + x 2 = 4 Þ t12 + t 22 = 4
\ a Î ç - , 0÷ È ç 0, ÷
è 2 ø è 2ø or (t1 + t 2 ) 2 - 2t1t 2 = 4
2
16. Tangent to y = 4x in terms of m is Þ (t1 + t 2 ) 2 = 8 [Q t1t 2 = 2]
1 or | t1 + t 2 | = 8
y = mx + …(i)
m
or | 2t1 + 2t 2 | = 2 8
and normal to x 2 = 4by in terms of m is
or | y1 + y 2 | = 2 8 = 4 2
b
y = mx + 2b + 2 …(ii) 19. Let any point at distance r from A on the parabola is
m
Q Eqs. (i) and (ii) are same, then ( - 2 + r cos q, r sin q),
1 b then r 2 sin 2 q = 4 ( - 2 + r cos q)
= 2b + 2
m m or r 2 sin 2 q - 4r cos q + 8 = 0
Þ 2bm 2 - m + b = 0
Y
For two different tangents
Q
\ D > 0 Þ 1 - 8b 3 > 0
1 P
or |b| < X′ θ X
2 2 A(–2,0) O
2 2
17. The given parabolas 2y = 2x - 1 and 2x = 2y - 1 are
symmetrical about the line y = x . The shortest distance occurs
along the common normal which is perpendicular to the line Y′
y = x.
Let P and Q are distances r1 and r2 from A, then
Therefore, the tangent at point A on 2y 2 = 2 x - 1 is parallel to
4 cos q
y = x. Therefore, r1 + r2 =
sin 2 q
dy dy 1
4y =2 Þ = =1 8
dx dx 2y and r1r2 =
sin 2 q
1 3
or y = and x = 1 1 1 1 r +r cos q
2 4 Now, + = + = 1 2 =
æ 3 1ö AP AQ r1 r2 r1r2 2
\ A º ç , ÷ , then coordinates of B on 2 x 2 = 2y - 1 is 1 1 1
è 4 2ø given that + <
æ1 3 ö AP AQ 4
Bºç , ÷
è2 4ø cos q 1
Þ <
2 4
2x2=2y–1 y=x 1
B or cos q <
2
or tan q > 3
A
æ pö
2y2=2x–1
[Q cos q is decreasing and tan q is increasing in ç 0, ÷ ]
è 2ø
\ Slope (m ) > 3
Chap 05 Parabola 435

20. Coordinates of A are (a - l cos 30° , l sin 30° ) 2a - 1


or x2 = >0
Y a2
or 2a - 1 > 0
A
1
l \ a>
60° 2
X′ 30° X
O M 30° S(a,0) æ1 ö
60° or a Î ç , ¥÷
l è2 ø
B ®
22. V = ( l2 - 1) $i + (2l - 0) $j
Y′ ®
V = ( l2 - 1 ) $i + (2 l ) $j …(i)
æ l 3 lö
or A º ça - , ÷
è 2 2ø Y
2
which lies on y = 4ax, then p(l2, 2l)

l2 æ l 3ö 2 2 (0,1) M →
= 4a ça - ÷ Þl + 8a 3l - 16a = 0 v
4 è 2 ø
o X
(1,0)S
- 8a 3 ± (192a 2 + 64a 2 ) x+y=1
\ l =
2
- 8a 3 ± 16a
=
2
= 4a (2 - 3 ) [taking +ve sign] ®
and n = ( 0 - 1 ) i$ + (1 - 0 ) $j
Aliter :
Let A º (at12, 2at1 ) and B º (at12 - 2at1 ). = - $i + $j
® ®
Slope of SA = tan (180° - 30° ) The projection of V on n = y [given]
2at1
= - tan 30° ® ®
at12 - a V× n -( l2 - 1 ) + 2 l
\ y = = …(ii)
2t1 1 ® 2
=- | n|
t12 - 1 3
dx
or t12 + 2 3t1 - 1 = 0' or t1 = - 3 ± 2 Given, =4
dt
Thus, t1 = 2 - 3 [Q t1 = - 2 - 3 rejected]
d 2 dl
Here, AB = 4at1 = 4a (2 - 3 ) Þ (l - 1) = 4 Þ 2l =4
dt dt
21. C : x 2 + (y - 1) 2 = 1 and P : y = ax 2 When P º ( 4, 4 ),
y We have l = 2, therefore
Putting x = in x 2 + (y - 1 ) 2 = 1
2
a dl
y =1 …(iii)
or + (y - 1 ) 2 = 1 dt
a
y From Eq. (ii),
or y 2 - 2y + = 0
a dl
(2 - 2 l )
1 dy dt
\ y = 0 or y = 2 - = [from Eq. (ii)]
a dt 2
Y (2 - 4 ) ´ 1
= =- 2
2
23. Given parabola is
(0,1) 3ö
æ
y 2 = 6 çx - ÷
è 2ø
X′ X
Y′ 3 6
The equation of directrix is x - + =0
2 4
1
On substituting y = 2 - in y = ax 2, then i.e. x=0
a
æ3 3 ö
1 Let the coordinates of P be ç + t 2, 3t ÷
ax 2 = 2 - è2 2 ø
a
436 Textbook of Coordinate Geometry

Y
Þ P , R, Q are collinear.
P 3 3 t2, 3t 2
+
(0,3t) M 60° 2 2 Slope of PQ = [Q OR ^ PQ]
60° t1 + t 2
60°
(t + t 2 )
X′
O
X \ Slope of OR = - 1 = tan f
S (3,0) 2
or t1 + t 2 = - 2 tan f
Þ cot q1 + cot q2 = - 2 tan f
Y′ 26. Let A º (at 2, 2at ) and B º (at 2, - 2at )
Therefore, MS = MP Y
3 3 A
(9 + 9t 2 ) = + t 2
2 2 A1
9
or 9 + 9t = (1 + t 2 ) 2
2
θ
4 X′ X
O θ
or 1 + t2 = 4 S(a,0) M
B1
\Length of each side = MS
B
= 9 (1 + t 2 ) = 36 = 6 Y′

24. The area of DABC is maximum if CD is maximum, because AB


The equations of tangents at A and B are
is fixed.
Y ty = x + at 2 and - ty = x + at 2.
B (9,6)
These tangents meet the Y -axis atA1 ( 0, at ) and B1( 0, - at )
C respectively.
Area of trapezium
X′ X 1
O D AA1 B1B = (AB + A1B1 ) ´ OM
2
A(
4,– 1
4) 24a = ( 4at + 2at ) ´ at 2
2
2
Y′
24a 2 = 3a 2t 3 Þ t 3 = 8
It is clear that tangent drawn to the parabola at C should be
parallel to AB. \ t =2
For y 2 = 4x \Coordinates of A1 and B1 are ( 0, 2a ) and ( 0, - 2a ) respectively.
dy dy 2 If ÐOSA1 = ÐOSB1 = q, then
\ 2y = 4 or = = slope of AB 2a
dx dx y \ tan q = =2
a
2 6+4 1
Þ = = 2 or y = 1, then x = Þ q = tan -1 2
y 9-4 4
Hence, subtended angle = 2 q = 2 tan -1 2
æ1 ö
Hence, coordinates of C are ç , 1 ÷ . 3
è4 ø 5 n æ1ö
27. Given, = C 3( px )n - 3 ç ÷ = nC 3 × pn - 3x n-6
…(i)
2 èxø
25. Q Tangent at P (at12, 2at1 ) is t1y = x + at12.
1 Since, LHS of Eq. (i) is independent of x
\ tan q1 = \ n - 6 = 0 Þn = 6
t1
Y 2 at 1) From Eq. (i),
,2
p(at 1 5 6
= C 3p 3 = 20 p 3
2
3
æ1ö 1
X′ O X Þ p3 = ç ÷ Þ p=
R è2ø 2
Given, parabola is y 2 = x
Q(a 2 Here, 4a = 1
t 2 , 2 at
Y′ 2) 1
Þ a=
1 4
Similarly, tan q2 =
t2 Since, three normals are drawn from point (q, 0 ),
Q Angle in a semicircle is 90°. 1 é 1ù
\ q > 2a or q > or q > p êëQ p = 2 úû
\ ÐORP = ÐORQ = 90° 2
Chap 05 Parabola 437

2
28. y 2 - 4 x - 2y + 5 = 0 æ x + 2y - 2 ö 12 æ 2 x - y - 1 ö
Þ ç ÷ = ç ÷
Þ (y - 1 ) 2 - 4 x + 4 = 0 è 5 ø 5 è 5 ø

Þ (y - 1 ) 2 = 4 ( x - 1 ) x + 2y - 2 2x - y - 1
Let = X, =Y
5 5
Let y - 1 = Y and x - 1 = X
12 5 2
then, y 2 = 4 X comparing with Y 2 = 4ax or X2 = Y or Y = X
5 12
\ a =1
5
Q All three normals to the parabola are real and meet on the \ k=
12
axis of parabola, then
X > 2a and Y = 0 31. Equation of tangent of parabola
10
i.e., x - 1 > 2 and y - 1 = 0 y 2 = 40 x is y = mx + … (i)
\ x > 3 and y = 1 m
or (k, 1 ); k > 3 which is also tangent of circle x + y 2 = 50 , then
2

29. Let P ( x 0, x 02 ) be any point on the parabola y = x 2 10


Y m
=5 2
(m 2 + 1 )
y = x 02
Þ m4 + m2 - 2 = 0
Q P(x0,x02)
Þ (m 2 + 2 ) (m 2 - 1 ) = 0
X \ m 2 = 1, m 2 + 2 ¹ 0 or m = ± 1
O
From Eq. (i), common tangents are
y = x + 10 and y = - x - 10
2 or x - y + 10 = 0 and x + y + 10 = 0
T (0,–X0 )
32. Let coordinates of P be (t 2, 2t )
Equation of tangent at P ( x 0, x 02 ) is
2t - 0 2
1 \ Slope of PV = =
xx 0 = (y + x 02 ) t2 - 0 t
2
Þ 2 xx 0 - y - x 02 = 0 t
Þ Slope of QV is -
2
Tangent meets the Y -axis at T ( 0, - x 02 ). -t
1 \ Equation of QV is y = x
Hence, the area of the triangle DPTQ = ´ PQ ´ QT 2
2 æ 16 - 8 ö
1 Solving it with y 2 = 4 x, we get Q ç 2 , ÷
= ´ x 0 ´ 2 x 02 = x 03 èt t ø
2
Y
which increases in the interval [1, 2 ] and hence is greatest
when x 0 = 2.
P
æ 2x - y + 2 ö 2 2
30. ç ÷ = ( x - 2 ) + (y - 0 ) [given]
è 5 ø
X¢ X
Þ (2 x - y + 2 ) 2 = 5 ( x 2 + y 2 - 4 x + 4 ) V
or x 2 + 4y 2 + 4 xy = 28 x - 4y - 16
Þ ( x + 2y ) 2 = 4 (7 x - y - 4 )
Q
Q x + 2y = 0 and 7 x - y - 4 = 0 and are not perpendicular. Y¢
\ ( x + 2y + l ) 2 = (2 l + 28 ) x + ( 4 l - 4 ) y + l2 - 16 …(i)
Area of DPVQ = 20 (given)
Now, (slope of x + 2y + l = 0) ´ (slope of 2
1 t 2t
(2 l + 28 ) x + ( 4 l - 4 ) y + l2 - 16 = 0 ) = - 1 Þ | 16 - 8 | = 20
1 (2 l + 28 ) 2
\ - ´- = -1 t2 t
2 (4l - 4) 4
Þ t+ =±5
Þ 2 l + 28 = - 8 l + 8 t
or 10 l = - 20 Þ t 2 - 5t + 4 = 0 or t 2 + 5t + 4 = 0
\ l = -2
\ t = 1, 4 or t = - 1, - 4
From Eq. (i),
Hence coordinates of P are (1, 2), (16, 8), (1 - 2 ), (16, - 8 )
( x + 2y - 2 ) 2 = (24 x - 12y - 12 )
438 Textbook of Coordinate Geometry

33. Parabola is y 2 = 4ax and circle is ( x + b ) 2 + (y - 0) 2 = b 2 35. Equation of normal in slope form is
If parabola and circle touch each other externally, then y = mx - 2am - am 3,

Y if normals meet at (h, k ), then


a>0 am 3 - (h - 2a )m + k = 0 ...(i)
Let P º (am12, - 2am1 ), Q = (am22, - 2am2 )
X¢ X and R º (am32, - 2am3 )
(–b, 0) S(a, 0)
Þ m1, m2, m3 are the roots of Eq. (i), then
m1 + m2 + m3 = 0 ...(ii)

i.e. algebraic sum of the slopes of the normals at P , Q and R
vanishes.
Y
a>0 From Eq. (ii)
- 2am1 - 2am2 - 2am3 = 0
i.e. algebraic sum of the ordinates of the points P , Q and R
X¢ X vanishes.
S(a, 0) (–b, 0)
Also, y-coordinate of centroid of DPQR is zero
\ centroid lies on X -axis
Y¢ and circle circumscribing the triangle PQR always passes
through the vertex of the parabola.
If a > 0, - b < 0 and if a < 0, - b > 0 36. Let P º ( l, l + 1), where l ¹ 0, - 1
or a > 0, b > 0 and a < 0, b < 0 or P º (l, l - 1), where l ¹ 0, 1
2
34. Let ( x1, y1 ) º (at , 2at ) The point (l, l + 1) is on y 2 = 4 x + 1, therefore
Equation of tangent at (at 2, 2at ) is ty = x + at 2 (l + 1)2 = 4 l + 1
æ l + at 2 ö Þ l2 - 2 l = 0
Let any point on this tangent is ç l, ÷
è t ø \ l =2
Therefore, the ordinate of P is 3
The chord of contact of this point w.r.t the circle x 2 + y 2 = a 2
and the point ( l, l - 1) is on y 2 = 4 x + 1, therefore
is
æ l + at 2 ö ( l - 1 ) 2 = 4( l) + 1
x×l + y × ç ÷ = a2 Þ l2 - 6 l = 0
è t ø
æ yö \ l =6
Þ (aty - a 2 ) + l × ç x + ÷ = 0 Therefore, the ordinate of P is 5,
è tø
which is family of straight lines passing through the point of 37. From figure,
intersection of
y
aty - a 2 = 0 and x + = 0 P (t2, 2t)
t (–1, 2t) A
æ-a aö 60° 60°
So, the fixed point is ç 2 , ÷ , therefore
è t tø
60°
a a
x2 = - 2 , y 2 =
t t O S (1, 0)
2 2
Clearly, x1x 2 = - a , y1y 2 = 2a
Þ x1x 2 + y1y 2 = a 2
x1 y1
Also, = - t4 , = 2t 2
x2 y2
2 Slope of AP = 0 and slope of AS = - t
æ y1 ö æ x1 ö
or ç ÷ + 4ç ÷ = 0 0 - (- t )
èy2 ø è x2 ø \ tan 60° =
1+0
y1 x1
Þ - 4, , are in G.P. Þ 3 =|t |
y 2 x2
y Þ t=± 3
Also, y1y 2 = 2a 2 Þ 1 , a, y 2 are in G.P.
2 \ Coordinates of P are (3, ± 2 3 )
Chap 05 Parabola 439

38. Let M( a, b) be the foot of the perpendicular from the focus QOPQR is a rectangle.
S (a, 0 ) on any tangent to the parabola at P (at 2, 2at ). \Mid-point of OQ = mid-point of PR
i.e. ty + x + at 2 16a x 16
Þ x = at 2 + 2 Þ = t2 + 2 … (i)
2 t a t
Þ a - tb + at = 0 …(i)
8a y 4
Y and y = 2at - Þ =t - … (ii)
t 2a t
P
M From Eqs. (i) and (ii), we get
2
æy ö 2 16 x
ç ÷ =t + 2 -8 = -8
è 2a ø t a
X¢ X
O S (a, 0) Þ y 2 = 4ax - 32a 2
or y 2 = 4a( x - 8a )
which is equal parabola and focus (9a, 0 ) and directrix
Y¢ x - 8a = - a
or x - 7a = 0.
Since, SM is perpendicular to the tangent
1 b-0 41. Let point of intersection of the line y = mx with the chord be
\ ´ = -1 (l, ml), then
t a -a
1.4 + 2. x1 3l - 4
Þ at + b - at = 0 …(ii) l= Þ x1 =
1+2 2
On solving Eqs. (i) and (ii), we get
1.4 + 2. y1 3ml - 4
a=0 and ml = Þ y1 =
1+2 2
The locus of M( a, b ) is the tangent at the vertex.
i.e. x=0 Y
y=mx
39. Given that the extremities of the latusrectum are (1, 1) and x2=4y
1
(1, - 1 ), then 4a = 2 or a = P (4, 4)
2
2
So, the focus of the parabola is (1, 0).
æ1 ö æ3 ö 1
Hence, the vertex can be ç , 0 ÷ or ç , 0 ÷ .
è2 ø è2 ø (x1, y1) Q
æ 1ö2 X¢ X
Therefore, the equations of the parabola can be y = 2 ç x - ÷
è 2ø
2 æ 3ö Y¢
or y = 2 ç x - ÷
è 2ø
QQ ( x1, y1 ) lies on the parabola x 2 = 4y , then
Þy 2 = 2 x - 1 or y 2 = 2 x - 3. 2
æ 3l - 4 ö æ 3ml - 4 ö
40. Let P º (at 2, 2at ) and R º (at12, 2at1 ) ç ÷ = 4ç ÷
è 2 ø è 2 ø
Q OP ^ OR
Þ 9 l2 - 24 l(1 + m ) + 48 = 0
P For two distinct chords D > 0
Þ (24 ) 2 (1 + m ) 2 - 4 × 9 × 48 > 0
or (1 + m ) 2 > 3
O Q (x, y) Þ 1+m<- 3
or 1+m> 3
R \ m < - 3 -1
or m > 3 -1
\Slope of OP ´ Slope of OR = - 1
2 2 Hence , m Î ( -¥, - 3 - 1 ) È ( 3 - 1, ¥ )
Þ ´ = -1
t t1 42. The given parabola is y 2 = 4(2) x
4
\ t1 = - Þ a =2
t
Since, P( - 2, 0 ) lies on the directrix and the axis.
æ 16a - 8a ö
Now, coordinates of R are ç 2 , ÷
è t t ø
440 Textbook of Coordinate Geometry

Þ The tangents will have slope m = ± 1 and the equations are The diagonals of the quadrilateral may be equal as the
y = x + 2 and y = - x - 2. quadrilateral may be an isosceles trapezium.
A rectangle cannot be inscribed in a parabola.
Directior

Q Sol. (Q. Nos. 46 to 48)


Let A º ( 4, 0 ) and B º ( 0, 3 ).
Q OA and OB are mutually perpendicular tangents to the
45°

45°
X parabola. Therefore, O will lie on the directrix of the parabola.
(–2, 0) P S(2, 0)
Y

The chord of contact of tangents is QR as x = 2 (i.e. L.R.)


Q Common chord of two circles is X -axis.
Q ÐQPR = 90° L
B (0, 3)
\Circumcentre is S(2, 0 ) on X -axis and orthocentre is P( - 2, 0 )
S(α, β)
on X -axis.
X
Centroid and incentre also lies on X -axis, O A (4, 0)
(Q orthocentre, centroid, circumcentre and
incentre are collinear). M
æ 1ö
43. Given parabola is (y - 2) 2 = 4ç x - ÷ Let S(a, b) be the focus of the parabola.
è 2ø
1 Q Portion of a tangent to a parabola intercepted between the
Let x - = X, y - 2 = Y directrix and point of contact subtends a right angle at the
2
focus.
\ Parabola is Y 2 = 4 X p
\ ÐOSA = ÐOSB =
Any point on axis of parabola is ( x, 2 ) for three distinct normals 2
X > 2.1 Now, OS ^ SA Þ Slope of OS ´ Slope of SA = - 1
1 5
Þ x - > 2 or x > æ b-0 ö æ b - 0 ö
2 2 Þ ç ÷´ç ÷ =- 1
è a - 0ø è a - 4ø
\ x = 3, 4, 5
Hence, points are (3, 2), (4, 2) and (5, 2). Þ a2 + b2 - 4a = 0 ...(i)
44. The given parabola is y 2 - 16x - 8y = 0 Again, OS ^ SB Þ Slope of OS ´ Slope of SB = - 1
2 æ b - 0 ö æ b- 3 ö
Þ (y - 4 ) = 16( x + 1 ) Þ ç ÷´ç ÷ = -1
è a - 0ø è a - 0ø
Shifting the origin to the point ( - 1, 4 ) the equation of parabola
becomes y 2 = 16 x Þ a 2 + b 2 - 3b = 0 ...(ii)
then the coordinates of the point (14, 7) becomes (15, 3). From Eqs. (i) and (ii), we get
\Equation of any normal to the parabola is Y + tX = 8t + 4t 3. 4 a = 3b ...(iii)
Since, tangents AO and BO at A and B to parabola are at right
Since, it passes through (15, 3)
angles, therefore AB will be a focal chord of the parabola.
\ 3 + 15t = 8t + 4t 3 Þ 4t 3 - 7t - 3 = 0
Equation of AB is
3 1 x y a b
or (t + 1 ) (2t - 3 ) (2t + 1 ) = 0 Þ t = - 1, , - + =1 Þ + =1
2 2 4 3 4 3
\Corresponding points are ( 4, - 8 ), (9, 12) and (1, - 4 ). Þ 3 a + 4b = 12 ...(iv)
Hence, the coordinates of the feet of the normals w.r.t. the
From Eqs. (iii) and (iv), we get
original system of coordinates are (3, - 4 ), (8, 16) and (0, 0).
36 48
45. As a circle can intersect a parabola at four points, the a = and b =
25 25
quadrilateral may be cyclic.
æ 36 48 ö
\ Focus of parabola is ç , ÷ .
è 25 25 ø
46. Q Focus is (a, b ).
12
\ b - a = b- a =
25
Chap 05 Parabola 441

36 ö
2
48 ö 16
2 3
æ æ 49. Product of coordinates of vertex = - 2l ´ = - 3l
47. Q AS = ç 4 - ÷ + ç 0 - ÷ = 2
è 25 ø è 25 ø 5
= -3 ´2 [Q l = 2]
2 2
æ 36 ö æ 48 ö 9 = -6
and BS = ç 0 - ÷ + ç3 - ÷ =
è 25 ø è 25 ø 5
50. Length of smallest focal chord = Length of latusrectum
If l be the semi-latusrectum, then l = HM of AS and BS 1
2 5 5 125 =
\ = + = 8l
l 16 9 144 1 1ù
é
288 = =4 êëQ l = 32 úû
Þ l = æ1ö
125 8´ç ÷
è 32 ø
576
\ 2l = 3
125 51. Let y - = Y , x + 2l = X
2
b 4
48. Q Slope of OS = = 1
a 3 \ Y2 = X
8l
3
\ Slope of directrix LM = - Curve is symmetrical about Y = 0
4
3
3 Þ y - =0
\ Equation of directrix is y = - x 2
4
3
Þ 3 x + 4y = 0. \ y =
2
Sol. (Q. Nos. 49 to 51)
Sol. (Q. Nos. 52 to 54)
Since, (a, b ) lies on parabola, y 2 = 4 lx
Given parabola ( P ) can be written as
\ b 2 = 4al ...(i) ( x - 2y ) 2 = 32 x - 4y - 16
It is clear that DPQR is right angled at P( 0, 2 ). On adding 2( x - 2y ) l + l2 both sides, it becomes
Y
(x - 2y + l)2 = 32 x - 4y - 16 + 2( x - 2y ) l + l2
Q P
= 2( l + 16 ) x - 4( l + 1 )y + l2 - 16 ...(i)
y=2
R We choose l such that lines
X′ X x - 2y + l = 0 and 2( l + 16 ) x - 4( l + 1 )y + l2 - 16 = 0
O
ax
+

are perpendicular to each other. It requires


by
=

1 2( l + 16 )
1

´ = -1
Y′ 2 4( l + 1 )
Þ l + 16 = - 4 l - 4
So, its circumcentre is the mid-point of Q and R, where
æ 1 - 2b ö æ 1ö Þ l=-4
Qºç , 2 ÷ and R º ç 0, ÷ . Hence, Eq. (i) becomes
è a ø è bø
æ 1 - 2b 1ö ( x - 2y - 4 ) 2 = 24 x + 12y = 12(2 x + y )
\ Circumcentre º ç ,1 + ÷ 2
è 2a 2b ø æ x - 2y - 4 ö 12 æ 2 x + y ö 2
Þ ç ÷ = ç ÷ Þ Y = 4rX
1 - 2b 1 è 5 ø 5 è 5 ø
\ x= ,y =1 +
2ab 2b 2x + y x - 2y - 4
where, X = ,Y =
1 (y - 2 ) 5 5
Þ 2b = and 2a = ...(ii)
y -1 x(y - 1 ) 12 3
and 4r = Þ r= .
5 5
From Eqs. (i) and (ii), we get 52. Q Equation of axis is Y = 0
x ...(i)
(y - 1 ) (y - 2 ) = Þ x - 2y - 4 = 0
8l and equation of latusrectum is X = r
2
æ 3ö 1 2x + y 3
Þ çy - ÷ = ( x + 2 l) Þ =
è 2 ø 8l 5 5
æ 3ö Þ 2x + y - 3 = 0 ...(ii)
\ Vertex is ç - 2 l, ÷
è 2ø On solving Eqs. (i) and (ii),
1 focus is ( x, y ) = (2, - 1 ).
and length of latusrectum is .
8l
442 Textbook of Coordinate Geometry

12 Sol. (Q. Nos. 58 to 60)


53. Length of latusrectum = 4r =
5 Q C1 : y 2 = 4 x
54. Equation of directrix is X + r = 0 \ S1 : (1, 0 )
2x + y 3 Let z = x + iy and z1 = x1 + iy1
Þ + =0
5 5 If z1 is obtained by rotating z, 120° in anti-clockwise direction,
\ 2 x +y + 3 = 0 then
Sol. (Q. Nos. 55 to 57) æ 1 i 3ö
z1 = ze 2pi /3 = ( x + iy ) ç - + ÷
Any parabola whose axis is parallel to the X -axis will be of the è 2 2 ø
form
æ x 3y ö æ 3 yö
(y - q ) 2 = 4l( x - p ) ...(i) =ç- - ÷ + iç x- ÷
è 2 2 ø è 2 2ø
Now, lx + my = 1 can be written as
\ Equation of curve C 2 is
l æ 1 - mq - lp ö
(y - q ) = - ( x - p ) + ç ÷ ...(ii) 2
m è m ø æ 3 yö æ x 3 ö
ç x - ÷ = 4ç - - y÷ ...(i)
Eq. (ii) will touch Eq. (i), then è 2 2ø è 2 2 ø
1 - mq - lp l x 3 3 y
= For focus - - y = 1 and x - = 0.
m l 2 2 2 2
-
m 1 3
\ x = - ,y = -
Þ - l + mlq + l 2p = m 2l 2 2
Þ pl 2 + qlm - lm 2 - l = 0 ...(iii) æ 1 3ö
Þ S2 : ç - , - ÷
But given that è 2 2 ø
5l 2 - 4lm + 6m 2 + 3l = 0 ...(iv) Q C 3 is reflection of C 2 with respect to y = x.
On comparing Eqs. (iii) and (iv), we get æ 3 1ö
\ S3 : ç - , - ÷.
p q - l -1 è 2 2ø
= = =
5 -4 6 3
58. Q(t 2, 2 t ) are parametric form of curve C1.
5 4
Þ p = - ,q = , l =2 \ From Eq. (i),
3 3
3 y x 3
So, the parabola is x - = 2t and - - y = t 2,
2 2 2 2 2
æ 4ö æ 5ö 1 1
çy - ÷ = 8ç x + ÷. we get x = ( - t 2 + 2 3t ), y = ( - 3 t 2 - 2t )
è 3ø è 3ø 2 2
æ 5 4ö \ Parametric coordinates of C 2 are
55. Q Vertex is ç - , ÷ .
è 3 3ø æ1 2 1 2 ö
ç ( - t + 2 3t ), ( - 3t - 2t ) ÷ .
5 4 è2 2 ø
Here, a = - and b =
3 3 59. Area of DOS 2S 3
\ a -b = -3
1 3
Þ |a -b | =3 1 -2 - 2
= | |
56. For focus, 2 3 1
5 4 - -
x+ = 2 and y - = 0 2 2
3 3 1 1 3 1
æ1 4ö = - = sq unit
\ Coordinates of focus are ç , ÷ . 2 4 4 4
è3 3 ø 2 2
2 2
1 4 æ 1ö æ 3ö æ 3ö æ 1ö
Here, c = and d = 60. Sx12 + Sy12 = (1) 2 + ç - ÷ + ç - 2
÷ + (0) + ç - ÷ + ç- ÷
è 2ø è 2 ø è 2 ø è 2ø
3 3
\ d -c =1 1 3 3 1
=1 + + +0 + + =3
Þ 2| d - c | = 2 4 4 4 4
æ 5ö Sol. (Q. Nos. 61 to 63)
57. For directrix ç x + ÷ + 2 = 0 Since, no point of the parabola y = x 2 + ax + 1 is below the
è 3ø
Þ 3 x + 11 = 0 given ex + f = 0 X -axis.
\ e =3, f = 11 \ D£0
Now, | e - f | = | 3 - 11 | = 8 Þ a2 - 4 £ 0
Þ -2 £a £2
Chap 05 Parabola 443

61. The maximum value of a is 2. Þ (1 + l)x 2 + (1 + 2 l)xy + (1 + l )y 2


2
The equation of the parabola is y = x + 2 x + 1. + 2( l - 1 ) x + 2( l - 1 )y + l + 1 = 0
2
It intersect the Y -axis at ( 0, 1 ). It will be a parabola, if h = ab
\Equation of tangent at ( 0, 1 ) is 1
Þ (1 + 2 l)2 = (1 + l)2
y +1 4
= 0 × x + (x + 0) + 1
2 Þ 1 + 4 l + 4 l2 = 4 + 8 l + 4 l2
Þ y = 2x + 1 3
Þ 2x - y + 1 = 0 \ l=-
4
Since, 2 x - y + 1 = 0 touches the circle x 2 + y 2 = c 2, then Hence, from Eq. (i), the required parabola is
0-0+1 3
=c [Qc > 0] ( x 2 + xy + y 2 - 2 x - 2y + 1 ) - ( x + y + 1 ) 2 = 0
4+1 4
2 2
1 Þ x - 2 xy + y - 14 x - 14y + 1 = 0 ...(ii)
Þ c=
5 64. Comparing parabola (ii) with
\ 5c 2 = 1 ax 2 + 2hxy + by 2 + 2 gx + 2 fy + c = 0
62. Equation of the tangent at ( 0, 1) to the parabola \ a = 1, h = -1, b = 1, g = - 7, f = - 7, c = 1
y = x 2 + ax + 1 is Now, | a + b + c + f + g + h | = | 1 + 1 + 1 - 7 - 7 - 1 | = 12
y +1 a 65. The parabola ( P ) can be written as
Þ = 0 × x + (x + 0) + 1
2 2 ( x - y ) 2 = (14 x + 14y - 1 )
Þ y = ax + 1
æx -yö
2 æ 14 x + 14y - 1 ö
Þ ax - y + 1 = 0 Þ ç ÷ = 7 2 çç ÷ ...(iii)
è 2 ø 2 2÷
As it touches the circle x 2 + y 2 = c 2, then è (14 ) + (14 ) ø
1
=c \Length of latusrectum is 7 2.
(a 2 + 1 )
66. For vertex,
i.e. c is maximum, when a = 0. x -y 14 x + 14y - 1
Therefore, the equation of the tangent is y = 1. = 0, =0 [from Eq. (iii)]
2 (14 ) 2 + (14 ) 2
\Slope of the tangent is 0.
Þ x = y , 14 x + 14y = 1
63. Equation of tangent is
1
y = ax + 1 \ x =y =
28
Þ ax - y = - 1
æ1 1ö
x y Vertex is ç , ÷ = (a, b ) [given]
Þ + =1 è 28 28 ø
æ 1 ö (1 )
ç- ÷ \ |a -b | = 0
è aø
Therefore, the area of the triangle bounded by the tangent and Sol. (Q. Nos. 67 to 69 )
the axes is Q y = 3 x is tangent to the parabola
1 1 1 2y = ax 2 + b ...(i)
- |1| =
2 a 2|a | 2
\ 2(3 x ) = ax + b
1
\ D= [for minimum area a = 2] [substitute the value of y = 3 x in 2y = ax 2 + b]
4
Þ 8D = 2 Þ ax 2 - 6 x + b = 0
D =0
Sol. (Q. Nos. 64 to 66) [Qy = 3 x is tangent to 2y = ax 2 + b]
The conic is S º x 2 + xy + y 2 - 2 x - 2y + 1 = 0 \ 36 - 4ab = 0
and the line is L º x + y + 1 = 0 Þ ab = 9 ...(ii)
It is required to find equation of the parabola ( P ) which From Eqs. (i) and (ii), we get
touches the conic S = 0 at those (two) points, where the line 9
2y = ax 2 + ...(iii)
L = 0 intersect the conic. Obviously at these points the a
parabola is in double contact with the conic. a+b
67. Q ³ ab = 3 [from Eq. (ii)]
\The equation of any such conic is f º S + lL2 = 0 2
Þ ( x 2 + xy + y 2 - 2 x - 2y + 1 ) + l(x + y + 1 ) 2 = 0 ...(i) Þ a + b ³6
\ Minimum value of a + b is 6.
444 Textbook of Coordinate Geometry

68. Q(2, 6) is the point of contact. Y

From Eq. (iii), we get L ( 1, 2)


9
12 = 4a +
a
Þ 4a 2 - 12a + 9 = 0 M
X′ X
2 (–1, 0) O (1, 0) N (3,0)
Þ (2a - 3 ) = 0
\ 2a = 3
69. For b = 18
1 L′ (1,–2)
From Eq. (ii), a= Y′
2
x2 The area of the square
From Eq. (iii), 2y = + 18 ...(iv) MLNL¢ is ( ML ) 2 = (1 + 1 ) 2 + (2 - 0 ) 2 = 8 sq units
2
On solving y = 3 x and Eq. (iv), we get \ l =8
x2 73. Parabola is y 2 = x
6x = + 18
2 1
\ a=
Þ x 2 - 12 x + 36 = 0 4
Þ (x - 6)2 = 0 The normal at t is
t3 t
\ x = 6, then y = 3 x = 18 tx + y = +
4 2
\ Point of contact is (6, 18 ).
It passes through (a, 0 ).
70. Equation of tangent in terms of slope (m ) of the parabola t3 t
1 \ ta = +
y 2 = 4 x is y = mx + . 4 2
m 1ö 1
2 æ
Q Point of intersection of tangents is ( - 2, - 1 ), then Þ t = 4 ça - ÷ Þa >
è 2ø 2
1
- 1 = - 2m + æ 1ö æ 1ö
m \ t1 = 2 ça - ÷ , t 2 = - 2 ça - ÷
è 2ø è 2ø
Þ 2m 2 - m - 1 = 0
The normals are perpendicular Þ t1t 2 = - 1
Y 1ö
æ
Þ - 4ça - ÷ = - 1
è 2ø
\ 4a = 3
X′ X
θ O æ 3 ö é 3ù
74. Let B as ç t 2, 3 t ÷ êë Here, a = 2 úû
(–2, –1) è2 ø
and let BD ^ AC and ÐDAB = q
Y′
Y 3 2
B 2 t , 3t
Let m1, m2 be slopes of the tangents, then
D (1 + 8 ) 3 θ
m1 - m2 = = =
a 2 2 X′ θ X
1 A D C
and m1m2 = -
2
m1 - m2 3/2
\ tan q = = =3 Y′
1 + m1m2 1 -1/2
71. Let S be the focus and point of intersection of tangents at P and BD 3 t 2
Q is R. Þ tan q =
= =
AD 3 t 2 t
Q (SR ) 2 = SP × SQ = 4 ´ 9 = 36 2
\ SR = 6 \ Projection of BC on the axis = DC
72. The four lines form a square. The tangents at L(1, 2) and æ2ö
= BD tan q = 3t ç ÷ = 6 units
L¢(1, - 2 ) are x - y + 1 = 0 and x + y + 1 = 0. They intersect at èt ø
M( - 1, 0 ). \ l =6
Chap 05 Parabola 445

75. The parabolas are y = x 2 - 9 and y = lx 2. 78. For maximum number of common chords, the circle and the
2 2 2 parabola must intersect at four points.
Þ x - 9 = lx Þ x (1 - l ) = 9
Now, solving the given curves, we have
9
Þ x2 = (x - 6)2 + 4x = r 2
1-l
3 Þ x 2 - 8 x + 36 - r 2 = 0
\ x=±
(1 - l ) The curves touch, if D=0
2
Now, from the symmetry about Y -axis, Þ 64 - 4 × 1 × (36 - r ) = 0
6 3 Þ r 2 = 20
AB = 2a = Þ a=
(1 - l ) (1 - l ) \ r =2 5
2 2 2
Þ a (1 - l ) = 9 Þ la + 9 = a Hence, the least integral value of r for which the curves
Þ 2
a +m =a 2
[given] intersect is 5.

\ m =9 79. Given parabola is


x 2 = 4 (y - 1 )
76. Given, y 2 - 8x < 0, x 2 + y 2 < 16 and x > 0.
For x = 1, \ Focus is (0, 2).
y 2 < 8 and y 2 < 15 Þy 2 < 8 Now, the shortest intercept of the line on the parabola which
passes through the focus is latusrectum. The axis of the given
\ y = 0, ± 1, ± 2 parabola is the Y -axis.
\ 5 points. Therefore, the latusrectum is parallel to the X -axis.
For x = 2 , \ Slope = 0
y 2 < 16, y 2 < 12 Þ y 2 < 12 80. (A) The given parabolas are symmetrical about the line y = x
\ y = 0, ± 1, ± 2, ± 3 as shown in the figure
\ 7 points Y
For x = 3,
y 2 < 24, y 2 < 7 Þy 2 < 7
\ y = 0, ± 1, ± 2 3
4
\ 5 points
1
Hence, total points is 17.
2
\ n = 17
Sum of digits of n is 8.
X′ X
77. Focus of the parabola y 2 = 4x is S(1, 0). O
Y
Y′

They intersect each other at four distinct points.


Hence, the number of common chords is
X 4 4 ×3
O S C2 = =6
C 1 ×2
Which is perfect number [Q1 ´ 2 ´ 3 = 1 + 2 + 3 = 6]

y
(B) Q tan q = ...(i)
Let radius of circle be r. x
\ Centre of circle is (1 + r , 0 ). Y
Þ Equation of circle is ( x - 1 - r ) 2 + y 2 = r 2 B(x,y)

Þ (x - 1 - r )2 + 4x = r 2 [Qy 2 = 4 x] θ 90°+θ
Þ x 2 + 2(2 - 1 - r ) x + 2r + 1 = 0 X′ θ X
A L C
2
Þ x + 2(1 - r ) x + 2r + 1 = 0
It would have same roots due to symmetry.
\ D=0
2 Y′
Þ 4(1 - r ) - 4 × 1 × (2r + 1 ) = 0
\ r = 0, 4 LC = Projection of BC on X -axis
Hence, r =4 [Qr ¹ 0] = BL tan q = y tan q
446 Textbook of Coordinate Geometry

y Also A, B lie on y - 2 x = 1
=y ´ [from Eq. (i)]
x \ y1 - 2 x1 = 1 and y 2 - 2 x 2 = 1
y2 or y 2 - y1 = 2( x 2 - x1 ) ...(iv)
=
x 2 2
\ Length of AB = ( x 2 - x1 ) + (y 2 - y1 )
=4 [Qy 2 = 4 x] = 5 | x1 - x 2 | [from Eq. (iv)]
(C) Normals to y 2 = 4ax and x 2 = 4by in terms of m are
= 2 5 (a 2 + a ) [from Eq. (iii)]
b
y = mx - 2am - am 3 and y = mx + 2b + 2 . 2
m Given, 2 5 (a + a ) = 40
For common normal, Þ a2 + a = 2
b
2b + 2 = - 2am - am 3 Þ a2 + a - 2 =0
m
or am 5 + 2am 3 + 2bm 2 + b = 0 Þ (a + 2 ) (a - 1 ) = 0
Þ a = - 2, 1
It is clear that at most five common normals.
(B) If tangents are drawn from ( x1, y1 ) to the parabola y 2 = 4ax
(D) Let middle point of P and B be (h, k ), then 2h = at 2
and angle between tangents is q, then
Y
at) (y12 - 4ax1 )
2 ,2 | tan q | =
at | x1 + a |
P(
(0,at) B 3p
Here, x1 = 0, y1 = 2, q = , then
X′ X 4
A S(a, 0) 2
| -1| =
|0+a|
Þ |a | =2
Y′ \ a=±2
Aliter :
and 2k = 3at
2 Observe that one tangent is the Y -axis, the other tangent is at
æ 2k ö p
or 2h = a ç ÷ q = and its equation is ty = x + at 2
è 3a ø 4
or 2k 2 = 9ah for t = 1, we get y = x + a
\Locus of mid-point is Þa = 2 from the symmetry a = - 2 is also possible.
2y 2 = 9ax (C) Let the other end be at (at 2, 2at )
9a æ at 2 + a 2at + 2a ö
\Length of latusrectum = So, mid-point is ç , ÷
2 è 2 2 ø
9
= ´2 [Qa = 2] which satisfy x + y = 1
2
=9 at 2 + a
or + at + a = 1
2
81. (A) Given parabola is x 2 = ay ...(i)
Þ at 2 + 2at + 3a - 2 = 0
and the given line is y - 2 x = 1 ...(ii)
Since, two distinct chords are possible, so D > 0.
Y x2=ay
\ 4a 2 - 4a(3a - 2 ) > 0
1 Þ - 8a 2 + 8a > 0
–2x=
B y
(x1,y1) (x2 , y2 )
Þ 8a(a - 1 ) < 0
A \ 0 <a <1
X′ X or 0 < 4a < 4
O
Y′ or 0 < Length of latusrectum < 4
\Length of latusrectum can be 1 or 2 or 3 from the given
On solving Eqs. (i) and (ii), we get values.
x 2 = a(2 x + 1 ) Þ x 2 - 2ax - a = 0 (D) The given parabola is
Let coordinates of A and B are ( x1, y1 ) and ( x 2, y 2 ) respectively, x 2 - ay + 3 = 0
then
æ 3ö
D ( 4a 2 + 4a ) or x 2 = a çy - ÷
| x1 - x 2 | = = = 2 (a 2 + a ) ...(iii) è aø
a 1
Chap 05 Parabola 447

Let x =X, y -
3
=Y 83. Equation of tangent to y 2 = 4x is
a 1
Then, the parabola is y = mx +
m
X 2 = aY it is also tangent to x 2 = 4y , then
a æ 1ö
For focus X = 0, Y = x 2 = 4 çmx + ÷
4 è mø
3 a
Þ x = 0, y - = 4
a 4 Þ x 2 - 4mx - = 0
m
æ 3 aö
\ Focus is ç 0, + ÷ given focus is ( 0, 2 ) It discriminant = 0
è a 4ø
16
3 a Þ 16m 2 + =0
\ + =2 m
a 4
Þ m3 = - 1
Þ a 2 - 8a + 12 = 0
\ m = -1
Þ (a - 6 ) (a - 2 ) = 0 \ Equation of common tangent is x + y + 1 = 0.
\ a = 2, 6 Statement II is also true and it is correct explanation of
Here, a1 = 6, a 2 = 2 Statement I.

\
a1
=3 84. The vertex of (y + 2) 2 = 2( x - 1) is (1, - 2) and equation of axis
a2 is y = - 2.
82. (A) Points (1, 2) and ( - 2 , 1) satisfy both the curves. Here, 4a = 2
2 2 1 3
(B) Equation of tangent at (t , 2t ) on y = 4 x is \ a= Þ 3a =
2 2
ty = x + t 2 æ 3 ö æ5 ö
\ Required point is ç1 + , - 2 ÷ , i.e. ç , - 2 ÷ .
It passes through the point (2, 3 ), then è 2 ø è2 ø
3t = 2 + t 2 Hence, both statements are true and Statement II is correct
2 explanation for Statement I.
Þ t - 3t + 2 = 0
85. Q y = mx + c …(i)
or (t - 1 ) (t - 2 ) = 0
or t = 1 or 2 and y 2 = 4ax …(ii)
The point of contact is (1, 2 ) or ( 4, 4 ). From Eqs. (i) and (ii),
(C) Let P( 5 cos q, 5 sin q), then the chord of contact of the æy - c ö
y 2 = 4a ç ÷
2 è m ø
parabola y = 4 x w.r.t. P is
Þ my 2 - 4ay + 4ac = 0
y × 5 sin q = 2( x + 5 cos q)
2x If line Eq. (i) intersect the parabola y 2 = 4ax at one point, then
or y = + 2 cot q
5 sin q B 2 = 4 AC
On comparing with y = 2( x - 2 ), then Þ ( - 4a ) 2 = 4 × m × 4ac
5 sin q = 1 and cot q = - 2 a
Þ c=
or 5 sin q = 1 and 5 cos q = - 2 m
From Eq. (i), equation of tangent is
Hence, coordinates of P are ( - 2, 1 ).
a
(D) Let coordinates of Q be (t 2, 2t ). y = mx +
m
Now, the area of DOPQ is \Statement I and Statement II are individual true and
1 t2 2t Statement II is correct explanation of Statement I.
| |=6 [given]
2 4 -4 86. Q ax + by = 1 …(i)

Þ 2
2t + 4t = ± 6 On squaring both sides, then
2 ax + by + 2 abxy = 1
or t + 2t ± 3 = 0
Þ (ax + by - 1 ) = - 2 abxy
\ t 2 + 2t - 3 = 0 [Qt 2 + 2t + 3 ¹ 0]
Again, on squaring both sides, we get
Þ (t + 3 ) (t - 1 ) = 0
a 2x 2 + b 2y 2 + 1 + 2abxy - 2ax - 2by + 1 = 4abxy
Then, t = 1 or - 3
Hence, the point Q are (1, 2 ) or (9, - 6 ). Þ a 2x 2 - 2abxy + b 2y 2 - 2ax - 2by + 1 = 0
448 Textbook of Coordinate Geometry

Now, comparing with From Eq. (i)


Ax 2 + 2 Hxy + By 2 + 2Gx + 2 Fy + C = 0 PQ = a(cot ( a / 2 ) + tan( a / 2 )) 2
\ A = a 2, H = - ab, B = b 2, G = - a, F = - b, C = 1 æ 1 ö
2
=aç ÷
\ D = ABC + 2 FGH - AF 2 - BG 2 - CH 2 è sin( a / 2 ) cos( a / 2 ) ø
= a 2b 2 - 2a 2b 2 - a 2b 2 - a 2b 2 - a 2b 2 æ 2 ö
2
2
=aç ÷ = 4a cosec a
2 2
= - 4a b ¹ 0 è sin a ø
2
and H = AB For a = 60°, 4a = 8 and
2
Hence, Eq. (i) represent a parabola. æ 2 ö
PQ = 8 cosec 2 60° = 8 ç ÷ = 32 / 3
\ Statement I is true and Statement II is false. è 3ø
2at - 0 2 \ Statement I is true and Statement II is false.
87. Q Slope of AP = m1 = 21 =
at1 - 0 t1 89. Q x + y = l Þy = l - x …(i)
2at 2 - 0 2 and y =x-x 2
…(ii)
and slope of AQ = m2 = =
at 22 - 0 t 2 From Eqs. (i) and (ii), we get
4 l - x = x - x2
Q m1m2 = ¹ -1
t1t 2
Þ x 2 - 2x + l = 0
Y
2 2at 1) Q Eq. (i) touch the parabola Eq. (ii), then
P(at 1,
B 2 - 4 AC = 0
Þ (- 2)2 - 4 × 1 × l = 0
X′ X
A S(a,0) \ l =1
Þ Statement I is true.
Q (a t 2 From Statement II,
2 , 2at
2)
(x - 1)2 = x - x 2
Y′
Þ 2x 2 - 3x + 1 = 0
Also, P , S , Q are collinear, then
2at1 - 0 0 - 2at 2 \ Discriminant = ( - 3 ) 2 - 4 × 2 × 1 = 1 ¹ 0
=
at12 - a a - at 22 \ Statement II is false.
Þ t1 - t1t 22 = - t12t 2 + t 2 90. Q 3x + 4y + 5 = 0 and 4x + 3y + 2 = 0 are not perpendicular
Þ (t1 - t 2 ) + t1t 2(t1 - t 2 ) = 0 to each other.
Q t1 - t 2 ¹ 0, 1 + t1t 2 = 0 \Latusrectum ¹ 4
\ t1t 2 = - 1 \Statement I is false and Statement II is true.
Hence, Statement I is false and Statement II is true. 91. Let P (at 2, 2at ) be any point on the parabola y 2 = 4ax .

2 Then, tangent at P (at 2, 2at ) is ty = x + at 2.
æ
88. Length of focal chord PQ = a çt + ÷ …(i) Since, tangent meet the axis of parabola in T and tangent at the
è tø
vertex in Y
2a æ1ö
2at + 2ç ÷ Y
t 2t èt ø
and tan a = = 2 = 2 P
2 a t - 1 æ1ö 2
at - 2 1-ç ÷ G (at, 2at)
t èt ø
X′ X
1 T A
\ = tan( a / 2 )
t
Y Y′
22at)
P(at ,
\ Coordinates of T and Y are ( - at 2, 0 ) and ( 0, at ) respectively.
α Let coordinates of G be ( x1, y1 ).
X′ X Since, TAYG is rectangle.
A S(a,0)
Q Mid-points of diagonals TY and GA is same.
Q a , –2 a x1 + 0 -at 2 + 0
t 2, Þ =
t 2 2
Y′
Þ x1 = - at 2 …(i)
Chap 05 Parabola 449

y1 + 0 0 + at 94. The centre and radius of the given circle are (0, 12) and 4,
and + Þ y1 = at …(ii)
2 2 respectively.
Eliminating t from Eqs. (i) and (ii), we get Now, the shortest distance always occurs along the common
2 normal.
æy ö
x1 = - a ç 1 ÷ Y
èaø (0, 12)
2 C P
Þ y1 = - ax1
Þ y12 + ax1 = 0 A

Hence, the locus of G( x1, y1 ) is y 2 + ax = 0. X′ X


O
92. Equation of incident ray parallel to axis of parabola (X -axis) is
y = l, which pass through ( - 1, 2 ), then 2 = l.
\ Equation of incident ray is y = 2 Y′
2
Incident ray strikes the parabola y = 4 x at (1, 2). Let A º (t 2, 2t )
Y Equation of normal at A is y + tx = 2t + t 3,
(–1,2) which passes through (0, 12), then
(1,2) 12 + 0 = 2t + t 3 Þ t 3 + 2t - 12 = 0
2
X′ X
or (t - 2 ) (t + 2t + 6 ) = 0
S(1,0) \ t =2 [Qt 2 + 2t + 6 ¹ 0]
Coordinates of A are (4, 4).
\ Shortest distance = AP = AC - CP = 80 - 4 = 4( 5 - 1 )
Y′
95. Let P (t12, 2 t1 ) and Q (t 22, 2 t 2 ) be extremities of the chord with
The reflected ray passes through the focus (1, 0). slope 2.
Hence, the equation of the reflected ray is x = 1. 2 t1 - 2 t 2
\ =2
t12 - t 22
93. Let PQ be a normal chord to a parabola at P (at 2, 2at ).
Since, the ordinate and abscissa of P are equal. Þ t1 + t 2 = 1 ...(i)
2 Y
Þ at = 2at, t ¹ 0 2
(t1, 2t1)
\ t =2 P
2 1
(at, 2 a t) R (h, k)
P X′ X
A 2
90° Q2
X′ X (t2, 2t2)
A S
Y′
Q Let R (h, k ) be coordinates of the point which divides PQ in the
2
(at1 , 2at1) ratio 1 : 2, then
2 t 2 + t 22
h= 1
Since, normal at P (at 2, 2at ) meet the parabola at Q (at12, 2at1 ). 3
2 4 t1 + 2 t 2
\ t1 = - t - and k=
t 3
or t1 = - 2 - 1 Þ 3h = 2 t12 + (1 - t1 ) 2
or t1 = - 3
and 3k = 4t1 + 2 (1 - t1 ) [from Eq. (i)]
\ Coordinates of P and Q are ( 4a , 4a ) and (9a , - 6a ), respectively.
or 3h = 3 t12 - 2 t1 + 1
4a - 0 4
\ Slope of SP = = = m1 (say)
and
4a - a 3 3k = 2 t1 + 2
-6a - 0 3 Eliminating t1
and slope of SQ = = - = m2 (say) 2
9a - a 4 æ 3k - 2 ö æ 3k - 2 ö
\ 3h = 3 ç ÷ - 2ç ÷ +1
è 2 ø è 2 ø
\ m1m2 = - 1
Hence, SP and SQ are perpendicular to each other Þ 9k 2 - 16k - 4h + 8 = 0
i.e. Ð PSQ = 90° 16k 4h 8
Þ k2 - - + =0
9 9 9
450 Textbook of Coordinate Geometry

2
æ 8ö 4æ 2ö Aliter :
Þ çk - ÷ = çh - ÷
è 9ø 9 è 9ø Let point of intersection of tangents at P and Q is T (h, k ) then
æ 8ö 4æ 2ö
2 equation of pair of tangents TP and TQ is
\ Locus of R (h, k ) is çy - ÷ = ç x - ÷ ,
è 9 ø 9 è 9ø SS1 = T 2
æ8 2ö Þ (y 2 - 4ax )(k 2 - 4ah ) = [ky - 2a( x + h )]2 …(i)
which is a parabola with vertex ç , ÷ .
è9 9ø
Let the pair of tangents cut the directrix x + a = 0 are in R and
96. Let coordinates of P and Q on the parabola y 2 = 4ax are S, then substitute x = - a in Eq. (i), then we get
(at12, 2at1 ) and (at 22, 2at 2 ).
hy 2 - k(h - a )y + a {(h + a ) 2 - k 2 } = 0
Equation of tangents at P and Q are
Now let coordinates of R and S be
t1y = x + at12 and t 2y = x + at 22
( -a , y1 ) and ( -a , y 2 ).
Let these tangents meet x + a = 0 at R and S, then coordinates k(h - a )
æ a(t 2 - 1 ) ö æ a(t 2 - 1 ) ö \ y1 + y 2 =
of R and S ¢ are ç - a, 1 ÷ and ç - a, 2 ÷, h
è t1 ø è t2 ø
a {(h + a ) 2 - k 2 }
respectively. and y1y 2 =
h
Y but given | RS | = d
P Þ ( RS ) 2 = d 2
R Þ (y1 - y 2 ) 2 = d 2
X′ d X
A S (a, 0)
T S′ Þ (y1 + y 2 ) 2 - 4y1y 2 = d 2
Q k2 4a
Þ (h - a ) 2 - {(h + a ) 2 - k 2 } = d 2
Y′ h2 h
x+a=0
Þ k 2 {(h - a ) 2 + 4ah } - 4ah(h + a ) 2 = d 2h 2
Given, | RS ¢| = d
Þ (k 2 - 4ah )(h + a ) 2 = d 2h 2
½ æ t12 - 1 ö æ t 2 - 1 ö½
\ ½a ç ÷ - aç 2 ÷½ = d Hence, locus of T (h, k ) is
½ è t1 ø è t 2 ø½
(y 2 - 4ax )( x + a ) 2 = d 2x 2.
½ a
Þ ½ t12t 2 - t 2 - t1t 22 + t1½
½= d 97. Let P (at12, 2at1 ) and Q(at 22, 2at 2 ) be two points on the parabola
½ 1t 2
t ½
y 2 = 4ax
½ a
Þ ½ ((t1 - t 2 )(1 + t1t 2 ))½
½= d The equations of tangents at P and Q are
½ 1t 2
t ½
t1y = x + at12
½a {(t1 + t 2 ) 2 - 4t1t 2 } (1 + t1t 2 )½
Þ ½ ½= d and t 2y = x + at 22
t1t 2
½ ½ 1 1
\ tan q1 = , tan q2 =
Let the point of intersection of tangents at P at Q is T then t1 t2
T = (at1t 2, a(t1 + t 2 )) Equations of circles with OP and OQ as diameters are
Now, let T = (h, k ) ( x - 0 )( x - at12 ) + (y - 0 )(y - 2at1 ) = 0
\ h = at1t 2 and k = a(t1 + t 2 )
and ( x - 0 )( x - at 22 ) + (y - 0 )(y - 2at 2 ) = 0 respectively.
h k
\ t1t 2 = and t1 + t 2 = …(ii) i.e. x 2 + y 2 - axt12 - 2at1y = 0
a a
From Eqs. (i) and (ii), we get and x 2 + y 2 - axt 22 - 2at 2y = 0
½ æ k 2 4h ö æ h ö½ then, point of intersection of circles are O (0, 0)
½a ç 2 - ÷ çè1 + ÷ø½
½ èa a ø a ½ æ -4at1t 2 2at t (t + t ) ö
=d and Rç 2
, 1 2 12 2 ÷
½ h ½ è (t1 + t 2 ) + 4 (t1 + t 2 ) + 4 ø
½ a ½
½ ½ Since, OR makes an angle f with the X-axis. Therefore,
2 2 2 2 æ t + t2 ö
Þ (k - 4ah )(h + a ) = h d tan f = - ç 1 ÷
è 2 ø
Hence, required locus is
Now, cot q1 + cot q2 + 2 tan f = t1 + t 2 - (t1 + t 2 )
(y 2 - 4ax )( x + a ) 2 = d 2x 2.
=0
Chap 05 Parabola 451

98. Any normal of the parabola y 2 = 4x with slope m is Eqs. (i) and (ii) both pass through ( x1, y1 ) which is the centre of
3 the circle
y = mx - 2m - m
ty1 = x1 + at 2 …(iii)
m 1)
Y 2 , –2a and 2 tx1 + 4y1 = at + 4at 3
…(iv)
(a m 1
A Multiplying Eq. (iii) by t and subtracting Eq. (iv), we have
t 2y1 + t( 4a - 3 x1 ) - 4y1 = 0 …(v)
X′ X 2
O Also, from Eq. (iii), at - ty1 + x1 = 0 …(vi)
(am22 , –2am2) B P (h, k)
Eliminating t from Eqs. (v) and (vi), we get
C t2 t 1
= =
Y′ (am32 , –2am2) x1( 4a - 3 x1 ) - 4y12 -4ay1 - x1y1 -y12 - a( 4a - 3 x1 )

It passes through P, then On simplyfying, we get

k = mh - 2m - m 3 2y12(2y12 + x12 - 12ax1 ) = ax1(3 x1 - 4a ) 2


Hence, required locus is
Þm 3 + (2 - h )m + k = 0 …(i)
2y 2(2y 2 + x 2 - 12ax ) = ax(3 x - 4a ) 2.
Thus, m1m2m3 = - k
am3 = - k (Q m1m2 = a ) 100. Let P (at12, 2at1 ) and Q (at 22, 2at 2 ) be two points on the parabola
k y 2 = 4ax such that the normals at P and Q intersect at a point
Þ m3 = -
a R (aT 2, 2aT ) on the parabola y 2 = 4ax, then
Q m3 is a root of Eq. (i), then 2 2
T = - t1 - = - t 2 - , then t1t 2 = 2
k3 æ kö t1 t2
- 3 + (2 - h ) ç - ÷ + k = 0
a è aø
Q Tangents at P and Q intersect at T (at1t 2, a (t1 + t 2 ))
Þ k 3 + (2 - h )ka 2 - ka 3 = 0 i.e. T (2a , a (t1 + t 2 )).
\ Locus of P (h, k ) is Y R
y 3 + (2 - x )ya 2 - ya 3 = 0
Þ y 2 + (2 - x )a 2 - a 3 = 0 [Q y ¹ 0 ]
(P does not lie on the axis of the parabola) X
O
2 2 2 3
Þ y = a x - 2a + a
P
If it is a part of the parabola y 2 = 4 x T
Q
then, a2 = 4
Also, the coordinates of R being the point of intersection of
and -2 a 2 + a 3 = 0
normals at P and Q are
Þ a 2( a - 2 ) = 0 (2a + a (t12 + t 22 + t1t 2 ), - at1t 2 (t1 + t 2 ))
Þ a - 2 = 0, a ¹ 0 Þ ( 4a + a (t12 + t 22 ), - 2a (t1 + t 2 ))
\ a =2
Now, Ð TPR = Ð TQR = 90°
99. Let P (at 2, 2at ) be any point on y 2 = 4ax. Then, vertex A( 0,0).
Þ Ð TPR + Ð TQR = 180°
The equation of tangent at P is
Þ Quadrilateral PTQR is a cyclic quadrilateral, therefore centre
ty = x + at 2 …(i) of the circle circumscribing the DTPQ is at the mid-point of TR.
Tangent at P will be normal to the circle, AP is a chord whose Let its coordinates be (h, k ), then
æ at 2 ö 2 2h = 2a + 4a + a (t12 + t 22 )
mid-point is ç , at ÷ and slope is .
è 2 ø t 2h - 6a
Þ = (t1 + t 2 ) 2 - 2 t1t 2
\ Equation of the line passing through mid-point of AP and a
perpendicular to AP is 2h - 2a
Þ = (t1 + t 2 ) 2 [Q t1t 2 = 2 ] ...(i)
t æ at 2 ö a
y - at = - ç x - ÷
2è 2 ø and 2k = a (t1 + t 2 ) - 2 a (t1 + t 2 )
3 2k
at Þ - = (t1 + t 2 ) ...(ii)
Þ tx + 2y = + 2at …(ii) a
2
452 Textbook of Coordinate Geometry

2
æ 2h - 2a ö æ 2k ö 102. Equation of normal at (am 2, - 2am ) is
From Eqs. (i) and (ii), then ç ÷ = ç- ÷
è a ø è aø
y = mx - 2am - am 3
Þ 2k 2 = a (h - a )
If the three normals of P , Q, R meet at (h, k ), then
Hence, locus of (h, k ) is 2y 2 = a ( x - a ). am 3 + m(2a - h ) + k = 0
101. Let the equation of straight line (h, k ) as its mid-point, \ Sm1 = 0
x -h y -k (2a - h )
then, = =r …(i) Sm1m2 = ,
cos q sin q a
Any point on the line Eq. (i) is (h + r cos q , k + r sin q). -k
m1m2m3 =
Y θ–π/4 a
P º (am12, - 2am1 )
r
(h,k) π/ θ Q º (am22, - 2am2 )
4
π/4 X
O
–r and R º (am32, - 2am3 )
π/4 Equation of PQ is
-y (m1 + m2 ) = 2( x + am1m2 ) …(i)
and equation of diameter through R is
Solving with the equation of parabola
y = - 2am3 …(ii)
y 2 = 4ax, we get
Point of intersection of Eqs. (i) and (ii) is on the directrix and
(k + r sin q) 2 = 4a(h + r cos q) hence it must be ( - a , - 2am3 ) and it satisfies Eq. (ii), then
Þ r 2 sin 2 q + 2r (k sin q - 2a cos q) + k 2 - 4ah = 0 …(ii) 2am3(m1 + m2 ) = 2 ( - a + am1m2 )
which is quadratic in r. Þ m3 ( -m3 ) = - 1 + m1m2
The roots of the quadratic equation will be equal but of Þ m1m2 = 1 - m32
opposite sign as (h, k ) is the mid-point. \ Equation of PQ becomes
2(k sin q - 2a cos q)
Sum of roots = - =0 -y ( 0 - m3 ) = 2 ( x + a - am32 )
sin 2 q
Þ 2am32 + m3y - 2 ( x + a ) = 0 [m3 is parameter]
\ k sin q - 2a cos q = 0
2a Its envelope is given by the discriminant of this quadratic
\ tan q = equated to zero.
k
Now, from Eq. (ii), \ (y ) 2 - 4 × 2a × { -2 × ( x + a )} = 0
r 2 sin 2 q + (k 2 - 4ah ) = 0 Þ y 2 + 16a ( x + a ) = 0
4a 2 103. Equation of normal at ‘t’ is y = - tx + 2at + at 3.
Þ r2 . + (k 2 - 4ah ) = 0 …(iii)
( 4a + k 2 )
2
Let A be (h, k ), then k = - th + 2at + at 3
Length of the chord will be 2r. Angle between the two lines or at 3 - t(h - 2a ) - k = 0 …(i)
will be ( q - p / 4 ) and the projection of the chord on the given
line will be 2r cos( q - p / 4 ) = c Let the coordinates of P , Q, R are (at12, 2at1 ), (at 22, 2at 2 ), (at 32, 2at 3 )
2r respectively, then from Eq. (i),
Þ (cos q + sin q) = c
2 Y
2r æç k + 2a ö÷ M
P (t1)
Þ =c
2 çè 4a 2 + k 2 ÷ø x+a=0
A (h,k)
Þ 2r 2(k + 2a ) 2 = c 2( 4a 2 + k 2 ) X
(t2)Q S
2r 2 c2
Þ = …(iv)
( 4a 2 + k 2 ) (k + 2a ) 2 (t3)R
From Eqs. (iii) and (iv), we get
2a 2c 2 t1 + t 2 + t 3 = 0
+ (k 2 - 4ah ) = 0
(k + 2a ) 2 æ h - 2a ö
Þ t1t 2 + t 2t 3 + t 3t1 = - ç ÷
Þ 2 2
(k - 4ah ) (k + 2a ) + 2a c = 0 2 2 è a ø
Hence, the locus of the middle points is k
and t1t 2t 3 =
2 2
(y - 4ax ) (y + 2a ) + 2a c = 0. 2 2 a
Chap 05 Parabola 453

Since, SP = PM = a + at12 æ a aö
Hence coordinates of S are ç - , ÷ .
Similarly, SQ = a + at 22 è 2 2ø

and SR = a + at 32 \ RS ¢ = a

\ SP × SQ × SR = a(1 + t12 ) × a(1 + t 22 ) × a(1 + t 32 ) Hence, quadrilateral PQRS ¢ is trapezium whose area
1 æ aö
= a 3 {1 + (t12 + t 22 + t 32 ) + (t12t 22 + t 22t 32 + t 32 t12 ) + (t12 t 22 t 32 )} = ( PQ + RS ¢ ) ´ ça + ÷
2 è 2ø
= a 3 {1 + (t1 + t 2 + t 3 ) 2 - 2(t1t 2 + t 2t 3 + t 3t1 )
1 3a 15a 2
+ (t1t 2 + t 2t 3 + t 3t1 ) 2 - 2 t1t 2t 3(t1 + t 2 + t 3 ) + (t1t 2t 3 ) 2 } = ´ ( 4a + a ) ´ = sq units.
2 2 4
ìï 2
æ h - 2a ö æ h - 2a ö k 2 üï Aliter :
= a 3 í1 + 0 + 2 ç ÷+ç ÷ - 0 + 2ý
ïî è a ø è a ø a ïþ Here, centre of the circle is the vertex of the parabola and both
2 circle and parabola are symmetrical about axis of parabola. In
ìï æ h - 2a ö k 2 üï
= a 3í ç + 1÷ + 2 ý this case the point of intersection of common tangents must lie
ïî è a ø a ïþ on the directrix and axis of the parabola.
i.e. A( -a , 0 ).
= a { (h - a ) 2 + (k - 0 ) 2 } = a(SA ) 2
Chord of contact of circle w.r.t. A( -a , 0 ) is
104. Equation of tangent at P (at 2, 2at ) is
a2
2 2 x( -a ) + y × 0 =
ty = x + at Þ x - ty + at = 0 …(i) 2
which is also tangent to the circle a
\ x=-
x2 + y 2 = a2 / 2 …(ii) 2
then, length of perpendicular from centre of Eq. (ii) to Eq. (i) æ a aö
\ Coordinates of R are ç - , ÷ and chord of contact of
radius of the circle è 2 2ø
| at 2 | a parabola w.r.t. A( -a, 0 ) is
Þ = y × 0 = 2a( x - a )
2 2
1+t
i.e. x =a
Þ 2 t 4 = (1 + t 2 ) \ Coordinates of P is (a, 2a )
Þ (t 2 - 1 )(2 t 2 + 1 ) = 0 \ Area of quadrilateral
\ 2t 2 + 1 ¹ 0 \ t 2 - 1 = 0 PQRS ¢ = 2 {Area of DPAS - Area of DRAN }
then, t=±1 ì1 1 a aü
= 2 í × 2a × 2a - × × ý
then, coordinates of P and Q are (a , 2a ) and (a , - 2a ) î2 2 2 2þ
respectively. a 2 15a 2
= 4a 2 - = sq units
P 4 4
R 105. Let parabola be y 2 = 4ax and let P (at12, 2at1 ), Q (at 22, 2at 2 ) and
R (at 32, 2at 3 ) are three points on the parabola.
A NO S
(a, 0) Q Tangents at P , Q and R on parabola y 2 = 4ax
S
Q are t1y = x + at12, t 2y = x + at 22

\ PQ = 4a and t 3y = x + at 32
\ Equation of tangent at P (a , 2a ) is 1 1 1
Slopes of these tangents are , ,
x -y + a = 0 …(iii) t1 t 2 t 3
[from Eq. (i)] but given slopes are in HP.
Let R be ( x1, y1 ) \ t1, t 2, t 3 are in AP if d is the common difference. Then,
then, equation of tangent at R( x1, y1 ) on Eq. (ii) is t 2 - t1 = d , t 3 - t 2 = d
a2 and t 3 - t1 = 2 d ...(i)
xx1 + yy1 = …(iv)
2 Let the tangents at Q and R meet at P ¢, R and P meet at Q¢, P
Hence, Eqs. (iii) and (iv) are identical. and Q meet at R ¢.
x1 y1 a \ P ¢ = {at 2t 3, a (t 2 + t 3 )}
\ = =-
1 -1 2 Q ¢ = {at 3t1, a (t 3 + t1 )}
æ a aö and R ¢ = {at1t 2, a (t1 + t 2 )}
\ ( x1, y1 ) = ç - , ÷
è 2 2ø
454 Textbook of Coordinate Geometry

at 2t 3 a (t 2 + t 3 ) 1 108. Coordinates of S are (2 2 cos 45°, 2 2 sin 45°) i.e. (2, 2).
1
\ Area of DP ¢Q ¢ R ¢ = at 3t1 a (t 3 + t1 ) 1
2 Y
at1t 2 a (t1 + t 2 ) 1
P (x, y)
1
= {at 2t 3 {a (t 3 + t1 ) - (t1 + t 2 )} + ... + ... } S (2, 2)
2 √2
1 √2 A
= a 2S t 2t 3 (t 3 - t 2 ) M 45°
2 X′ X
O
1 x+
= a 2 (t1 - t 2 ) (t 2 - t 3 ) (t 3 - t1 ) y=

Di
2

re
0

ct
1

r ix
= a 2 ( -d ) ( -d ) (2d ) [by using Eq. (i)]
2
Y′
= a 2d 3, which is constant.
\ SP = PM
Remark Þ (SP ) 2 = ( PM ) 2
St12 ( t2 - t3 ) = S t2t3 ( t3 - t2 ) = ( t1 - t2 ) ( t2 - t3 ) ( t3 - t1 ) 2
é (x + y ) ù
Þ ( x - 2 ) 2 + (y - 2 ) 2 = ê
Corollary Area of triangle of P( at12, 2at1 ), Q( at22, 2at2 ) and ë 2 úû
R( at32, 2at3 ) is a2( t1 - t2 )( t2 - t3 )( t3 - t1 ) = D (say)
Þ 2( x 2 + y 2 - 4 x - 4y + 8 ) = x 2 + y 2 + 2 xy
It is clear from just previous example the area of the triangle
inscribed in a parabola is twice the area of the triangle Þ x 2 + y 2 - 2 xy - 8 x - 8y + 16 = 0
formed by the tangents at the vertices. \ ( x - y ) 2 = 8( x + y - 2 )
2
106. Equation of tangent at (1, 7) to y = x + 6 109. Equation of tangent to y = x 2 is
1 1
Þ (y + 7 ) = x × 1 + 6 y = mx - m 2 …(i)
2 4
Þ y = 2x + 5 …(i)
Equation of tangent to ( x - 2 ) 2 = - y is
This tangent also touches the circle.
1 2
x 2 + y 2 + 16 x + 12y + c = 0 …(ii) y = m( x - 2 ) + m …(ii)
4
Now, solving Eqs. (i) and (ii), we get Q Eqs. (i) and (ii) are identical.
x 2 + (2 x + 5 ) 2 + 16 x + 12(2 x + 5 ) + c = 0 Þ m = 0 or 4
Þ 5 x 2 + 60 x + 85 + c = 0 \ Common tangents are y = 0 and y = 4 x - 4 = 4 ( x - 1 ).
Since, roots are equal, so 110. Given parabola is
B 2 - 4 AC = 0 a 3x 2 a 2x
y = + - 2a …(i)
2 3 2
Þ (60 ) - 4 ´ 5 ´ (85 + c ) = 0
Þ 85 + c = 180 dy 3
For vertex =0 Þ x=-
Þ 5 x 2 + 60 x + 180 = 0 dx 4a
60 3
Þ x=- = -6 Þ y = -7 Substitute x = - in Eq. (i), we get
10 4a
Hence, point of contact is ( -6, - 7 ). 35a
y =-
107. Q P º (1,0), let Q º (h, k ) 16
æ 3 35a ö
such that k 2 = 8h …(i) \ Coordinates of vertex are ç - , - ÷.
è 4a 16 ø
Let ( a, b ) be the mid-point of PQ.
3 35a
h+1 k+0 For locus let x = - and y = - .
\ a= ,b = 4a 16
2 2
105
Þ h = 2 a - 1, k = 2b \ xy = , which is the required locus.
64
From Eq. (i), we get
(2b ) 2 = 8(2 a - 1 ) 111. Q y = x 2 - 5x + 6

Þ b2 = 4a - 2 \ Equation of tangent at (2, 0) is


y +0 5
Þ b2 - 4a + 2 = 0 = x × 2 - (x + 2) + 6
2 2
\ Required locus is y 2 - 4 x + 2 = 0. Þ y =-x+2 …(i)
Chap 05 Parabola 455

and equation of tangent at (3, 0) is 115. The circle and the parabola touch each other at x = 1, i.e. at the
Þ y = x -3 …(ii) points (1, 2) and (1, - 2 ) as shown in figure.
Q Eqs. (i) and (ii) are perpendicular. (1,2)
\ Angle between tangents is p / 2.
(3,0)
112. (i) Coordinates of P and Q are (1, 2 2 ) and (1, - 2 2 ). X′ X
(1,0) 2 2
Y √

(1, –2)

P
116. Vertex is (1,0).
(1,2 √2 ) Y
,0)

S R x=2
(1

X′ X
(–3,0) (–1, O
0 ) (9,0) (0, 0) (1, 0)
X
Q S A (2, 0)
(1,–2√2 )

Y′ Y′
1 117. G º (h, k )
Area of DPQR = × 4 2 × 8 = 16 2
2 2a + at 2 2at
1 Þ h= ,k =
Area of DPQS = × 4 2 × 2 = 4 2 3 3
2 Y 2)
2at
\ Ratio of area of DPQS and DPQR is 1 : 4. P(at,
(ii) Equation of circumcircle of DPRS is
( x + 1 ) ( x - 9 ) + y 2 + ly = 0 X
T(–at2,0) (0,0) N(2a +at2, 0)
It will pass through (1, 2 2 ), then
8
- 16 + 8 + l 2 2 = 0 Þ l = =2 2
2 2
2
Equation of circumcircle is æ 3h - 2a ö 9k
Þ ç ÷= 2
x 2 + y 2 - 8 x + 2 2y - 9 = 0 è a ø 4a
Hence, radius is 3 3. \ Required parabola is
Aliter : 9y 2 (3 x - 2a ) 3 æ 2a ö
2
= = çx - ÷
4a a a è 3ø
2 2
Let ÐPSR = q Þ sin q = 4a æ 2a ö
2 3 Þ y2 = çx - ÷
3 è 3ø
Þ PR = 6 2 = 2 R × sin q Þ R = 3 3.
æ 2a ö
D \ Vertex º ç , 0 ÷ ; Focus º (a, 0 ).
(iii) Radius of incircle is r = . è3 ø
s
2t 2 - 2t1 2
As D = 16 2 118. Slope of AB = = …(i)
(t 22 - t12 ) (t 2 + t1 )
6 2+6 2+4 2
\ s= =8 2 Y
2 2
B(t2,2t2)
(t 1, 2t 1)
2
16 2
\ r= =2 r M
8 2
X
x2 3 1
113. y = - + x + 1 Þ y - = - (x - 1)2
2 2 2
Þ It is symmetric about x = 1.
Hence, both statement are true and Statement II is correct
explanation of Statement I. æ t 2 + t 22 ö
M = Mid-point of AB = ç 1 , t1 + t 2 ÷
114. Q Point of intersection of two perpendicular tangents to the è 2 ø
parabola lies on directrix of the parabola. \ r = | t1 + t 2 | Þ t1 + t 2 = ± r
\ Equation of directrix is x + 2 = 0. Now, from Eq. (i),
So, point is ( - 2, 0 ). 2
slope of AB = ± .
r
456 Textbook of Coordinate Geometry

119. The locus of perpendicular tangent is directrix, i.e., x + 1 = 0 or 123. The shortest distance between y = x - 1 and y 2 = x is along the
x = - 1. normal of y 2 = x.
120. D1 = Area of DPLL¢ Y
1 æ 1ö
= ´ 8 ´ ç2 - ÷ = 6 sq unitss Q
2 è 2ø
1
Y x= P(t2, t)
y–
A L(2,4) X′ X
P O
1,
P 2
2
X′ X
C O
B

L′(2,–4) Y′
Y′

Let P (t 2, t ) be any point on y 2 = x.


Now, equation of AB is y = 2 x + 1,
x t
equation of AC is y = x + 2 and \ Tangent at P is y = + .
2t 2
equation of BC is y = - x - 2
1
On solving above equations, we get \ Slope of tangent =
2t
A(1, 3 ), B ( - 1, - 1 ) and C ( - 2, 0 )
and tangent at P is parallel to y - x = 1
1 1+2 3-0
\ D2 = = 3 sq units 1 1 æ 1 1ö
2 -1 + 2 -1 - 0 \ =1 Þ t = Þ Pç , ÷
2t 2 è 4 2ø
D1
\ =2 1 1
D2 - -1
2 4 3 3 2
121. Let A( x, y ) = A(t 2, 2t ) be any point on the parabola y 2 = 4x, then Hence, shortest distance = PQ = = =
(1 + 1 ) 4 2 8
t2
x= …(i) 124. We observe that both parabola y 2 = 8x and circle
4
x 2 + y 2 - 2 x - 4y = 0 pass through origin say P( 0, 0 ).
2t
and y = …(ii) Y
4

Y A(t2, 2t) Q(2,4)

3 )
1 : (x, y (1,2)
P
X′ X 90°
O X′ X
(0,0)P S (2,0)
Y′
Y′

From Eqs. (i) and (ii), we get


x =y2
Let Q be the point (2 t 2, 4 t ), then it will satisfy the equation of
122. The equation of normal to
circle.
y 2 = 4 x is y = mx - 2m - m 3 …(i)
\ (2t 2 ) 2 + ( 4t ) 2 - 2(2t 2 ) - 4( 4t ) = 0
As it passes through (9, 6), then
Þ 4t 4 + 12 t 2 - 16 t = 0
6 = 9m - 2m - m 3
Þ t(t - 1 ) (t 2 + t - 4 ) = 0 Þ t = 0 or 1
Þ m 3 - 7m + 6 = 0
For t = 0, we get point P, therefore t = 1 gives point Q as (2, 4).
Þ(m - 1 ) (m - 2 ) (m + 3 ) = 0
Here, P( 0, 0 ) and Q(2, 4 ) are end points of diameter of the given
Þ m = 1, 2, - 3 circle and focus of the parabola is the point S(2, 0 ).
From Eq. (i), equations of normals are \ ÐPSQ = 90°
y = x - 3, y = 2 x - 12 , y = - 3 x + 33 1
Þ y - x + 3 = 0, y - 2 x + 12 = 0, y + 3 x - 33 = 0 Hence, area of DPQS = ´ 2 ´ 4 = 4 sq units.
2
Chap 05 Parabola 457

Sol. (Q. Nos. 125 and 126) 2


128. Let the tangent to, y 2 = 8x be y = mx + .
Q PQ is the focal chord of y = 4ax. 2 m
2a ö Y R
æa
\ Coordinates of P and Q are (at 2, 2at ) and ç 2 , - ÷ .
èt t ø P
Y
P (at2,2at)
X′ X
T(−2, O
R 90° 0)
O Q
X′ θ X
S(a,0) Y′ S
Q If it is common tangent to parabola and circle, then
a ,– 2a
Y′ 2
t2 t y = mx + is a tangent to x 2 + y 2 = 2.
m
Tangents at P and Q are 2
ty = x + at 2 and ty = xt 2 + a \ m = 2
æ æ 1öö (1 + m 2 )
which intersect each other at R ç - a, a çt - ÷ ÷ .
è è t øø 4
Þ =2
As R lies on the line y = 2 x + a, a > 0 m 2(1 + m 2 )

\
æ 1ö
a çt - ÷ = - 2a + a Þ m4 + m2 - 2 = 0
è tø Þ (m 2 + 2 ) (m 2 - 1 ) = 0
1
Þ t - = -1 \ m=±1
t
\ Required tangents are y = x + 2 and y = - x - 2.
2
125. Q Slope of OP = and slope of OQ = - 2 t Their common point is T ( - 2, 0 ).
t
Chord of contact PQ to circle is
2
+ 2t x × (- 2) + y × 0 = 2
2 1
\ tan q = t = t+ Þ x = -1
1-4 3 t
Hence, coordinates of P and Q are ( - 1, 1 ) and ( - 1, - 1 ) and
2
chord of contact RS to parabola is
2 æ 1ö 2 é 1 ù y × 0 = 4 (x - 2)
= çt - ÷ + 4 = 5 êëQ t - t = - 1 úû
3 è tø 3 Þ x =2
Q q > 90° Hence, coordinates of R and S are (2, 4) and (2, - 4 ).
2 1
\ tan q = - 5 \Area of trapezium PQRS = (2 + 8 ) ´ 3 = 15 sq units
3 2
2 ìï æ 2 üï
æ 1ö 1ö Sol. (Q. Nos. 129 and 130)
126. PQ = a çt + ÷ = a í çt - ÷ + 4ý
è tø ïî è tø ïþ æ a - 2a ö
Q PQ is a focal chord, then Q ç 2 , ÷.
é 1 ù èt t ø
= a(1 + 4 ) = 5a êëQ t - t = - 1 úû
Also, QR || PK Þ mQR = mPK
127. Equation of tangent of y 2 = 4x in terms of slope is 2a
2ar +
1 Þ t = 0 - 2at
y = mx + …(i) a 2
m ar - 2 2a - at
2

Q Line Eq. (i) touches x 2 = - 32y t


2 - 2t
æ 1ö Þ =
Þ x 2 = - 32 çmx + ÷ 1 2 - t2
è mø r-
32 t
2
Þ x + 32mx + =0 …(ii) 1
m [Qr + ¹ 0, otherwise Q will coincide with R]
For touching roots of Eq. (ii) are equal. t
\ D=0 Þ 2 - t 2 = - rt + 1
æ 32 ö t2 - 1
Þ (32m ) 2 = 4 × 1 × ç ÷ \ r=
èmø t
1 t2 - 1
Þ m3 = 129. r =
8 t
\ m =1/2
458 Textbook of Coordinate Geometry

130. Tangent at P is ty = x + at 2 …(i) æt2 ö æt2 ö


134. Let P ç 1 , t1 ÷ and Q ç 2 , t 2 ÷ such that t1 > 0
3 è2 ø è2 ø
Normal at S is y + sx = 2as + as …(ii)
[Q P lies in first quadrant]
Putting the value of x from Eq. (i) in Eq. (ii), then
Q Circle with PQ as diameter passes through the vertex O( 0, 0 )
y + s (ty - at 2 ) = 2as + as 3 of the parabola.
Þ y + (st )y - a(st )t = 2as + as 3 \ ÐPOQ = 90°
2a a Þ Slope of OP ´ Slope of OQ = - 1
Þ y + y - at = + 3 [Q st = 1 ]
t t 2 2
Þ ´ = -1
æ 2 1ö t1 t 2
Þ 2y = a çt + + 3 ÷
è t t ø Þ t1t 2 = - 4 [Q t 2 < 0 ]
2 2
a(t + 1 ) 2
\ y = P t1 , t
2t 3 Y 2
1

131. Let any point Q on x 2 = 8y is ( 4t, 2 t 2 ) and given P (h, k ) divides


OQ in the ratio 1:3 (internally).
Y X′ X
O

3 Q (4t,2t2) Q
P Y′ 2
1 t2 , t
2
X 2
O
Now, area of DOPQ = 3 2
4t 2t 2
Then, h= = t and k = Þ 2k = h 2 t12
4 4 1 2 t1
Þ =3 2
\ Required locus of P is x 2 = 2y . 2 t 22
t2
132. End points of latusrectum of 2
y 2 = 4 x are (1, ± 2 ). 1
Þ t1t 2(t1 - t 2 ) = ± 3 2
4
Equation of normal to y 2 = 4 x at (1, 2) is
Þ t1 - t 2 = ± 3 2 [Q t1t 2 = - 4 ]
2
y - 2 = - (x - 1) 4
2 Þ t1 + = ± 3 2
t1
Þ x + y -3 = 0
4
As it is tangent to circle ( x - 3 ) 2 + (y + 2 ) 2 = r 2 or t1 + = 3 2 [Q t1 > 0 ]
t1
|3 -2 -3|
\ = r Þ r2 = 2 Þ t12 - 3 2t1 + 4 = 0
(1 + 1 )
3 2± 2
133. Let (t 2, 2 t ) be any point on y 2 = 4x. Let (h, k ) be image of (t 2, 2t ) \ t1 = = 2 2 , 2.
2
with respect to the line x + y + 4 = 0, then
\ Point P can be ( 4, 2 2 ) or (1, 2 ).
h - t 2 k - 2t - 2(t 2 + 2 t + 4 )
= = 135. Let P (2 t 2, 4t ) and C( 0, - 6).
1 1 1+1
Þ h = - (2 t + 4 ) and k = - (t 2 + 4 ) \ (CP ) 2 = 4 t 4 + ( 4 t + 6 ) 2 = z (say)
2 dz
æh + 4ö \ =0
Þ (k + 4 ) = - ç ÷ dt
è -2 ø
Þ 16t 3 + 2( 4t + 6 ) × 4 = 0
Þ (h + 4 ) 2 = - 4(k + 4 )
Þ t 3 + 2t + 3 = 0
2
Locus of (h, k ) is ( x + 4 ) = - 4(y + 4 ).
Þ (t + 1 ) (t 2 - t + 3 ) = 0
2
\ Curve C is ( x + 4 ) = - 4(y + 4 ) \ t = -1
Now, intersection of C with y = - 5, then Þ P(2, - 4 )
( x + 4 ) 2 = - 4( - 5 + 4 ) = 4 Equation of circle is
\ x + 4 = ± 2 Þ x = - 6,- 2 ( x - 2 ) 2 + (y + 4 ) 2 = (2 - 0 ) 2 + ( - 4 + 6 ) 2
\ A( - 6, - 5 ) and B( - 2, - 5 ) Þ x 2 + y 2 - 4 x + 8y + 12 = 0
\ AB = 4
Chap 05 Parabola 459

136. QC1 : x 2 + y 2 = 3 and parabola x 2 = 2y , then Alternate (c) Equation of normal at P( 4, 4 ) is


2
y + 2y - 3 = 0 Þ y = 1,- 3 4
y - 4 = - (x - 4)
\ P( 2, 1 ) [Q P lies in first quadrant] 2
Þ y - 4 = - 2x + 8
Now, tangent at P( 2, 1 ) on the circle C1 is
Þ 2 x + y = 12
x 2 + y =3 \ Intercept on X -axis is 6.
Let Q2 or Q3( 0, l ) Alternate (d) Slope of tangent at Q = Slope of tangent at
| 0 + l -3| 1
\ =2 3 P =
(2 + 1 ) 2
Þ | l -3| =6 138. Centre of circle
\ l = 9 or - 3 Y
Þ Q2( 0, - 3 ) and Q3( 0, 9 ).
y= (0, 4)
Alternate (a) Q2Q3 = 12 x
–x
r
Alternate (b) R2R3 = Length of external common tangent C r y=
M
= (Q2Q3 ) 2 - (2 3 + 2 3 ) 2 X′ X
O
= (144 - 48 = 4 6
1 | 0 + 0 - 3|
Alternate (c) Area of DOR2R3 = ´ R 2R 3 ´
2 (2 + 1 )
1 3
=
´4 6´ =6 2 Y′
2 3
1 C º ( 0, 4 - r )
Alternate (d) Area of DPQ2Q3 = ´ Q2Q3 ´ 2
2 Q CM = r
1 | 10 - ( 4 - r )|
= ´ 12 ´ 2 = 6 2 \ =r
2 2
137. Let P (t 2, 2 t ), S(2, 8) and r = ( 4 + 64 - 64) = 2 Þ 4 -r =r 2
We know that, shortest distance between two curves lies along 4
or r=
their common normal. The common normal will pass through 2+1
centre of circle.
= 4( 2 - 1 )
\ Slope of PS = Slope of normal to the parabola
139. (a) Equation of chord of parabola y 2 = 16x whose mid-point
y 2 = 4 x at P (t 2, 2t )
(h, k ) is
2t - 8
Þ = - t or t 3 = 8 Þ t = 2 T = S1
t2 - 2
or ky - 8( x + h ) = k 2 - 16h
\ P( 4, 4 )
or 8 x - ky = 8h - k 2 ...(i)
Alternate (a) SP = (2 - 4 ) 2 + (8 - 4 ) 2 = 2 5
Now comparing Eq. (i) and 2x + y = p, then
Alternate (b) SQ = r = 2
8 - k 8h - k 2
SQ SQ 2 = =
\ = = 2 1 p
QP SP - SQ 2 5 - 2
Þ k = - 4 and 4 p = 8h - k 2
1 ( 5 + 1) 5+1
= ´ = or k = - 4 and p = 2h - 4
( 5 - 1) ( 5 + 1) 4
Hence, p = 2, h = 3, k = - 4
Þ SQ : QP = ( 5 + 1 ) : 4

You might also like